`data interpretation replica questions that have appeared in cat in the last 4 years

63
1 DATA INTERPRETATION REPLICA QUESTIONS THAT HAVE APPEARED IN CAT IN THE LAST 4 YEARS TABLES Directions for questions 1 to 3: Answer the following questions based on the information given below. Telecom operators get revenue from transfer of data and voice. Average revenue received from transfer of each unit of data is known as ARDT. In the table below, the revenue received from data transfer as a percentage of total revenue received and the ARDT in US Dollars (USD) are given for various countries. Country ARDT (in USD) Revenue from data transfer as a percentage of total revenue Japan 13 70 India 1 9 Brazil 2 8 Thailand 1 11 Israel 4 13 Hungary 2 15 Ireland 14 23 Russia 1 18 China 2 25 Indonesia 2 42 Philippines 3 54 UK 13 30 Spain 7 14 Sweden 6 18 Poland 6 22 Germany 7 24 South Korea 10 24 Norway 11 20 USA 11 17 Singapore 9 21 1. It was found that the volume of data transfer in India is the same as that of Singapore. Then which of the following statements is true? (1) Total revenue is the same in both countries. (2) Total revenue in India is about 2 times that of Singapore. (3) Total revenue in India is about 4 times that of Singapore. (4) Total revenue in Singapore is about 2 times that of India. (5) Total revenue in Singapore is about 4 times that of India. 2. It is expected that by 2010, revenue from data transfer as a percentage of total revenue will triple for India and double for Sweden. Assume that in 2010, the total revenue in India is twice that of Sweden and that the volume of data transfer is the same in both the countries. What is the percentage increase in ARDT in India if there is no change in ARDT in Sweden? (1) 400% (2) 550% (3) 800% (4) 750% (5) Cannot be determined 3. If the total revenue received is the same for the pairs of countries listed in the choices below, choose the pair that has approximately the same volume of data transfer. (1) Philippines and Brazil (2) South Korea and Poland (3) Germany and USA (4) UK and Spain (5) Isreal and Norway Directions for questions 4 to 6: Answer the following questions based on the information given below. For admissions to its postgraduate programme, six institutes conducted a common admission test. The test had four sections with the maximum marks in each section being 50. The following tables gives the aggregate as well as sectional cut-off marks fixed by the six institutes. A student will get interview calls only if he/she gets marks higher than or equal to the cut-off marks in each of the sections and his aggregate marks are at least equal to the aggregate cut-off marks as specified by the institute. Sectional cut-off marks Section A Section B Section C Section D Aggregate cut-off marks Institute 1 41 41 41 172 Institute 2 44 44 171 Institute 3 45 167 Institute 4 42 44 174 Institute 5 44 42 176 Institute 6 40 43 172 4. Arun got calls from all colleges. What could be the minimum aggregate marks obtained by him? (1) 176 (2) 177 (3) 192 (4) 172 (5) 180 5. Bala got calls from two colleges. What could be the minimum marks obtained by him in a section? (1) 0 (2) 17 (3) 21 (4) 31 (5) 37 6. Chandu did not get a call from even a single college. What could be the maximum aggregate marks obtained by him? (1) 179 (2) 174 (3) 182 (4) 194 (5) 188

Upload: alexander-bennett

Post on 20-Dec-2015

59 views

Category:

Documents


6 download

DESCRIPTION

cgffytfyvytvy

TRANSCRIPT

Page 1: `DATA INTERPRETATION REPLICA QUESTIONS THAT HAVE APPEARED IN CAT IN THE LAST 4 YEARS

1

DATA INTERPRETATION REPLICA QUESTIONS THAT HAVE APPEARED IN CAT IN THE LAST 4 YEARS

TABLES

Directions for questions 1 to 3: Answer the following questions based on the information given below. Telecom operators get revenue from transfer of data and voice. Average revenue received from transfer of each unit of data is known as ARDT. In the table below, the revenue received from data transfer as a percentage of total revenue received and the ARDT in US Dollars (USD) are given for various countries.

Country ARDT (in USD)

Revenue from data transfer as a

percentage of total revenue

Japan 13 70 India 1 9 Brazil 2 8 Thailand 1 11 Israel 4 13 Hungary 2 15 Ireland 14 23 Russia 1 18 China 2 25 Indonesia 2 42 Philippines 3 54 UK 13 30 Spain 7 14 Sweden 6 18 Poland 6 22 Germany 7 24 South Korea 10 24 Norway 11 20 USA 11 17 Singapore 9 21

1. It was found that the volume of data transfer in India is the same as that of Singapore. Then which of the following statements is true? (1) Total revenue is the same in both countries. (2) Total revenue in India is about 2 times that of

Singapore. (3) Total revenue in India is about 4 times that of

Singapore. (4) Total revenue in Singapore is about 2 times

that of India. (5) Total revenue in Singapore is about 4 times

that of India. 2. It is expected that by 2010, revenue from data

transfer as a percentage of total revenue will triple for India and double for Sweden. Assume that in 2010, the total revenue in India is twice that of Sweden and that the volume of data transfer is the same in both the countries. What is the percentage increase in ARDT in India if there is no change in ARDT in Sweden?

(1) 400% (2) 550% (3) 800% (4) 750% (5) Cannot be determined 3. If the total revenue received is the same for the

pairs of countries listed in the choices below, choose the pair that has approximately the same volume of data transfer.

(1) Philippines and Brazil (2) South Korea and Poland (3) Germany and USA (4) UK and Spain (5) Isreal and Norway

Directions for questions 4 to 6: Answer the following questions based on the information given below. For admissions to its postgraduate programme, six institutes conducted a common admission test. The test had four sections with the maximum marks in each section being 50. The following tables gives the aggregate as well as sectional cut-off marks fixed by the six institutes. A student will get interview calls only if he/she gets marks higher than or equal to the cut-off marks in each of the sections and his aggregate marks are at least equal to the aggregate cut-off marks as specified by the institute.

Sectional cut-off marks Section A Section B Section C Section D

Aggregate cut-off marks

Institute 1 41 41 41 172 Institute 2 44 44 171 Institute 3 45 167 Institute 4 42 44 174 Institute 5 44 42 176 Institute 6 40 43 172

4. Arun got calls from all colleges. What could be

the minimum aggregate marks obtained by him? (1) 176 (2) 177 (3) 192 (4) 172 (5) 180 5. Bala got calls from two colleges. What could be

the minimum marks obtained by him in a section?

(1) 0 (2) 17 (3) 21 (4) 31 (5) 37 6. Chandu did not get a call from even a single

college. What could be the maximum aggregate marks obtained by him?

(1) 179 (2) 174 (3) 182 (4) 194 (5) 188

Page 2: `DATA INTERPRETATION REPLICA QUESTIONS THAT HAVE APPEARED IN CAT IN THE LAST 4 YEARS

2

Directions for questions 7 to 9: Answer the following questions based on the information given below. There are 100 employees in an organisation across five departments. The following table gives the department-wise distribution of average age, average basic pay and allowances. The gross pay of an employee is the sum of his/her basic pay and allowances.

Department Number of Employees

Average age (years)

Average basic pay (Rs)

Allowances (% of Basic pay)

HR 5 46 10000 60

Marketing 3 36 12000 70

Finance 20 31 13000 50

Business Development 35 43 15000 65

Maintenance 10 36 11000 40 There are limited number of employees considered for transfer/promotion across departments. Whenever a person is transferred/promoted from a department of lower average age to a department of higher average age, he/she will get an additional allowance of 10% of basic pay over and above his/her current allowance. There will not be any change in pay structure if a person is transferred/promoted from a department with higher average age to a department with lower average age. Questions below are independent of each other. 7. What is the approximate percentage change in

the average gross pay of the HR department due to transfer of a 40 year old person with basic pay of `16000 from the marketing department?

(1) 9% (2) 11% (3) 13% (4) 15% (5) 17% 8. There was a mutual transfer of an employee

between Marketing and Finance departments and transfer of one employee from Marketing to HR. As a result, the average age of Finance department increased by one year and that of Marketing department remained the same. What is the new average age of the HR department?

(1) 31 (2) 36 (3) 41 (4) 46 (5) Cannot be determined 9. If two employees (each with a basic pay of

`12000) are transferred from Maintenance department to HR department and one person (with a basic pay of `16000) was transferred from Marketing department to HR department, what will be the percentage change in average basic pay of HR department?

(1) 10.5% (2) 12.5% (3) 15% (4) 30% (5) 40% Directions for questions 10 to 13: Answer the following questions based on the information given below. An athletics coach was trying to make an energy drink which is best suited for the athletes. For this purpose he took five of the best known energy drinks – A, B, C, D and E in the market with the idea of mixing them to get the result he desired. The table below gives the composition of these drinks. The cost of each of these energy drinks per litre is A – 150, B – 50, C – 200 D – 500 and E – 100.

Composition Energydrink Carbohydrate

(%) Protein

(%) Fat (%)

Minerals(%)

A 50 30 10 10

B 80 20 0 0

C 10 30 50 10

D 5 50 40 5

E 45 50 0 5 10. For the sprinters, he has to prepare a drink

containing 10% minerals and at least 30% protein. In how many different ways can we prepare this drink by mixing at least two ingredients?

(1) One (2) Two (3) Three (4) Four (5) Five 11. Which among the following is the combination

having the lowest cost per unit for a drink having 10% fat and at least 30% proteins? The drink has to be formed by mixing two ingredients together. (1) B and C (2) B and E (3) B and D (4) C and E (5) D and E

12. In what proportion should B, C and E be mixed

to make a drink having at least 60% carbohydrate at the lowest cost per unit? (1) 2 : 1 : 3 (2) 4 : 1 : 2 (3) 2 : 1 : 4 (4) 3 : 1 : 2 (5) 4 : 1 : 1

13. A drink containing 30% each of carbohydrate and

protein, no more than 25% fat and at least 5% minerals is to be made. Which of the following two drinks must be mixed in equal quantities to obtain the required drink? (1) A and B (2) D and E (3) B and E (4) C and D (5) A and E

Page 3: `DATA INTERPRETATION REPLICA QUESTIONS THAT HAVE APPEARED IN CAT IN THE LAST 4 YEARS

3

Directions for questions 14 to 17: Answer the following questions based on the information given below: The following table shows the breakup of actual costs incurred by a company in the last five years (year 2004 to year 2008) to produce a particular product.

2004 2005 2006 2007 2008 Volume of production and sale (units) 1500 1350 1650 1800 1800 Costs (Rs) Material 75000 67650 82800 89850 90000 Labour 30000 27000 33150 36225 36000 Consumables 3000 3300 2700 2400 2100 Rent of building 1500 1500 1650 1650 1800 Rates and taxes 600 600 600 600 600 Repair and maintenance expenses 1200 1230 1170 1185 1200 Operating cost of machines 45000 40500 50250 54030 54000 Selling and marketing expenses 8625 8700 8700 8625 8700

The production capacity of the company is 3000 units. The selling price for the year 2008 was `125 per unit. Some costs change almost in direct proportion to the change in the volume of production, while others do not follow any obvious pattern to change with respect to the volume of production and hence are considered fixed. Using the information provided for the year 2008 as the basis for projecting the figures for the year 2009, answer the following questions. 14. What is the approximate cost per unit in rupees, if

the company produces and sells 2100 units in the year 2009? (1) 104 (2) 107 (3) 110 (4) 115 (5) 116

15. What is the minimum number of units that the

company needs to produce and sell to avoid any loss? (1) 470 (2) 525 (3) 576 (4) 1120 (5) 1392

16. If the company reduces the price by 5%, it can

produce and sell as many units as it desires.

How many units should the company produce to maximize its profit? (1) 2100 (2) 2400 (3) 2700 (4) 2850 (5) 3000

17. Given that the company cannot sell more than

2550 units, and it will have to reduce the price by `5 for all units, if it wants to sell more than 2100 units what is the maximum profit, in rupees, that the company can earn? (1) 38100 (2) 36600 (3) 1800 (4) 1900 (5) 2000

Directions for questions 18 to 21: Answer the following questions based on the information given below. The table below shows the comparative costs, in US Dollars, of certain items in USA and a select few Asian countries. The models considered are the most popular ones in the respective countries.

Comparative Costs in USA and some Asian countries (in US Dollars) Product

USA India Thailand Singapore Malaysia LCD TV 2300 1000 1100 1850 900 Home gym 1900 900 1000 1250 900 Refrigerator 2000 1100 1300 1300 1100 Air conditioner 2300 900 1200 1200 1000 Washing machine 1200 300 450 600 300 Music system 2000 850 1000 1300 800 Digital camera 1600 550 700 900 600

The equivalent of one US Dollar in the local currencies is given below.

I US Dollar equalent India 40.93 Rupees Malaysia 3.51 Ringits Thailand 32.90 Bahts Singapore 1.53 S Dollars

A consulting firm found that the quality of the products was not the same in all the countries above. A poor quality product would result in higher servicing costs over its life time. The cost of poor quality of the products is given in the table

Page 4: `DATA INTERPRETATION REPLICA QUESTIONS THAT HAVE APPEARED IN CAT IN THE LAST 4 YEARS

4

Comparative cost of poor quality in USA and Asian countries (in US Dollars) Products

USA India Thailand Singapore Malaysia

LCD TV 0 300 300 200 400

Home gym 0 500 400 500 500

Refrigerator 0 500 500 400 600

Air Conditioner 0 700 500 500 800

Washing machine 0 500 600 500 400

Music system 0 900 600 400 400

Digital camera 0 500 600 500 600 Note: For all questions assume that the models considered are the ones for which the prices are mentioned. 18. A US citizen requires a refrigerator, air

conditioner and a music system. He can buy them through an internet portal from any of the given countries without having to pay for any transportation costs. Which country will result in the cheapest overall cost, taking cost of poor quality into account?

(1) India (2) Thailand (3) Malaysia (4) Singapore (5) USA 19. Taking the cost of poor quality into account,

which country/countries will be the most expensive if one has to buy a music system? (1) India (2) Thailand (3) Malaysia (4) Singapore (5) India and Singapore

20. Approximately what difference in Bahts will it

make to a Thai citizen who is touring India if she were to get a washing machine from India instead of her native country, taking into account the cost of poor quality? One has to pay a duty of 1500 Bahts for transporting the washing machine from India to Thailand.

(1) 2350 (2) 4050 (3) 5150 (4) 6725 (5) 7500 21. The rupee values increases to `35 for a US

Dollar, and all other things including quality, remain the same. What is the approximate difference in cost, in US Dollars, between Singapore and India for a digital camera, taking this change into account? (1) 70 (2) 250 (3) 450 (4) 800 (5) No difference

Directions for questions 22 to 26: Answer the following questions based on the information given below. An all India tour operator connects ten cities A to J. The table below gives the distance between a pair of cities and the corresponding price charged by the operator. Travel is permitted only from the city of departure to the city of arrival. The customers do not travel by a route when they have to stop at more than two intermediate cities.

Sl. No

City of Departure

City of Arrival

Distance between the two cities. (in kms)

Fare (Rs).

1 A B 280 335 2 A C 395 675 3 A D 425 625 4 A E 625 800 5 A F 672 850 6 A G 675 1225 7 A H 975 925 8 B C 825 1000 9 B H 875 950 10 B I 1050 1225 11 B J 1150 1135 12 C D 230 225 13 C F 205 215 14 C G 455 550 15 D E 270 295 16 D F 312 350 17 D G 320 375 18 D H 475 625 19 D J 825 1225 20 E F 625 850 21 E G 485 575 22 E H 425 435 23 F G 450 525 24 F I 437 475 25 F J 485 575 26 G I 255 275 27 G J 415 445 28 H I 395 485 29 H J 200 210 30 I J 230 270

22. What is the lowest price, in rupees, a passenger has

to pay for traveling by the shortest route from A to J? (1) 1135 (2) 1425 (3) 1445 (4) 1465 (5) 1670 23. The operator plans to introduce a direct service

between cities A and J. The market research results indicate that all its existing passengers traveling between A and J will use the direct service if it is priced 5% below the minimum price that they pay at present. What should the operator charge, approximately, in rupees, for the direct service? (1) 995 (2) 1078 (3) 1349 (4) 1372 (5) 1392

Page 5: `DATA INTERPRETATION REPLICA QUESTIONS THAT HAVE APPEARED IN CAT IN THE LAST 4 YEARS

5

24. If cities C, D and H are not accessible due to security reasons, what would be the minimum price, in rupees, to be paid by a passenger traveling from A to J?

(1) 1140 (2) 1310 (3) 1425 (4) 1475 (5) 1545 25. If the prices include a margin of 10% over the

total cost that the operator incurs, what is the minimum cost per kilometer that the operator incurs for the service from A to J? (1) 0.77 (2) 0.88 (3) 0.99

(4) 1.06 (5) 1.08 26. If the prices include a margin of 15% over the

total cost that the company incurs, which among the following is the distance to be covered in traveling from A to J that minimizes the total cost per kilometer for the operator? (1) 1085 (2) 1090 (3) 1160

(4) 1175 (5) 1195 Directions for questions 27 to 30: Answer the following questions based on the information given below: The proportion of male employees and the proportion of post graduates in a company are given below. The company has a total of 800 employees, 80% of whom are in the production department and the rest equally divided between the marketing and the accounts department.

Department Male Post graduates Marketing 0.60 Accounts 0.55 0.50 Production 0.55 Total 0.475 0.53

27. What is the percentage of male employees in the production department? (1) 40 (2) 45 (3) 50 (4) 55 (5) 60

28. In the marketing department, twenty five per cent

of the post graduates are male. What is the difference between the number of female post graduates and male employees who are not post graduates?

(1) less than 8 (2) 10 (3) 12 (4) 14 (5) 16 29. What percentage of employees in the marketing

department are post graduates? (1) 40 (2) 45 (3) 50 (4) 55 (5) 60

30. In the production department, 50% of the males

are post graduates. Which of the following statements is correct? (1) Except post graduate males, all other groups

have the same number of employees. (2) Except males who are not post graduates, all

other groups have the same number of employees.

(3) Except post graduates females, all other groups have the same number of employees.

(4) Except females who are not post graduates, all the other groups have the same number of employees.

(5) All of the above groups have the same number of employees.

Directions for questions 31 to 33: Buziki motors, a two wheeler manufacturer, introduced a variant in the 125cc category in the beginning of 2009. The number of bookings received in a city for a period 12 months is as given below.

Q1 Q2 Q3 Q4 Month Jan Feb March April May June July Aug Sep Oct Nov Dec Number of bookings 346 412 380 450 308 359 462 333 345 250 506 370 Number of deliveries 200 362 356 445 395 297 496 300 249 420 403

The company promises the delivery of the vehicles within a span of two months i.e., a booking made in January will be delivered in February or March. The entire period of 12 months is divided into 4 quarters. The price of the motorcycle was increased every quarter. The customer had to pay the total amount at the time of the booking.

Quarter Price (in Rs) per motor cycle Q1 42000 Q2 42500 Q3 43100 Q4 44000

31. In which quarter was the average number of

bookings per month, the highest? (A) Q1 (B) Q2 (C) Q3 (D) Q4

32. The number of deliveries made in December from the bookings made in November is how many times the number of deliveries made in August from the bookings made in June? (A) 1.39 (B) 1.58 (C) 2.38 (D) 2.58

33. What is the highest revenue (in `crore) obtained

by the company from the sales of the new 125cc variant in any of the four quarters? (A) 4.91 (B) 4.95 (C) 4.99 (D) 5.05

Page 6: `DATA INTERPRETATION REPLICA QUESTIONS THAT HAVE APPEARED IN CAT IN THE LAST 4 YEARS

6

Directions for questions 34 to 36: The following table gives the relation between the scaled scores of three teams and the number of wins in the matches played.

Scaled score Number of wins Team A Team B Team C

10 – 12 18 19 20 13 – 15 22 22 24 16 – 18 28 36 36 19 – 21 30 27 39 22 – 24 41 51 46 25 – 27 68 64 63 28 – 30 62 51 52 31 – 33 94 99 95

34. If the scaled score for two of the teams is the

same and greater than that of the third team, then which of the following could be the range for the ‘number of wins’? (A) 13 – 15 (B) 16 – 18 (C) > 16 (D) Either (A) or (C)

35. For which of the following range for the ‘number

of wins’ is the difference between the scaled scores of team A and team B as a percentage of that of team C, the 3rd least? (A) 19 – 21 (B) 10 – 12 (C) 31 – 33 (D) None of these

36. If average scaled score of the three teams is 2.5

times the average of the corresponding ‘number of wins’, Then which of the following could be the range for the ‘number of wins’. (A) 25 – 27 (B) 31 – 33 (C) 22 – 24 (D) All the three

Directions for questions 37 to 39: A stock brokerage firm accepts the investments and places the amounts invested in seven different companies under three schemes. Scheme X is applicable for an investment of 5 to 10 lakh rupees, scheme Y is applicable for an investment of 11 to 20 lakh rupees and scheme Z is applicable for an investment of 21 to 40 lakh rupees.

Percentage of investments in the seven companies under the different schemes are as follows:

Companies Scheme X

Scheme Y

Scheme Z

GB Holdings 5% 20% 15% Solar Computers 12% 2.5% 10% NLP Industries 16% 12.5% 7.5% Techies’ Technologies 10% 30% 16

32

%

Wiz craft solutions 25% 10% 15% OBCC bank 15% 10% 22.5% Live life mutual funds

17% 15% 1331

%

The brokerage firm promises a rate of return of 2% on scheme X, 2.5% on Scheme Y and 3% on Scheme Z. Assume that the rate of return on each of the companies in a scheme is same as the overall rate of return of the scheme. 37. If Mahesh invested `12 lakh in the stock

brokerage farm, but he immediately withdrew an amount of `3 lakh, then what is the percentage change in the investment in NLP Industries because of the withdrawal? (A) 2% (B) 2.5% (C) 3% (D) 4%

38. Mr. Anil and Ms. Shivani invests an amount of

`7 lakh and `13 lakh respectively. What is the difference (in `) in their returns on investment made in Wizcraft solutions? (A) 250 (B) 300 (C) 400 (D) More than 500

39. If three persons A, B and C make investments in the ratio of 10 : 20 : 21, such that their investments fall under the schemes X,Y and Z respectively, then what is the increase in their combined return on investment (in `) if the firm increases the rate of return on the schemes X, Y and Z by 10%, 20%, and 10% respectively? (A) 16100 (B) 16300 (C) 17300 (D) None of these

Directions for questions 40 and 41: The following table gives the numbers of music payers, of two companies - NOSY and BOSS, sold in Delhi across three years.

2001 2002 2003 Type of music system NOSY BOSS NOSY BOSS NOSY BOSS

Mono speaker 1000 1600 3000 2700 2600 4000 Dual speaker – 1000w 1800 2000 3100 2500 4400 3400 Dual speaker – 2000w 2300 1200 2900 3200 3600 4200 Four speaker – 5000w 1400 2200 3000 4200 4600 3800 Home theatre 1600 2400 3200 2400 4000 5000 Total 8100 9400 15200 15000 19200 20400

40. From 2001 to 2003, for how many types of music

systems is there an increase in the percentage contribution for each of the two companies? (A) 0 (B) 1 (C) 2 (D) More than 2

41. From 2001 to 2003, which type of music system has shown the maximum change in percentage

points in its contribution to the total sales of the company NOSY? (A) Mono speaker (B) Dual speaker – 1000w (C) Dual speaker – 2000w (D) Four speaker – 5000w

Page 7: `DATA INTERPRETATION REPLICA QUESTIONS THAT HAVE APPEARED IN CAT IN THE LAST 4 YEARS

7

Directions for question 42: The following table gives the break up of the number of cars sold by two showrooms in the first 10 days of their opening.

Day Showroom A Showroom B 1 16 18 2 20 19 3 35 26 4 30 42 5 33 39 6 24 29 7 51 48 8 63 52 9 60 71 10 79 81

42. Which of the following statement (s) is/are ‘true’? Ι. The total number of cars sold by showroom A

at the end of 7 days lies between 90% and 110% of that sold by showroom B.

ΙΙ. In the given period, the total number of cars sold by showroom A on odd numbered days is less than 90% of that sold by showroom B on even numbered days.

(A) Only Ι (B) Only ΙΙ. (C) Both Ι and ΙΙ (D) Neither Ι nor ΙΙ.

Directions for questions 43 and 44: The following data gives the details of the establishment fee and average number of customers estimated (per year) in franchises of a restaurant chain, Foodies, in class A and class B centers of different cities in India.

City Est. fee in class A centers (in ` lakh)

Est. fee in class B centers (in ` lakh)

Estimated customers peryear in class A centers

Estimated customers per year in class B

centers Hyderabad 126 75 51,860 42,500

Bengaluru 144 90 60.200 50.246

Pune 132 104 70,000 52,400

Chennai 125 95 48,800 40,000

Kolkata 115 65 45,500 37,000 43. If each customer spends an average amount of

`240 in restaurants in class A centers and `180 in class B centers, then in which of the given cities will the franchise earn revenues more than the establishment fees (for each type of centre), in one year? (A) Hyderabad (B) Pune (C) Chennai (D) Bengaluru

44. If a person owns two franchises of Foodies in

Pune, one in a class A center and the other in a class B center, and it is estimated that the average amount each person spends in class A and class B centers are `300 and `130 respectively, then find the minimum number of customers, who are required to come to the two restaurants together in the first year, such that the revenues are not less than the establishment fees for each? (A) 12.4 lakh (B) 1.24 lakh (C) 2.4 lakh (D) 9.6 lakh

Directions for questions 45 and 46: The number of dropouts from primary schools, as a percentage of the total enrollments in a year in five districts across six years are given below. The values represented by ‘–‘ are unknown. Districts 2001 2002 2003 2004 2005 2006

P 52.3 51.0 45.6 43.9 42.0 40.8 Q 52.4 53.2 54.1 57.3 61.3 61.1 R 45.9 46.2 – 44.5 43.0 – S 36.5 – 37.4 – 38.2 39.6 T 41.2 43.4 42.6 44.5 44.1 45.0

45. Which of the following is definitely false? (A) For more than one of the five districts the

percentages are continuously increasing or decreasing.

(B) In the given period, the number of dropouts is the highest in 2006 for at least two districts

(C) The total number of dropouts as a percentage of total number of enrolments, in the given period, is not the highest for Q if it is given that the districts R and S have registered highest percentages in 2002 and 2006 respectively.

(D) In the given period, the maximum number of dropouts in all the five districts combined was registered in 2006, if it is given that R has registered its maximum percentage in 2006 and in any given year each of the five districts have equal number of enrolments in primary schools.

46. If the percentage of dropouts in a year has

decreased with respect to that in the previous year, than it is considered as an ‘achievement’ for a district. What is the minimum number of such ‘achievements’, in the given period, for all the five districts combined?

(A) 5 (B) 10

(C) 11 (D) More than 12

Page 8: `DATA INTERPRETATION REPLICA QUESTIONS THAT HAVE APPEARED IN CAT IN THE LAST 4 YEARS

8

Directions for questions 47 and 48: The following table gives the temperatures in six cities at three different times of the day.

City 5.00 a.m. 12 noon 6.00 p.m. P 24 42 29 Q 25 46 28 R 25 44 32 S 32 49 31 T 30 46 30 U 22 44 26

47. If the increase in temperature from 5 a.m. to

12 noon is linear, then what is the temperature in city Q at 10 a.m.?

(A) 30°C (B) 32°C (C) 36°C (D) 40°C 48. If the decrease in temperature from 12 noon to

6.00 p.m. is linear, then in which city was the temperature the highest at 3.30 p.m.?

(A) Q (B) R (C) S (D) U Directions for questions 49 to 52: The following table gives the details about the traffic flow on a particular day through the roads connecting six different cities. For example, 846 vehicles travel from P to Q and 964 travel from Q to P. So, total traffic on the road connecting the cities A and B is 846 + 964 = 1810. Assume that these are the only cities interconnected and traffic flow is among these cities only.

P Q R S T U P – 846 808 400 472 820 Q 964 – 564 540 840 844 R 536 664 – 248 888 200 S 848 242 624 – 478 428 T 484 364 784 672 – 648 U 672 496 528 992 372 –

49. The maximum traffic flow occurs on the road

connecting which two cities? (A) P – Q (B) P – R

(C) T – R (D) None of these 50. The total number of vehicles passing through the

road connecting any two of the given cities is the second least for

(A) S – U (B) R – S (C) P – T (D) None of these

51. Traffic flowing from which of the given cities is the maximum?

(A) P (B) Q (C) T (D) U 52. What is the least difference between the traffic

flowing from a particular city and traffic flowing to the same city?

(A) 120 (B) 158 (C) 98 (D) 232 Directions for questions 53 and 54: The following table gives the different payments options for a `1 lakh loan provided by a rural bank to persons with different income levels, depending on the time in which they would repay the loan.

Payment options (interest amount in `) Income level

(in ‘000 Rs) 1 year 2 years 3 years

20 – 40 2,000 3,000 4,000

41 – 60 2200 3,600 4,100

61 – 80 2,600 3,800 4,400

81 – 100 2,900 4,000 4,800 A person with an income between `41,000 to `60,000 has to repay the loan of `1 lakh with an interest amount of `2,200 if the loan is repaid in 1 year, `3,600 if the loan repaid in 2 years and so on. The interest amount to be paid gets multiplied by the factor of loan amount (for example a loan of `3.4 lakh lent to a person with an income between `20,000 to `40,000, in one year, accumulates to an amount (in `) of 3,40,000 + 3.4 × 2,000 = 3,46,800.) 53. Mr. A, whose annual income is `45,000 takes a

loan of `2.2 lakh and promises to repay it in 2 years. Mr. B, whose annual income is `76,000 takes a loan of `3.6 lakh and promises to repay it in 2 years. What is the difference in the interest amounts (in `) they have to pay?

(A) 5,440 (B) 5,760 (C) 6,120 (D) 6.260 54. A person with an annual income of `96,000 takes

two loans-one of `5.6 lakhs for 3 years and another of `6.4 lakh for 2 years. What is the average of the interest amount paid by him?

(A) `24,840 (B) `26,000 (C) `26,240 (D) `28,140

Directions for questions 55 and 56: The following table gives the distribution of number of male employees and female employees owning a four wheeler or a two wheeler or both in three companies X, Y and Z.

Company Four Wheeler Two wheeler Neither Male employees 45% 65% 5% X Female employees 30% 70% 10% Male employees 50% 60% 20% Y Female employees 25% 80% 15% Male employees 36% 54% 20% Z Female employees 24% 67% 9%

The number of male employees in company X is 70% of the total number of employees in the company and the number of female employees in company Y and Z is 40% each of the total number of employees of their respective companies.

Page 9: `DATA INTERPRETATION REPLICA QUESTIONS THAT HAVE APPEARED IN CAT IN THE LAST 4 YEARS

9

55. In which company is the percentage of employees who own both two wheeler and four wheeler, the highest?

(A) X (B) Y (C) Z (D) Cannot be determined

56. What percentage of the male employees in

companies Y and Z together own either a four wheeler or a two wheeler but not both, if it is given that the total number of employees in both the companies is the same?

(A) 40% (B) 50% (C) 80% (D) None of these Directions for questions 57 and 58: The following table gives the details about the percentage distribution of the total bikes sold by ACE Motors Ltd. The percentage distribution was the same in 2007 and 2008. The total number of bikes sold in 2007 is 1,50,000 which is the same as that in 2008.

Model Percentage of total bikes sold RL-100 13% BCZ 25% Thunder 20% WB-150 30% Muzzle 12%

The table below shows the revenue of the company from each of the five models in both the years.

Model Revenue

(in ` crore) In 2007

Revenue (in ` crore)

In 2008 RL-100 78 87.75 BCZ 93.75 105 Thunder 93 105 WB-150 90 103.5 Muzzle 93.6 99

57. For which model is the percentage increase in

the average selling price per bike, the highest? (A) RL-100 (B) Thunder (C) WB-150 (D) Muzzle.

58. What is the average of the selling prices (in `) of

the five models in 2007? (A) 37,600 (B) 33,600 (C) 32,400 (D) None of these.

Directions for questions 59 and 60: The following table gives the performance of four companies, all listed on National Stock Exchange (NSE), from 2001 to 2010.

Company Ι Company ΙΙ Company ΙΙΙ Company ΙV

Year Share price

(in `) Dividend

(in `) Share price

(in `) Dividend

(in `) Share price

(in `) Dividend

(in `) Share price

(in `) Dividend

(in `)

2001 128 120 283 112 148 128 400 128

2002 132 116 289 115 152 132 420 124

2003 126 123 295 128 163 145 432 136

2004 148 152 296 138 168 148 440 144

2005 158 148 312 142 172 140 453 148

2006 123 121 328 154 184 152 448 132

2007 172 113 324 132 196 128 432 120

2008 164 109 345 106 212 136 464 112

2009 128 105 360 121 252 138 484 128

2010 132 102 364 143 286 140 496 132 For any company,

Gx = Dx +

−+ −+

2

P2PP 1x1xx

Where Gx is gain an company;s share in yea x. Dx is the divided declared by the company in year x. Px and Px + 1 are share prices of the company in year x and x + c respectively. 59. What was the gain from the shares of company

IV in 2006? (1) 116 (2) 132 (3) 148 (4) 138

60. What was the highest percentage increase in the gain from the shares of company III in a year with respect to the previous year?

(1) 13.48% (2) 29.38% (3) 16.32% (4) 38.45%

Page 10: `DATA INTERPRETATION REPLICA QUESTIONS THAT HAVE APPEARED IN CAT IN THE LAST 4 YEARS

10

61. The following table gives the per capita income of countries in the year 2004. From the table determine the number of countries having their per capita income more than 40% of the median per capita income of these countries? (1) 9 (2) 10 (3) 11 (4) 8

Per capita income (gross) in US $ Switzerland 24,369 New Zealand 15,350 Lithuania 4,965 Romania 2,916 Spain 11,692 Sweden 13,746 United States 23,484 France 13,477 Mexico 3,523 Hong Kong 10,372 United Kingdom 19,207 Brazil 5,663 Germany 24,337

Directions for question 62: Bolvo bus service has the following revenue data (in ` crore) regarding its operations in 2007.

A/c sleeper

A/c semisleeper

Non A/c semi-

sleeper

Non A/C general Total

Inter-state services

Intra-state services 240 2880

Total 1200

A/c sleeper and A/C semi-sleeper accounted for 37.5% of the total revenue whereas non A/c semi-sleeper accounted for 25% of the total revenue. 50% of Intra-state services revenue was generated from non A/c general services. 60% of the total revenue generated was from Intra-state services. The revenue generated from A/c sleeper in inter-state services to that in intra-state services was in the ratio of 1 : 2 62. What was the total revenue (in ` crore) generated

from non A/c general in Intra-state services? (1) 600 (2) 1440 (3) 950 (4) 600

Directions for question 63: 63. The following table gives list of the major coal

producing countries in the world along with the total production of coal in the year 2007(in million tonnes)

Country Coal production China 2536.7 Russia 314.2 Poland 145.8 Ukraine 76.3 India 478.2 Turkey 76.6 United States 1039.2 Australia 393.9 Germany 201.9 Indonesia 174.9

What percent of the coal production of the top four (i.e, the four highest) countries is the coal production of the bottom four (i.e. the four lowest) countries? (1) 10.66% (2) 8.98%

(3) 11.32% (4) 12.45% Directions for questions 64 and 65: The following table gives the gender, height, weight and age of fifteen students of a college who have cleared the preliminary round of the selection process for being selected in the Air Force. The names of the students being denoted by A, B, C, D, . . . . . and O.

Names Gender Height (in cm)

Weight (in kg)

Age (in years)

1 A M 164 64 22 2 B M 169 62 21 3 C F 152 49 17 4 D M 148 68 18 5 E F 154 78 21 6 F M 172 68 23 7 G M 168 67 27 8 H M 165 70 22 9 I F 145 58 20

10 J M 152 78 18 11 K M 156 64 18 12 L F 146 51 19 13 M F 138 56 23 14 N M 171 67 24 15 O F 162 60 25 Air Force follows a selection criterion where the height and weight of a person must lie in one of the following ranges: Male

HEIGHT (in cm) WEIGHT (in kg) 155 – 160 58 – 62 160 – 165 60–64

165 – 170 63–68

170 – 175 66–74 Candidates whose height is less than 155 cm or more than 175 cm, are not eligible for selection. Female

HEIGHT (in cm) WEIGHT (in kg)

145 – 150 48 – 52

150 –155 52 –56

155– 160 55 –59

160 – 165 58 – 62 Candidates with weight less than 145 cm or more than 165 cm are not eligible for selection. 64. Find the ratio of the number of male and female

students who were eligible for selection. (1) 2 : 1 (2) 5 : 2 (3) 3 : 2 (4) 4 : 1

Page 11: `DATA INTERPRETATION REPLICA QUESTIONS THAT HAVE APPEARED IN CAT IN THE LAST 4 YEARS

11

65. If x represents the number of male students whose age lies in the range of 18 to 22 years (both inclusive) and y represents the number of females students whose weight lies in the range of 49 to 58 kg (both inclusive) then (1) x > 2y (2) x < y (3) x = y (4) 2x = 3y

Directions for questions 66 and 67: The following table gives the cost and revenue of a company for a period of five years from 2005 to 2009. The total cost is the sum of the costs under three heads H1, H2 and H3. Operating Expense of the company for each year is equal to 20% (H1) + 25% (H2) + 30% (H3) the profitability of the company in each year is defined as

venueReExpenseOperating

. Study the given table carefully

and answer the questions that follow: All values are given in '000s of `.

Year H1 H2 H3 Revenue

2009 20.8 30.6 40.8 104.2 2008 21.2 24.3 38.2 96.6 2007 29.6 38.4 21.6 112.4 2006 30.8 23.4 42.4 128.2 2005 24.8 42.8 36.4 130.6

66. In which of the following years was the

profitability of the company the least? (1) 2005 (2) 2006

(3) 2007 (4) 2008 67. What was the maximum percentage decrease in

the total cost of the company in a given year with respect to the previous year? (1) 7.04% (2) 7.24% (3) 7.11% (4) 8.92%

Directions for questions 68 and 69: The following table gives the distribution of the number of students based on the marks obtained in a certain examination for five sections A, B, C, D and E. Students were categorised as per their marks being 'less than 45', from 45 to 85 and greater than 85. M is the marks obtained.

Sections M < 45 45 < M ≤ 85 M > 85

A 28 72 24 B 15 68 36 C 18 52 28 D 29 58 47 E 30 60 35

68. What percentage of the total number of students

got scores less than 45? (1) 15.6% (2) 18.8% (3) 21.2% (4) 20% 69. If the qualifying mark in the paper is 48, then the

maximum number of students from a section who passed in that examination was

(1) 96 (2) 104 (3) 105 (4) 95

Directions for questions 70 to 72: The following table gives the exports and imports (values given in millions of `) of the 4 companies in the years 2002 – 03, 2003 – 04, 2004 – 05

Company Year Exports Imports Total trade

Rahul & co

2002 - 03 2003 - 04 2004 - 05

7.13 12.15 15.3

5.14 11.67 17.41

12.27 23.82 32.71

Chandu & co

2002 - 03 2003 - 04 2004 - 05

12.4 14.1 16.2

11.61 16.31 17.1

24.01 30.31 33.3

Shiva & co

2002 - 03 2003 - 04 2004 - 05

5.4 9.3 12.1

8.72 9.39

13.17

14.12 18.69 25.27

Kanta & co

2002 - 03 2003 - 04 2004 - 05

6.54 10.41 13.73

7.46 11.51 14.33

14.00 21.92 28.06

70. Which of the following company registered the

highest percentage growth in exports from 2003 – 04 to 2004 – 05? (1) Rahul & co. (2) Chandu & co. (3) Shiva & co. (4) Kanta & co.

71. Which company registered the least growth rate

in imports from 2002 – 03 to 2003 – 04? (1) Rahul & co. (2) Chandu & co. (3) Shiva & co. (4) Kanta & co.

72. Which company had the highest trade-deficit (= imports – exports) in 2004-05? (1) Rahul & co. (2) Chandu & co. (3) Shiva & co. (4) Kanta & co.

Directions for questions 73 and 74: Five companies held examinations for all the employees who are in their probation period. The following table gives the details of all the employees who have taken the exam

Company

No. of employees who crossed the cut off

% of employees who got more than 90% of

marks

No. of employees who wrote the

exam

A 180 10 300 B 225 8 450 C 150 12 250 D 400 16 600 E 300 20 575

73. If the employees who do not clear the cut offs are

rejected, then which company rejected the maximum number of employees? (1) B (2) C (3) D (4) E

74. What is percentage of employees who got more

than 90% of marks out of the total number of employees who cleared the cut off for all companies combined (approximately)? (1) 20% (2) 22% (3) 24% (4) 28%

Page 12: `DATA INTERPRETATION REPLICA QUESTIONS THAT HAVE APPEARED IN CAT IN THE LAST 4 YEARS

12

Directions for questions 75 and 76: The following table gives the sales of 5 companies in 2008 and 2009

2008 Company Price/unit

(in `)

No. of units products (in 1000)

Closing stock

P 8 12 500 Q 6 14 750 R 5 9 675 S 10 11 890 T 11 13 1200

2009 P 7 11 485 Q 4 15 690 R 8 10 775 S 6 9 1245 T 9 12 865

75. Which company had the least sales in the year

2009? (1) P (2) S (3) Q (4) T

76. In which year did R have lower sales?

(1) 2008 (2) 2009 (3) Both 2008 and 2009 (4) None of these

Directions for questions 77 and78: The following table gives the number of members in seven families and the details regarding their income and expenses.

No. of members Average income of the family in (`)

Expenses of the family in (`)

Overhead expenses in (`)

Kapoor family 6 24500 9000 3000

Khanna family 5 21000 11000 2500

Kirsten family 7 24000 14000 3750

Kumble family 4 35000 12500 4250

Khan family 6 27500 13000 6000

Kittu family 3 40000 14200 3250 Kala family 7 28000 17000 4375

77. What is the total savings made by all the families

(in `)? (1) 913175 (2) 923175 (3) 923075 (4) 921075

78. If the average income of the Khan family increased by 2% where as the expenses of the family decreased by 3%, then the savings of the Khan family would increase by (1) `3370 (2) `3570 (3) `3870 (4) `3670

Directions for question 79: The following table gives the diet statistics of 10 Drinks named P to Y. The values are in percentages.

Drinks Proteins Vitamins Carbohydrat Fats Sugar content

P 16% 24% 12% 27% 21%

Q 21% 18% 18% 14% 29%

R 17% 25% 20% 24% 14%

S 23% 26% 20% 16% 15%

T 18% 29% 19% 18% 16%

U 25% 21% 16% 15% 23%

V 24% 22% 14% 19% 21%

W 16% 29% 15% 17% 23%

X 27% 24% 14% 18% 17%

Y 19% 25% 18% 14% 24% A healthy drink is considered to have at-least 20% of proteins, at-least 23% of vitamins and at-most 20% of other ingredients. Otherwise, it is considered as an unhealthy drink. 79. What is the ratio of healthy drinks to unhealthy drinks in the given group?

(1) 2 : 3 (2) 3 : 7 (3) 1 : 4 (4) 1 : 9

Page 13: `DATA INTERPRETATION REPLICA QUESTIONS THAT HAVE APPEARED IN CAT IN THE LAST 4 YEARS

13

10

20

30

40

50

60

70

80

90

100

50 100 150 200 250 300 350 400 450

Number of Students (in hundreds)

Ave

rage

Mar

ks

Directions for questions 80 and 81: The following table shows the number of students in government schools in six different states of India during 2007, 2008 and 2009. Students in Government schools (in ‘000)

State 2007 2008 2009 Andhra Pradesh 15.4 17.2 16.1 M.P 21.2 19.6 20.9 U.P 20.1 21.4 22.1 Karnataka 18.7 17.3 19.6 Kerala 16.3 18.5 17.9 Tamil Nadu 14 19.2 20.3

80. In 2008, which state experienced the maximum increase in the number of students studying in government schools with respect to that in the previous year? (1) Andhra Pradesh (2) U.P (3) Kerala (4) Tamil Nadu

81. Which state has shown a consistent increase in the number of students studying in government schools from 2007 to 2009? (1) M.P (2) U.P (3) Kerala (4) Karnataka

Directions for questions 82 and 83: The following table gives the number of students in different sections A, B, C, D, E and F of a school in 2007

Section Students New students

A 18 12 B 12 4 C 20 8 D 17 10

E 14 11

F 19 9

82. With respect to the number of students in each section, how many sections have the number of students more than the median of the number of students in a section for the given sections? (1) 2 (2) 3 (3) 4 (4) 1

83. The student who fails in any class is retained in the

same class and section. Which of the following sections have the highest number of failed students? (1) A (2) B (3) C (4) D

Directions for questions 84 and 85: The following table gives the details of the area utilized and the production of wheat by 3 countries P, Q and R from 2003 to 2008.

P Q R Area Production Area Production Area Production 2003 2.1 5280 1.7 2850 3.5 5450 2004 2.6 1380 2.2 4850 2.7 4250 2005 1.8 2790 1.5 3950 2.6 4900 2006 1.9 5550 2.4 4800 2.3 4650 2007 2.3 5950 2.5 6800 2.1 3350 2008 3.4 5180 1.9 7800 3.2 4880

Yield return = areaUnit

oductionPr

84. Which of the following is the highest yield return

obtained by country R? (1) 2021 (2) 1885 (3) 2011 (4) 1914

85. In which year is the percentage increase in the yield return the highest for country Q? (1) 2005 (2) 2006 (3) 2007 (4) 2008

Directions for questions 86 and 87: The following graph shows the average marks obtained by the students and the number of students.

Page 14: `DATA INTERPRETATION REPLICA QUESTIONS THAT HAVE APPEARED IN CAT IN THE LAST 4 YEARS

14

The table given below shows the average marks obtained and the number of students in each class.

Class Average marks

Number of students (in hundreds)

I 45 450 II 60 325 III 31 120 IV 17 180 V 57 220 VI 37 110 VII 83 180 VIII 71 240 IX 62 305 X 79 400

86. The statement "the higher the average marks, the higher the number of students" is true in which of the following classes? (1) II, VIII, X (2) III, V, VII (3) II, IX, X (4) I, IV, VI

87. The statement "the lower the number of the students higher is the average marks" is true for which of the following classes? (1) VII, VIII, V (2) I, X, VI (3) I, II, V (4) V, VII, IX

Directions for question 88: The following table gives the details of the number of students in 6 states

No.of students (in lakhs)

Change in this year (in '000)

AP 13 +21 UP 17 +46 MP 16 −210

Bihar 18 +114 Assam 19 −612 Orissa 12 −112

88. In which state are the number of students the third highest this year?

(1) Assam (2) MP (3) Orissa (4) AP Directions for question 89: Select the correct alternative from the given choices. 89. Lakshmi, an employee, wants to invest in 3 types

of business X, Y and Z. The following table gives the investment and revenue obtained by Lakshmi from her investments

Name of

the business

X Y Z

Investments (in lakhs)

16.2 14.5 12.9

Revenue (in lakhs)

21.2 18.4 16.5

Which of the following would be the most

profitable investment for Lakshmi if she spends 20% of the revenue earned from each business to maintain her house?

(1) X (2) Y (3) Z (4) Both (1) and (3)

Directions for questions 90 and 91: The table given below gives the details of income and expenditure for some states in different regions in 2006 and 2007

Region Income (in ` crore) Expenditure (in ` crore)Per capita income (in ` crore)

Per capita expenditure(in ` crore)

2006 2007 2006 2007 2006 2007 2006 2007

North J & K 16.2 17.3 18.1 17.6 250 265 215 221

Punjab 15.1 16.5 15.5 16.9 261 272 218 224

South AP 18.3 17.5 18.9 17.9 271 281 224 236

Karnataka 14.6 16.3 15.1 17.3 283 294 241 249

East West Bengal 17.5 18.4 17.8 18.9 241 261 213 232

Assam 12.1 13.5 12.7 13.8 264 270 222 234

West Gujarat

19.4 20.2 19.8 20.9 256 271 233 241

Maharashtra 19.6 19.9 19.9 20.4 245 259 219 232

90. Which of the following regions had the ratio of income to expenditure in 2007 the highest? (1) North (2) South (3) East (4) West

Page 15: `DATA INTERPRETATION REPLICA QUESTIONS THAT HAVE APPEARED IN CAT IN THE LAST 4 YEARS

15

050

100150200250300350400450500

2003 2004 2005 2006 2007 2008 2009 2010Year

Rev

enue

in m

illio

n U

SD

US Asia

80%70%

80%95%

75%

40%

25% 30%

42%

27%

Pythons Deers Wild bisons Wolves Bears

91. What is ratio of the number of states in which per capita income increased by more than 5% to the number of states in which per capita income did not increase by more than 5%?

(1) 1 : 7 (2) 1 : 1 (3) 3 : 5 (4) 1 : 3

BAR GRAPH Directions for questions 1 to 4: Answer the following questions based on the information given below. The bar chart below shows the revenue, in million US Dollars(USD), of a cosmetics company. The data covers the period 2003 to 2007 for the United States(US) and Asia. The bar chart also shows the estimated revenues of the company for the period 2008 to 2010. 1. The difference between the estimated revenue in

Asia in 2008 and what it would have been if it were computed using the percentage growth rate of 2007(over 2006) is closest to

(1) 25 (2) 40 (3) 10 (4) 5 (5) 0 2. In 2003, sixty percent of the people who used the

company’s products in Asia were men. Given that women who used the company’s products increase at the rate of 10 percent per annum and men at the rate of 5 percent per annum, what is the approximate percentage growth of customers between 2003 and 2010 in Asia? The prices of the company’s products are volatile and may change each year.

(1) 62 (2) 15 (3) 78 (4) 84 (5) 50

3. Consider the annual percent change in the gap between revenues in the US and Asia. What is the year in which the absolute value of this change is the highest?

(1) 30-04 (2) 05-06 (3) 06-07 (4) 08-09 (5) 09-10 4. While the revenues from Asia has been growing

steadly towards that of the US, the growth rate in Asia seems to be declining. Which of the following is closest to the percent change in growth of 2007 (over 2006) relative to the growth rate of 2005 (over 2004)?

(1) 17 (2) 10 (3) 35 (4) 60 (5) 100

Directions for questions 5 and 6: The first part of the bar graph gives the number of animals of a given species in the Amazon forest as a percentage of the total number of animals of that species in South America and the second part of the bar graph gives the number of animals of that species in South America as a percentage of the total number of animals of that species in the world. The total number of deers and wild bisons in the world are 24,000 and 18,000 respectively. The total number of animals of the five given species in South America is 25,800.

Page 16: `DATA INTERPRETATION REPLICA QUESTIONS THAT HAVE APPEARED IN CAT IN THE LAST 4 YEARS

16

42

00

43

50

48

00 5

60

0

30

00

28

00 35

00 42

00

44

80

48

50 54

00

50

005

80

0

54

00

45

00

45

00

0

1000

2000

3000

4000

5000

6000

2006 2007 2008 2009

P Q R S

PP

P

QQ

Q

R

R

R

S

SS

0

500

1000

1500

2000

2500

3000

2004 2005 2006

5. If the number of Pythons and Bears in South America are 4800 and 4200 respectively, then what is the number of Wolves in the Amazon forest?

(A) 3,990 (B) 4,050 (C) 4,200 (D) 4,320

6. Using the information in the previous question, arrange the animals in the decreasing order of their number in the Amazon Forest? (A) Deers, Wild bisons, Deers, Wolves, Bears,

Pythons. (B) Wild bison, Deers, Wolves, Bears, Pythons. (C) Deers, Wolves, Wild bisons, Bears, Pythons. (D) Deers, Wolves, Wild bisons, Pythons, Bears.

Directions for questions 7 to 9: The following graphs shows the sales (by volume) of four PC manufacturing companies P, Q, R and S, in Hyderabad across four years.

7. The sales volume of which company increased

by the highest percentage from 2006 to 2008? (A) P (B) Q (C) R (D) S 8. If in 2010, company S goes bankrupt and the

sales volume of the other companies increases by 10% each, when compared to that in 2009, then what is increase, in percentage points, in the market share of company Q if these are the only companies in the market and the cost of PC is same for all the companies?

(A) 6.61 (B) 6.73 (C) 6.95 (D) 7.31 9. The market share of which of the following is the

highest if these are the only companies in the market and price per PC of the companies P, Q, R and S in each of the given years was in the ratio 1 : 2 : 1 : 2? (A) S in 2006 (B) R in 2008 (C) S in 2007 (D) S in 2009

Directions for question 10: The following table gives the distribution of the number of handsets sold by a mobile-phone manufacturing company from 2004 to 2006. P, Q, R and S denote the different models of mobile-phones sold every year. Further it is given that the selling prices of the four models of mobile phones in the year 2004,SPP, SPQ, SPR, SPS were in the ratio 3 : 4 : 5 : 6 and the ratio of the selling price of each model in the year 2004, 2005 and 2006 was 2 : 3 : 4.

10. The sales revenue of R in 2006 was more than the sales revenue of Q in 2004 by what %?

(1) 50% (2) 56.25% (3) 32

66 % (4) 75%

Page 17: `DATA INTERPRETATION REPLICA QUESTIONS THAT HAVE APPEARED IN CAT IN THE LAST 4 YEARS

17

8

2018

10

20

9

1618

22

0

5

10

15

20

25

Alto Swift Estilo

Sa

les

by

vo

lum

e (

in '0

00)

2007 2008 2009

13

2220

15

21 21

14

2522

0

5

10

15

20

25

30

Alto Swift Estilo

Pro

du

ctio

n (

in '0

00)

2007 2008 2009

Directions for questions 11 to 13: The following bar graphs gives the sales by volume of 3 cars sold in the market during 2007 to 2009 and also the number of units of these cars produced in these years. 11. In which year is the ratio of the total production to

the total sales of all 3 cars the highest? (1) 2007 (2) 2008 (3) 2009 (4) Both (1) and (2)

12 In which year is the ratio of the production of Alto

to the sales, the highest? (1) 2007 (2) 2008 (3) 2009 (4) Both (1) and (2)

13. Production − sales = Exports In which year is the ratio of the exports to sales of Swift the highest? (1) 2007 (2) 2008 (3) 2009 (4) Both (1) & (2)

PIE CHART

Directions for questions 1 to 6: The total scaled scores obtained by a student in five AIMCATs is as shown in the table below. An AIMCAT has five sections – Quantitative Ability (QA), Logical Reasoning (LR), Verbal Ability (VA), Reading Comprehension (RC) and Data Interpretation (DI).

Exam Total scaled scores AIMCAT 1 300 AIMCAT 2 280 AIMCAT 3 360 AIMCAT 4 320 AIMCAT 5 350

Page 18: `DATA INTERPRETATION REPLICA QUESTIONS THAT HAVE APPEARED IN CAT IN THE LAST 4 YEARS

18

AIMCAT 1

QA 17%

LR 23%

RC 15%

DI 25%

VA 20%

AIMCAT 2

QA 37.5%

DI 19%

RC 20%

LR 11%

VA 12.5%

AIMCAT 5

QA24%

VA26%

LR12%

RC20%

DI18%

AIMCAT 3

DI 22.5%

RC 15%

LR 12.5%

VA

162/3%

QA

331/3%

AIMCAT 4

QA 25%

VA 10%

LR 20%

RC 20%

DI 25%

The following pie charts give the distribution of the scaled scores in each AIMCAT. NOTE: The scores given are actually the ‘scaled scores’. The ‘actual scores’ are obtained by dividing the scaled scores by the ‘scoring factors.’ The maximum scaled scores in the five sections are also given below

Section QA LR VA RC DI Scoring factor 1.5 1.25 2.5 1.25 1.2 Maximum scaled score 150 100 125 75 120

1. The ‘scaled score’ obtained by the student in the

VA section in AIMCAT 3 is what percent of the maximum possible’ actual score’ in that section? (A) 48% (B) 60% (C) 80% (D) None of these

2. What is the least difference between the ‘scaled score’ in the RC section and the maximum possible ‘actual score’ in that section in any of the given AIMCATs? (A) 4 (B) 6 (C) 11 (D) 19

3. In an AIMCAT, if in at least three of the five sections a candidate has a ‘scaled score’ in a section greater than 80% of the maximum possible ‘actual score’ in that section, the performance of the candidate is considered to be

‘significant’. In how many of the five AIMCATs, the student shows a ‘significant’ performance?

(A) 1 (B) 2 (C) 3 (D) 5

4. From AIMCAT 1 to AIMCAT 5, in which section did the student have the highest percentage increase in the scaled score? (A) LR (B) QA (C) RC (D) DI

5. From AIMCAT 1 to AIMCAT 5, in which section,

did the student have the least percentage change in the actual score?

(A) LR (B) VA (C) RC (D) DI 6. In which AIMCAT was the marks obtained by the

student in the RC section the highest? (A) AIMCAT 4 (B) AIMCAT 5 (C) AIMCAT 3 (D) AIMCAT 1

Page 19: `DATA INTERPRETATION REPLICA QUESTIONS THAT HAVE APPEARED IN CAT IN THE LAST 4 YEARS

19

State Govt. Jobs

AP 20%

MP 14%

UP 7%

Karnataka 9%

TN 25%

Kerala, 25%

Central Govt. Jobs

UP 12%

Karnataka 20%

Kerala, 15%

MP 8% AP

25%

TN20%

Total number of employees in P = 17500

Operations dept 30%

HR dept10%

Academic dept 60%

Total number of employees in Q = 18000

Operations dept 62%

HR dept21%

Academic dept 17%

Painting 15%

Karate 15%

Singing 20%

Embroidery 5%

Dancing 45%

Direction for question 7: 1000 students in a school have to choose one extra-curricular activity (in which they are interested) among 5 activities – Dancing, Singing, Printing, Embroidery classes and Karate. Only boys chose Karate, and only girls chose Embroidery classes. The ratio of the number of boys to girls in painting is 1 : 1. 80% of the students who choose Dancing are boys and 80% of the students who choose singing are girls. The following pie chart gives the distribution of the students in the five activities.

7. If the students who selected Painting and Singing

are made to sit in the same class then what would be the ratio of boys to girls in that class? (1) 23 : 45 (2) 1 : 2 (3) 23 : 47 (4) 23 : 49

Direction for question 8: The following pie charts give the number of employees in central government jobs and state government jobs in 6 states of India 8. The number of employees in central government jobs and in state government jobs are in the ratio

6 : 1. Find the ratio of the number of employees in central govt. jobs in AP to that in state jobs in Kerala? (1) 6 : 1 (2) 4 : 1 (3) 5 : 2 (4) 3 : 1 Directions for questions 9 and 10: The following pie charts give the details of all the employees of 2 companies P & Q:

9. What percent of the employees in both the

companies belong to the HR department? (1) 17.5% (2) 15.5% (3) 16.1% (4) 17%

10. What is the approximate ratio of the number of employees in the Academic department to the Management department (HR dept + operation dept) in both the companies combined?

(1) 0.71 (2) 0.62 (3) 0.9 (4) 0.84

Page 20: `DATA INTERPRETATION REPLICA QUESTIONS THAT HAVE APPEARED IN CAT IN THE LAST 4 YEARS

20

0

5

10

15

Marketing HR Finance Operation Systems

Males Females

Finance 12.50%operations

10%

systems15%

HR 25% Marketing

37.50%

Pie Charts + Bar Charts Directions for questions 1 and 2: In a management institute, students opt for various disciplines. The distribution of students across disciplines is shown in the pie chart and the ratio of the number of males and females in each discipline is shown in the bar chart. The institute has sixteen centres in the country and the students whose data is represented in the following charts are from those centres in the year 2009-2010. Study the given charts carefully to answer the following questions.

Total number of students = 10080. 1. In the year 2009 – 2010, the total number of

female students in the institute was less than the total number of male students by what percentage?

(1) 11% (2) 9.2% (3) 10.6% (4) 12.4%

2. For which discipline was the difference between the number of male and female students the highest?

(1) H R (2) Finance (3) Marketing (4) Systems

Page 21: `DATA INTERPRETATION REPLICA QUESTIONS THAT HAVE APPEARED IN CAT IN THE LAST 4 YEARS

21

0

2000

4000

6000

8000

10000

50 100 150 200 250 300 350 400

Number of units produced

Tota

l exp

ense

s (in

Rs.

)

40

7

21

3935

0

10

20

30

40

50

0 1 2 3 4 5Ene

rgy

Con

sum

ptio

n (in

KW

h)LINE GRAPH

Directions for questions 1 and 2: The following graph gives the relation between the total expenses and the number of units produced in a factory. Assume that all units produced are sold and that on a normal day 200 units are produced. The selling price of each unit is `35. 1. On a particular day if 300 units are sold, find the

percentage change in profit when compared to a normal day (A) 100 (B) 150 (C) 200 (D) 250

2. What is the average additional cost (in `) per unit

produced in comparison to a normal day, when 350 units are produced? (A) 18 (B) 23 (C) 25 (D) 28

Directions for questions 3 and 4: A family uses the following electrical appliances– TV, Refrigerator, Geyser, Washing machine and Grinder. The monthly electricity bill generated has two components- a fixed cost of `60 and a variable cost of `0.35 per kWh. The family uses Refrigerator throughout the day, Geyser for 2 hours, Washing machine for 0.5 hours, Grinder 0.25 hours and watches TV for 15 hours everyday. The line graph given below shows the energy consumption of the above mentioned appliances in a week for the family. 1 → Geyser

2 → Geyser + Refrigerator 3 → Geyser + Refrigerator + TV 4 → Geyser + Refrigerator + TV + Washing machine 5 → Geyser + Refrigerator + TV + Washing machine

+ Grinder 3. Which of the following is true?

(1) The energy consumed by TV for 3 days is more than that of Refrigerator for 3 days.

(2) The energy consumed by Geyser for 4 days is less than that of Grinder for 7 days.

(3) The energy consumed by Washing machine in a week is less than that of Geyser for 2 weeks.

(4) The energy consumed by TV for 2 days is less than that of Washing machine for a week.

4. If the fixed cost increases by 25%, then what

would be the percentage increase in the total cost of energy consumption by the family in a month of 30 days. (1) 10% (2) 12.5% (3) 15% (4) 19%

DATA SUFFICIENCY

Directions for questions 1 to 4: Each question is followed by two statements, A and B. Answer each question using the following instructions: Mark (1) If the question can be answered by using

the statement A alone but not using the statement B alone

Mark (2) If the question can be answered by using the statement B alone but not by using the statement A alone.

Mark (3) If the question can be answered by using either of the statements alone.

Mark (4) If the question can be answered by using both the statements together but not by either of the statements alone.

Page 22: `DATA INTERPRETATION REPLICA QUESTIONS THAT HAVE APPEARED IN CAT IN THE LAST 4 YEARS

22

Mark (5) If the question cannot be answered on the basis of the two statements.

1. In a particular company, sixty employees were

managers. Ten among them were also among the people who had newly joined. How many employees in the company were newly joined? (A) Sixty percent of the newly joined employees

were not managers. (B) All the newly joined employees were not

necessarily managers. 2. Five people Amar, Babu, Craig, David and

Edward were the only ones who participated in a chess tournament. They were ranked on the basis of the points they scored. David got a higher rank as compared to Edward while Babu got a higher rank as compared to Craig. Craig’s rank was lower than the median. Who among the five got the highest rank? (A) Amar got the last rank. (B) Babu was not among the top two rankers.

3. Thirty percent of the students of a school are

boys. Ten percent of the girls in the school are athletes. What is the percentage of boys in the school who are athletes? (A) Twenty five percent of the students are athletes. (B) Number of boys in the school who are

athletes is 20% more than the number of girls who are athletes.

4. In a basketball match, team A was trailing by

25 points at the end of the first half. Did it win the match? (A) In the second half team A scored 35 points. (B) The opponent scored 35 points in the match.

PPL Directions for question 1: (1) If the question can be answered from one of the

statements alone but not from the other. (2) If either statement alone is sufficient to answer

the question. (3) If both the statements together are sufficient but

either statement alone is not sufficient. (4) If the question cannot be answered even by

combining both the statements 1. Pramod bought a new car after selling his old car.

If the cost of the old car was 40% that of the new car, find the price of the new car. Ι. He borrowed an amount which was equal to

60% of the cost of the old car from his friend and raised the remaining amount by withdrawing from his personal savings account.

ΙΙ. His total personal savings were `3,00,000

Numbers

Directions for questions 1 and 2: Mark option (1), if the question can be answered by any one of

the statements alone but not by the other. (2), if the question can be answered by either

statement alone. (3), if the question can be answered by combining

both the statements but not by each statement alone.

(4), if the question cannot be answered even after combining both the statements.

1. Which of x, y, z is the maximum? A. xy = 18 and yz = 21

B. xz = 42, where x, y and z are natural numbers.

2. If ONE = O + N + E, i.e. considering 'O'=6, N= 7

and E = 8 we will get ONE = O + N + E = 6 + 7 + 8 = 21. Find the value of SEEN, if SEVEN = 19 and all alphabets have distinct values which are natural numbers. A. FIVE = 14 B. NINE = 7

Directions for question 3: This question is based on the following information. Raju's dad goes to various temples on certain days in a year of 365 days which are numbered 1, 2, ….365. He goes to the Shiva temple on days which are multiples of 3. He goes to the Venkateshwara temple on days which are multiples of 4. He goes to the Saibaba temple on the days which are multiples of 7. 3. On how many days did Raju's dad go to only one

temple? (1) 204 (2) 214 (3) 220 (4) 240

4. If A, B, C, D, E, F and G are distinct single digit

natural numbers from 1 to 7such that A + B + C = C + D+ E = E + F + G = 11, then how many ordered pairs (C, E) exist which satisfy the given relation? (1) 0 (2) 2 (3) 4 (4) 8

CASELET

Directions for questions 1 to 3: Answer the following questions based on the statements given below. (i) There are three buildings on each side of the

road. (ii) These six buildings are labelled as A, B, C, D, E

and F. (iii) The buildings are of different colours, namely,

Violet, Indigo, Blue, Green, Yellow, and Orange. (iv) The buildings are of different heights.

Page 23: `DATA INTERPRETATION REPLICA QUESTIONS THAT HAVE APPEARED IN CAT IN THE LAST 4 YEARS

23

(v) E, the tallest building, is exactly opposite to the Violet coloured building.

(vi) The shortest building is exactly opposite to the Blue coloured building.

(vii) F, the Green coloured building is located between A and D.

(viii) C, the Yellow coloured building is exactly opposite to A.

(ix) B, the Blue coloured building is exactly opposite to F.

(x) A, the Orange coloured building, is taller than C, but shorter than B and D.

1. What is the colour of the building diagonally

opposite to the Yellow coloured building? (1) Orange (2) Indigo (3) Blue (4) Violet (5) None of these 2. Which is the second tallest building? (1) A (2) B (3) C (4) D (5) Cannot be determined 3. What is the colour of the tallest building? (1) Violet (2) Indigo (3) Blue (4) Yellow (5) None of these Directions for questions 4 to 7: Answer the following questions based on the information given below. Six teams (P, Q, R, S, T and U) are taking part in a cricket tournament. Matches are scheduled in two stages. Each team plays three matches in stage-Ι and two matches in stage-ΙΙ. No team plays against the same team more than once in the event. No ties are permitted is any of the matches. The observations after the completion of stage-Ι and stage-ΙΙ are as given below. Stage Ι: • One team won all the three matches. • Two teams lost all the matches. • S lost to P but won against R and U. • T lost to Q but won against R and U. • Q lost at least one match. • U did not play against the top team of stage-Ι. Stage ΙΙ: • The leader of stage-Ι lost the next two matches. • Of the two teams at the bottom after stage-Ι, one

team won both matches, while the other lost both the matches.

• One more team lost both matches in stage-ΙΙΙ. 4. The two teams that defeated the leader of stage-Ι

are: (1) U and S (2) T and U (3) Q and S (4) T and S (5) U and Q 5. The only team(s) that won both matches in

stage-ΙΙ is (are): (1) Q (2) T and U (3) P, T and U (4) Q, T and U (5) Q and U 6. The teams that won exactly two matches in the

event are:

(1) P, S and U (2) S and T (3) T and U (4) S, T and U (5) S and U 7. The team(s) with the most wins in the event is (are): (1) p (2) P and R (3) U (4) T (5) Q and T Directions for questions 8 to 12: Answer the following questions based on the information given below. Anand, Bala and Chandu are three professional traders who traded in gold in the commodities market. Anand followed the strategy of buying at the opening of the day at 10 a.m. and selling the whole lot at the close of the day at 3 p.m. Bala followed the strategy of buying at hourly intervals: 10 a.m., 11 a.m, 12 noon, 1 p.m. and 2 p.m. and selling the whole lot at the close of the day. Further, he buys an equal quantity (by weight) in each purchase. Chandu followed a similar pattern as Bala but his strategy is some what different. Chandu total investment amount is divided equally among his purchases. The profit or loss made by each investor is the difference between the sale value at the close of the day less the investment. The return for each investor is defined as the ratio of the profit or loss to the investment amount expressed as a percentage. 8. On a day of fluctuating prices, the price of gold

ends with a gain, i.e., it is higher at the close of the day compared to the opening value. Which trader got the maximum return on that day?

(1) Bala (2) Chandu (3) Anand (4) Bala or Chandu (5) Cannot be determined 9. Which one of the following statements is always

true? (1) Anand will not be the one with minimum return. (2) Return for Chandu will be higher than that for

Bala. (3) Return for Bala will be higher than that of

Chandu. (4) Return for Chandu cannot be higher than that

of Anand. (5) None of the above

10. On a “boom” day the price of gold keeps rising throughout the day and peaks at the close of the day. Which trader got the minimum return on that day?

(1) Bala (2) Chandu (3) Anand (4) Anand or Chandu (5) Cannot be determined One day, two more traders, David and Emma joined Anand, Bala and Chandu for trading in gold. David followed a strategy of buying equal quantity of gold at 10 a.m., 11 a.m. and 12 noon and selling the same quantity at 1 p.m., 2 p.m. and 3 p.m. Emma on the other hand followed the strategy of buying using all her money at 10 a.m. and selling all of them at 12 noon and again buying using all her money at 1 p.m. and again selling them at the close of the day at 3 p.m. At the close of the day the following was observed. (i) Anand lost money in the transaction.

Page 24: `DATA INTERPRETATION REPLICA QUESTIONS THAT HAVE APPEARED IN CAT IN THE LAST 4 YEARS

24

(ii) Both David and Emma made profits. (iii) There was an increase in the price of gold during

the closing hour compared to the price at 2 p.m. (iv) The price of gold at 12 noon was lower than the

closing price. 11. The price of gold was its highest at (1) 10 a.m. (2) 11 a.m. (3) 12 noon (4) 1 p.m. (5) Cannot be determined 12. Which of the following is necessarily false?

(1) The price of gold was not at its lowest at 2 p.m. (2) The price of gold was at its lowest at 11 a.m. (3) The price of gold at 1 p.m. was higher than

the price at 2 p.m. (4) The price of gold at 1 p.m. was higher than

the price at 12 noon. (5) None of the above

Directions for questions 13 to 15: Select the correct alternative from the given choices. 13. Four persons A, B, C, and D on a tour to a hill

station booked four consecutive rooms among 101, 102, 103 and 104 (all the rooms being on the same side in that order) in a hotel for their stay there. A had a dispute with B and did not want to stay in a room adjacent to him. C being a childhood friend of D booked a room adjacent to D who in turn booked the room adjacent to B. If B booked an odd numbered room, then which room did C book?

(A) 101 (B) 102 (C) 103 (D) 104 14. Four girls Dolly, Molly, Polly and Kelly appeared

for their semester paper on Mass Communication in which two of them failed. When asked as to who passed in the exam, they gave the following replies. Dolly: I did not fail in the examination Molly: I passed in the examination and so did Kelly. Polly: Only one among Dolly and Kelly failed in the examination Kelly: Only one among Polly and Dolly passed

in the examination. If it is given that exactly three of them were telling the truth, then find the person who was lying. (A) Dolly (B) Molly (C) Polly (D) Kelly

15. In IIBM, a reputed B school with a total strength

of 270 students, every student opted for at least one specialization among the three – Finance, HR and Marketing. The number of students who opted for all three was 37.5% of those who opted for exactly one. The number of students who did not opt for Finance was 50% of these who opted for Finance which in turn was 25% more than those who did not opt for HR. The number of students who opted for only marketing and HR was equal to those who opted for only Finance and Marketing and also 331/3% of those who opted for all three categories. If the number of students who opted for only Finance and HR was 50% of those who opted for only Finance, then how many students opted for exactly two of the three specializations?

(A) 36 (B) 72 (C) 45 (D) 54

Directions for questions 16 and 17: Akira and Aroki read four books each from among A, B, C, D, E, F, G and H such that each of the eight books was read by exactly one person. Further the following information was known. The person who read D, also read F. Books A and B were not read by the same person. The person who read book F, did not read book C. 16. If Akira read books E and G, then Aroki did not

read book (A) D (B) F (C) H (D) C 17. If books C and E were not read by the same

person, then which of the following two books was definitely read by the same person

(A) A and C (B) B and C (C) G and H (D) Cannot be determined Directions for questions 18 to 25: Select the correct alternative from the given choices. 18. Nine workers – W1, W2, W3, W4, W5, W6, W7, W8,

and W9 are to be allotted work in four shifts –Morning, Afternoon, Evening and Night with not more than three workers in the same shift. The workers are allotted the shifts as per the following conditions. Each worker can be allotted work in only a single shift. W1 and W2 do not work in the same shift.

W3 is to be allotted a shift, earlier than W6 but later than W2.

W2 is to be allotted the morning shift and W4 is to be allotted a shift which is two shifts after W1.

W5 and W7 are both allotted the same shift and it is one shift earlier than the shift allotted to W6.

W9 is allotted the same shift as W3. W8 cannot be allotted work in which of the

following shift? (A) Morning (B) Afternoon (C) Evening (D) Night 19. Four players – A, B, C and D are the members of a

cricket team. They are to lead their team in the upcoming tournament where they will be playing five matches: Match1, Match 2, Match 3, Match 4 and Match 5 to be held one after the other with each match having two leaders, one as captain and the other as vice captain. No person can Captain the side in two consecutive matches and neither can the same person be the vice captain on consecutive matches. These players are to lead their team subject to the following conditions. (1) B was the captain of the side for Matches

1 and 3. (2) C can be the vice captain only if A is the

captain. (3) D refused to lead the team as captain if A or

B led the team as captain in the preceding match.

(4) Each of the four players was the captain and the vice captain in at least one of the five matches.

Page 25: `DATA INTERPRETATION REPLICA QUESTIONS THAT HAVE APPEARED IN CAT IN THE LAST 4 YEARS

25

Which of the following statements is true? (A) C was the vice captain in Match 5. (B) B was the Vice captain in Match 4. (C) C was the vice captain in Match 2. (D) A was the captain in Match 5 20. Each of the three persons – A, B and C have to

buy three household appliances – AC, Refrigerator and Water Purifier on three days – Monday, Tuesday and Wednesday. No two persons buys the same type of appliance on any single day. B buys the Refrigerator on Tuesday, while he has to buy the Water Purifier before buying the AC.

Which of the following is true regarding A? (A) A buys the Water Purifier before the

Refrigerator. (B) A buys the AC before buying the Water Purifier. (C) A buys the AC before buying the Refrigerator. (D) A buys the Refrigerator before buying the AC. 21. The following table gives the average runs scored

by four players in all the matches in a year.

Player Average Runs Sachin 48 Dravid 50 Mongia 40 Hussey 42 Sachin & Mongia 44 Dravid & Hussey 46

Sachin played more number of matches than Dravid. If n is the average runs scored by all four players, then what must be true about x?

(A) 44 < x < 45 (B) 45 < x < 46 (C) x = 45 (D) None of the above 22. In an international maths olympaid, there were

seven questions. The marks scored by two candidates in seven questions (though not necessarily in the same order) from Q1 to Q7 are as follows

Ramesh: 26, 30, 34, 42, 46, 50, 54 Sanjay: 13, 34, 42, 46, 50, 54, 62

One of the questions was declared invalid (after the exam and the marking was over). The average marks of both the candidates in the remaining six questions were calculated. Both candidates scored same marks in the “invalid question”, and so as a result of its exclusion, the average score of one candidate increased while that of the other candidate decreased. What was the marks scored by each candidate in the “invalid question”?

(A) 42 (B) 46 (C) 34 (D) 50 23. (1) If A wins, then B wins. (2) If B wins, then C does not win. (3) Only if D wins, then at most one of A or C wins.

If there are only four players A, B, C and D, then which of the following must be true?

(A) D wins (B) D does not win (C) B wins (D) A does not win

24. A race is held on three days and there are three drivers – Schumi, Sebastian and Mclaren. On each day they are ranked from one to three in the order in which they finish the race. On any day, Schumi is always ranked ahead of Mclaren. No driver secures the same rank on more than two days. Which of the following must be false?

(A) Sebastian wins the race on exactly two days. (B) If Sebastian wins the race on Day 1, then he

is ranked below Mclaren on the remaining two days.

(C) Sehbastian comes last on exactly two days. (D) Sebastian beats Mclaren on all the three days. 25. A company has two branches, one at Kukatpally

and the other at Narayanguda. Six persons – Tom, Raj, Ryan, Mokambo, Sashi and Govind have to work in these branches. However, no person works in both the branches. Each branch has three employees. Further, it is known that

(1) Tom & Raj do not work in the same branch. (2) Sashi and Govind works in the same branch. Which of the following must be true?

(A) If Ryan works in the Narayanguda Branch, then Mokambo does not work in the Kukatpally branch

(B) Both Tom and Mokambo work in the same branch.

(C) Both Govind & Raj work in the same branch. (D) More than one of the above. Directions for questions 26 to 28: The analysis of the way the three star players of Mumbai Indians made runs is given below. The runs made by Pollard, Dumminy and Bhajji consist of three types of shots “straight drive”, “pull shot” and “others”. (i) The total runs made by Pollard is 40 more than

that made by Bhajji. (ii) Pollard scored 20% of his runs through the ‘pull

shot’. (iii) The runs made by Dumminy is the average of the

runs made by Pollard & Bhajji. (iv) Bhajji scored 25% of his runs, i.e., 20 runs,

through he ‘pull shot’. (v) The runs scored by Dumminy through ‘others’ is

15% of the sum of the total runs scored by Pollard and Bhajji.

(vi) The runs scored by Dumminy through “straight drive” is 60% of the total runs made by him.

(vii) Each player scored at least one run through each type of shot.

26. Find the maximum possible difference between

the runs scored by Pollard through ‘Straight drive’ and that by Dumminy through ‘Pull shot’.

(A) 110 (B) 86 (C) 50 (D) None of these 27. The runs scored by Bhajji through ‘straight drive’ is

x% of the total runs scored by all the three players put together. Find the maximum possible value of x.

(A) 20 (B) 30 (C) 19 (D) 192/3

Page 26: `DATA INTERPRETATION REPLICA QUESTIONS THAT HAVE APPEARED IN CAT IN THE LAST 4 YEARS

26

28. Find the runs scored by Bhajji through ‘others’. (A) 50 (B) 60 (C) 40 (D) Cannot be determined Directions for questions 29 to 41: Select the correct alternative from the given choices. 29. (1) All shoes are pens. (2) Not all pens are pencils. (3) All pens are chocolates. (4) Not all chocolates are pens.

Which of the following must be true? (A) Some chocolates are not shoes. (B) Some shoes are chocolates. (C) Some pencils are not chocolates. (D) More than one of the above. 30. Ramesh was gifted a wonderful watch by his

father. When the watch was showing 4 p.m. Ramesh started for his friend Umesh’s house to show off his watch. He was driving at a constant speed. His friends watch was showing 6:10 p.m. at the moment Ramesh arrived at Umesh’s house. Ramesh immediately drove back at

th45

of the earlier speed. As soon as he reched

home, he saw that the time shown by his watch was 7:45 p.m. By how many minutes is Ramesh’s watch faster/slower as compared to Umesh’s watch?

(A) No difference (B) 10 mins slower (C) 10 mins faster (D) 5 mins slower 31. In a selection processes, each candidate has to

appear for two types of tests – T1 and T2. 200 candidates failed in T2 while 300 failed in T1. Ratio of the number of candidates who failed in both T1 and T2 to those who passed in both T1 and T2 is same as the ratio of the number of candidates who passed in T2 to those who passed in T1. This ratio is an integral value. Find the number of candidates who passed in both the tests.

(A) 100 (B) 50 (C) 20 (D) Cannot be determined 32. Four persons – W, X, Y, Z secured distinct ranks

from 1 to 6 in four events – Swimming, Running, Cycling and Walking.

Swimming Running Cycling Walking TotalW 5 6 15 X 1 6 Y 5 2 4 14 Z 1 18

Which of the following was ranked 3? (A) X in Cycling (B) W in Swimming (C) Z in Running (D) More than one of the above. 33. Each of the five students Adam, Ben, Cathy,

Dimitry and Emmanuel wrote four exams, one each in Mathematics, Physics, Chemistry and

Biology and were ranked from 1 to 5 (1 being the highest and 5 being the lowest) in each of the exams. The sum of the ranks obtained by them in the four exams were 11, 15, 12, 8 and 14 respectively in the same order where no two students got the same rank in any exam. Further it was known that (1) Dimitry got the 4th rank in Physics and his ranks

in Chemistry and Biology were the same. (2) The rank of Emmanuel in Chemistry was the

same as that of Dimitry in Mathematics. (3) The ranks secured by Ben in Mathematics

and Chemistry was the same. (4) Ben had the least rank in Physics and Cathy

got the 3rd rank in Mathematics. (5) Emmanuel got distinct ranks in all four

subjects and did not get the best rank in any subject.

Which of the following is definitely true. (A) Adam’s rank in Maths was 2. (B) Adam’s rank in Physics was 2. (C) Adam’s rank in Chemistry was 4. (D) Cathy’s rank in Chemistry was 3. 34. There are 120 families living in a housing society

where each family owns at least one among a Refrigerator, an Air Conditioner and LCD TV.

• 24 families have only a Refrigerator. • 20 families have only an Air Conditioner. • 26 families have only a LCD TV.

• At least 40 families own both Refrigerator as well as Air conditioners.

At most how many families own a refrigerator and an LCD TV but not an air conditioner?

(A) 10 (B) 12 (C) 15 (D) 30 35. Five lecturers L1, L2, L3, L4 and L5 are to deliver

lectures in a college on four consecutive days from Monday to Thursday with two lectures to be delivered on each day. No lecturer is to deliver more than two lectures in the given period. Further it is known that,

L1 delivers lectures on Monday and Thursday only. L3 delivers a lecture on a day only if L2 delivered a lecture the preceeding day. L1 and L4 do not deliver lectures on the same day. Each lecturer delivered at least one lecture in the given period. L3 and L4 do not deliver lectures on consecutive days.

On which day did L3 deliver the lecture? (A) Monday (B) Tuesday (C) Wednesday (D) Thursday. 36. A committee consisting of five members is to be

formed from five boys among B1, B2, B3, B4 and B5 and three girls among G1, G2 and G3. There must be three boys and two girls in the committee. Further it is known that,

(1) If B1 is selected, then B2 cannot be selected. (2) If B3 is selected, then B4 cannot be selected.

(3) Both G1 and G2 cannot be selected at the same time.

(4) If B4 is selected then B5 must also be selected. (5) If B2 is selected, then G2 must also be selected.

Page 27: `DATA INTERPRETATION REPLICA QUESTIONS THAT HAVE APPEARED IN CAT IN THE LAST 4 YEARS

27

A

B C

D

E F

G

(2)

(4) (5)

(1) (1)

(7)

(3) (8)

(6)

(3)

(3)

(5)

H

The total number of ways in which the committee can be formed is

(A) 5 (B) 4 (C) 3 (D) 6 37. Three products P1, P2 and P3 needs to be

machined in two machines M1 and M2. A product cannot be machined in two machines at the same time and the entire machining of a product in a machine must be completed in one go. A product can be machined in the machines in any order (i.e., M1 before M2 or M2 before M1). The duration (in hours) for which the products needs to be machined in each machine is given in the table below.

M1 M2 P1 3 2 P2 4 3 P3 2 5

If the total time taken to finish machining all the three products is the minimum, then which of the following is definitely false.

(A) Product P3 is machined in M1 before Product P2 (B) Product P2 is machined in M2 before Product P1. (C) Product P2 is machined in M1 before Product P1.

(D) None of these. 38. In a group of 30 members belonging to a Sports

Club, each member played at least one of the three games from football, cricket and hockey. 18 members played at least two games. If the number of members playing exactly one game was three times that of those playing all the three games, then how many played exactly two games?

(A) 12 (B) 14 (C) 16 (D) 10 39. Ten athletes M1 to M10 competing in an athletics

meet represented five countries among UK, Germany, France, Switzerland and Turkey with each country being represented by two athletes. Further it was known that M1 and M3 represented the same country. Both M2 and M4 were either from France or from Germany.

M5 and M9 were from different countries. M6 belonged to UK and so did M8

M7 was from Turkey and M9 did not belong to France or Switzerland. Which country did M10 belong to, if M5 did not belong to France or Switzerland?

(A) France (B) Turkey (C) Germany (D) Switzerland 40. Each of the four friends Sneha, Shikha, Sushma

and Sushmita bought a birthday present for their common friend Rahul. The gifts bought by them were a shirt costing `1200, a tie costing `800, trousers costing `2000 and a pair of shoes costing `2800 (not in the same order). The sum of the costs of the gifts bought by Sneha and Sushma was equal to that of the gift bought by Shikha. Further the difference between the cost of Sushma’s gift and Sushmita’s gift was equal to the cost of Sneha’s gift. What was the gift bought by Shikha?

(A) Shirt (B) Trousers (C) Tie (D) Shoes

41. Two teams, each with three members, are to be selected from among the seven students – P, Q, R, S, T, U and V for the Inter School Meet consisting of a Debate and an Elocution contest. In addition it is also known that (1) P was the only student who represented his

school in both the contests. (2) If Q was selected for Debate, then R must be

selected for the Elocution contest. (3) Both S and T cannot be selected for the

same event. (4) V represented his School in Debate where as

U was selected for one of the two categories. (5) If R was selected for the Elocution contest,

then U cannot be selected for Debate. Which of the following statement is definitely false? (A) Both U and Q cannot be selected for the

same event. (B) If both V and U are selected for the same

event, then Q must be selected. (C) If S is selected for Debate then either R or T

or Q must be selected for Elocution. (D) None of these.

NETWORKS

Directions for question 1 and 2: The following network gives the bus routes of APSRTC in Hyderabad for its new A/C buses introduced in the previous month. Any passenger boarding a bus is charged `5 as local service charge and `8 as fixed charge in addition to a charge of `4 per km. Further it is also known that (1) a bus does not visit the same city more than

once. (2) between any two cities only one mode of

transport is available, i.e. the bus. (3) in the network shown, the values in brackets

denote the distance in kilometers. 1. Find the minimum cost incurred by a person to

travel from A to H. (1) `53 (2) `75 (3) `61 (4) `73

2. If the road connecting A to E is under repair, then what is the minimum cost incurred by a person to travel from A to H? (1) `57 (2) `59 (3) `61 (4) `73

Page 28: `DATA INTERPRETATION REPLICA QUESTIONS THAT HAVE APPEARED IN CAT IN THE LAST 4 YEARS

28

1.0 0.8 0.6 0.4 0.2

3000 4000 5000 6000 7000 8000 9000 10000 11000 12000 13000 14000

Hap

pine

ss Q

uotie

nt

A B

C

D

E

F G

I

J

K

L H

M

N

O

• •

• • • •

• •

• •

QBR (Miscellaneous)

Directions for question 1: Select the correct alternative from the given choices. 1. A group of 450 persons was tested for HIV

infection, but there was an error in the testing process due to which four types of results were observed. The results identified the following four categories of persons. C1: Persons infected with HIV but reported

negative in the test. C2: Persons not infected with HIV but reported

positive in the test C3: Persons not infected with HIV and reported

negative in the test C4: Persons infected with HIV and reported

positive in the test If it is known that the results of 275 people were correctly reported and that the number of infected people is 50% that of non-infected people, find the difference in the number of people under categories C2 and C4. (1) 11 (2) 25 (3) 91 (4) 100

DI (Miscellaneous)

Directions for question 1: A company manufacturing cricket balls incurs a manufacturing cost of `50 per ball. The sale of the number of balls decreases linearly such that the number of balls sold decreases by 20 for every `2 rise in the selling price. It is known that when the selling price of each ball is `59, the number of balls sold is 700.

1. Which of the following will give maximum profit? Selling price Number of balls sold

(1) `89 400 (2) `93 360 (3) `85 440 (4) `99 300

Directions for questions 2 and 3: There are two different investment schemes Ι and ΙΙ yielding different percentage returns based on the market conditions. Market conditions are catagorised into three types, namely bearish, steady and bullish. The probabilities of the market conditions and the respective yield percentages for the two schemes are given below

Scheme Ι

Market conditions Probabilities Yield percentage Bearish 0.25 –30 Steady 0.55 80 Bullish 0.20 100

Scheme ΙΙ Market conditions Probabilities Yield percentage

Bearish –10 Steady 0.4 60 Bullish 100

2. If the total yield from both the schemes is the

same, what is the probability of the market condition being bearish for scheme ΙΙ? (1) 15% (2) 20% (3) 25% (4) 35%

3. If the probabilities for the market conditions being bearish, steady and bullish for scheme Ι are 0.2, 0.45 and 0.35 respectively instead of what is given in the table, then for the same yield percentage (as given in the table), what would be the percentage increase in the total yield from scheme Ι? (1) 8.9% (2) 9.1% (3) 10.2% (4) 15%

Directions for questions 4 and 5: The following figure gives the per capital income and the Happiness Quotient of fifteen countries in the world vs the Per capita income. (Only for families with 3 or more members)

Per capita Income (in US $) The following table gives the names of the countries represented in the above figure.

A-Taiwan E-China I-Philippines M-Thailand

B-Australia F-Czech Republic J-Poland N-Singapore

C-Japan G-Israel K-Slovakia O-Austria D-South Korea H-Brazil L-Latvia

Page 29: `DATA INTERPRETATION REPLICA QUESTIONS THAT HAVE APPEARED IN CAT IN THE LAST 4 YEARS

29

Rahul _ _ _ _ _ Ramya

WRITTEN SCORE 30%

GROUP DISCUSSION

25%

WORK EXPERIENCE 10%

ESSAY WRITING 10% INTERVIEW 25%

5

5 5

5

5

4

4

4

4

4

3

3

3

3

3

22

2

2

2 1

1

11

1

3 star

S/o $600 D/o $1100

CWB $250 CWNB $125

Hotel Booking

5 star

S/o $1000 D/o $1900

CWB $350 CWNB $200

City tour Yes($ 40)

No

Hunters Valley Yes($ 35)

No

4. Which of the following gives the list of the countries which indicate “Low per capita Income with low happiness Quotient”

(1) China, Thailand, Slovakia (2) China, Slovakia, Brazil (3) China, Poland, Brazil

(4) China, Slovakia, Poland

5. Which of the following lists indicate “high happiness quotient with high per capita income”. (1) Australia Latvia, Austria (2) Australia, South Korea, Austria (3) Australia, Austria, Israel (4) Australia, Latvia, Czech Republic.

Direction for question 6: The selection of a candidate into a B - school consisted of evaluating various factors - Written score, Group Discussion, Essay writing, Interview and Work experience. The B - school assigns some weightage to each of these factors. The cumulative score of a student is the sum of the product of the scores of the student attained in these factors and the weightage assigned to the respective factor. The following diagram gives the scores of two students Rahul and Ramya and also the weightage assigned to each factor. 6. What is the difference in the cumilative values of the scores obtained by Rahul and Ramya?

(1) 0.1 (2) 0.2 (3) 0.25 (4) 0.15

Directions for questions 7 and 8: In a game of cards played by two gamblers Raju and Ramu, the payout when a card out of King, Ace and Joker turns up, is given below along with the probability of occurrence.

Card Gambler Pay out (in `)

Probability of occurrence

King Raju 80 0.5 Ace Raju 40 0.3

Joker Raju −20 0.2 King Ramu 80 0.5 Ace Ramu 60 0.3

Joker Ramu −20 0.2

Expected pay-out for any gambler is the weighted average of probability of occurrence and payout. 7. What is the expected pay-out for Raju? (1) 40 (2) 48 (3) 60 (4) 70 8. If the probability of getting a King and a Ace for

Ramu is interchanged then what will be the percentage decrease in pay-out after the interchange as compared to the original pay-out?

(1) 7.4% (2) 7.2% (3) 6.25% (4) 12.5%

Direction for question 9: An agency "Book a Hotel" offers its customers an incertive to book a hotel room in advance. It also offers its customers a half-a-day tour and also a tour of the Hunters Valley. The data is given in the following flow chart: Note: S/O – Single Occupancy D/O – Double Occupancy CWB – Child With Bed CWNB – Child With No Bed.

Page 30: `DATA INTERPRETATION REPLICA QUESTIONS THAT HAVE APPEARED IN CAT IN THE LAST 4 YEARS

30

0

0.2

0.4

0.6

0.8

Vis

ibili

ty In

dex

1

• G

• H

• J

• I

• C• K

• X • L • D

• T • A

• R • S

• Q

• M • W • B • U • F

• Y• Z

• N• E

• V • O • P

9. If Ramu, an employee of EMIT, books a hotel room in a 3 star hotel, goes on a city tour and visits the Hunters Valley, then what is the amount spent by Ramu?

(1) $685 (2) $665 (3) $675 (4) $655 10. 3 groups for 3 different modules have to be

made from 4 junior software engineers (JSE) P, Q, R, S and 4 senior software engineers (SSE) X, Y, Z, W.

(1) P and S are in the group. (2) X and Y cannot be in the same group as Q. (3) R is a member of a group which has 3

people. (4) R, Z, W are males. (5) There has to be a male in every module.

(6) There has to be a JSE and a SSE in every module.

The person who is in the group having 3 members is

(1) X (2) Q (3) Z (4) W Directions for questions 11 and 12: The following table shows some integers in a 5 × 5 grid.

a b c d e a 4 13 b 8 12 c 14 10 d 17 12 7 e 8

The numbers in the blocks are placed according to the following conditions. 1. The number in column 'b' and row 'd' is 1/3 the

sum of the numbers in column 'b'. 2. The numbers in column 'a' are squares of prime

numbers in ascending order starting with the first odd prime number.

11. Which of the following is the value of the

number in column 'b' and row 'b'? (1) 23 (2) 19 (3) 17 (4) 16

12. What is the total sum of the numbers in column 'a'?

(1) 208 (2) 312 (3) 216 (4) 373 Directions for questions 13 and 14: Ram, a saree seller bought 400 sarees of 3 different types (Kanchipattu, Benarasi and Mangalagiri) in the ratio 5:3:2. He sells the sarees based on market demand. On the 1st day he sells 20% of the total volume. On the 2nd day he sells 50% of the total volume. On the 3rd day, he sells 30% of the total volume. The cost of each Kanchipattu, Benarasi and Mangalagiri saree is `350, `400 and `375 respectively. On each day he sells the sarees of each variety in the same ratio as he bought. 13. If on the 3rd day he sells Benarasi saree for a

price which is more than 20% than their cost price whereas the other two sarees are sold at their cost price, then find the total amount received by Ram by selling the sarees on the 3rd day?

(1) `45280 (2) `47280 (3) `51280 (4) `49280 14. It is known that 25% of the sarees were slightly

damaged. If Ram managed to sell the sarees which were slightly damaged at 20% loss, then what is the amount made by Ram by selling the damaged sarees. [consider the ratio of the damaged sarees in the three categories is the same as the ratio in which he bought] (1) `37500 (2) `35000

(3) `29600 (4) `28000 15. The population of China in 2009 is 1.6 billion and

it is expected to grow at 12% every year till 2020. If in 2020 the population of China is 15% of the total population of the world, then find the total population of the world in 2020 approximately?

(1) 30 billion (2) 37 billion (3) 45 billion (4) 51 billion

Direction for question 16 and 17: The following graph gives the visibility index of a certain number of people: 16. For what percentage of the total number of

persons is the visibility index more than U? (1) 50% (2) 54% (3) 60% (4) 64%

17. For which pair of persons the visibility index are equal?

(1) F, M (2) C, L (3) R, S (4) A, T

Page 31: `DATA INTERPRETATION REPLICA QUESTIONS THAT HAVE APPEARED IN CAT IN THE LAST 4 YEARS

31

K-1 J-2 A-2 Q-1

K-2 J-1 A-1 Q-3

K-1 J-2 A-1 Q-0

K-2 J-2 A-1 Q-1 K-1

J-0 A-2 Q-2

K-1 J-1 A-1 Q-1

A B

C

DE

F

18. Six friends A-F play a game of cards. The winner of the game is found by adding the points allotted to the cards. The person having the highest number points is declared the winner of the game. The six friends are sitting around a circular table as given below

Who among the following is the winner? (1) A (2) B (3) C (4) E

Quant SI – CI Directions for question 1 and 2: A company offers four schemes for investors to invest. The schemes are as follows. Scheme 1: Gives a return of 8% p. a. interest being

compounded annually Scheme 2: Gives a return of 15% p. a. simple

interest Scheme 3: Gives a return of 10% p. a. interest being

compounded semi-annually Scheme 4: Gives a return of 20% p. a. compounded

annually but 10% of the amount at the end of each year is paid as administrative charges and only the remaining 90% is reinvested.

1. If a person decides to invest an amount of

`20000 equally in scheme 2 and scheme 3, then after how many years will his interest from scheme 3 be more than that from scheme 2? (1) 7 (2) 9 (3) 10 (4) 11

2. If a person invests his entire money equally in all

four schemes, then which scheme will fetch him the maximum interest after 8 years? (1) scheme 1 (2) scheme 2

(3) scheme 3 (4) scheme 4

LA (Venn Diagram) 1. In a class all the students applied in at least one

of the three examinations among CAT, FMS and XAT. 24 students applied for FMS, 32 students applied XAT and 36 students applied for CAT. 12 students applied for both FMS and XAT, 15 students applied for both XAT and CAT whereas 9 students applied for both FMS and CAT. If the number of students who applied for all the three was 25% of those who applied for at least two of the three examinations, then how many students were there in the class?

(1) 54 (2) 60 (3) 62 (4) 69 Directions for questions 2 and 3: Out of 800 persons living in a locality, 50% own a car, 60% can speak in Hindi, and 500 can speak in English. 15% of the total number of persons living in the locality own a car and can speak both in Hindi and English. Of the people who own a car, 240 cannot speak in English. Out of the people who can speak in Hindi, 180 own a car. There is no one in the locality, who does not own a car, cannot speak in Hindi and cannot speak in English. 2. What percent of the people who can speak in

English, can also speak in Hindi? (1) 48% (2) 54% (3) 72% (4) 75%

3. What proportion of the people in that locality do

not own a car or cannot speak in English? (1) 0.44 (2) 0.64 (3) 0.72 (4) 0.8

4. In a survey conducted among 120 families living

in a locality regarding the daily newspaper they read among The Telegraph, The times of India and The Hindu, the following data was recorded. The number of families reading The Hindu, The Times of India and The Telegraph were 64, 48 and 45 respectively.21 families did not read any of the three newspapers whereas 9 families read all three newspapers. How many families read exactly one newspaper? (1) 40 (2) 45 (3) 50 (4) Cannot be determined

Directions for questions 5 and 6: 300 employees work in a company. 20% of the employees are HOD's. 70% of the employees have weekly off on Sunday. 180 employees own a car. Half of the HOD's own a car. All the HOD's have their weekly off on Sunday. 30 employees have a car but do not have their weekly off on a Sunday.

Card Points

K - King 30

J - Jack 20

A - Ace 10

Q - Queen 5

Page 32: `DATA INTERPRETATION REPLICA QUESTIONS THAT HAVE APPEARED IN CAT IN THE LAST 4 YEARS

32

5. What percentage of the company HOD's own a car and have their weekly off on Sunday? (1) 5% (2) 10% (3) 15% (4) 20%

6. How many employees are not HOD's and do not

own a car? (1) 60 (2) 70 (3) 80 (4) 90

7. 300 students passed engineering and chose only

one of the two PG courses – MBA and MS. 70 students chose MS. In MBA, students chose at least one of the three specialisations among Finance, Marketing and HR. 20 chose general specialisation which includes all three specialisations. 100 students chose Marketing. 150 chose Finance. It is also known that the number of students who chose only Marketing and Finance is twice the number of students who chose only HR and Marketing which in turn is equal to the number of students who choose all three specialisations. How many students chose only HR specialization? (1) 60 (2) 50 (3) 20 (4) 30

8. A few children visit a toy shop and buy some

toys. 16 children buy toys of T & J, 26 children buy toys of C & H and 34 children buy toys of B & B. It is known that each child buys exactly two toys. How many children visited the toy shop? (1) 28 (2) 32 (3) 34 (4) 38

Directions for questions 9 and 10: A survey was conducted to find out which of the four movies P, Q, R and S is liked by most people. The number of people who like P, Q, R and S are 24, 36, 29 and 25 respectively. It is further known that – every person surveyed likes at least one movie – the number of people who like P and Q are 12. – the number of people who like R and P, Q and

R are 18 and 16 respectively. – the number of people who like only P, R and S

are 8 and those who like P, Q and R are 6 – people who like S also like R. 9. How many people like only Q? (1) 10 (2) 11 (3) 12 (4) 14 10. How many people like all 4 movies? (1) 7 (2) 6 (3) 5 (4) 4

LA (Miscellaneous) 1. Six pizza stores are ranked as per the price

offered by them on a particular variety of pizza and the time taken by them to deliver the order. The store offering the least price is ranked 1 and when ranked as per the delivery time, the store which delivered the earliest was ranked 1. If more than one store offered the same price or delivered in the same time, then they are given the median of all the rankings. The price ranking is given a weightage of 0.7 whereas the delivery time ranking is given a weightage of 0.3. Study the following table carefully and answer the given question:

Pizza store Price (in `)

Delivery time (in minutes)

A 120 30 B 155 25 C 85 20 D 95 45 E 125 30 F 120 40

Which of the following pizza stores got the third lowest net score? (1) A (2) D (3) E (4) F

Directions for questions 2 to 4: In the Recruitment process, a company asked the people who applied to write four objective papers each consisting of four questions. In the first paper, for every correct answer 15 marks were awarded whereas for every wrong answer 5 marks were deducted. In the second paper, for every correct answer 20 marks were awarded and for every wrong answer 15 marks were deducted, but if a person got all 4 questions correct, an additional 30 marks were awarded. In the third paper, for every correct answer 25 marks were awarded and for every wrong answer 15 marks were deducted. In the fourth paper, 20 marks were awarded for every correct answer and 5 marks deducted for every wrong answer but if a student got three or more questions wrong, 50 marks were deducted from his total. 2. What is the least score obtained by a person in

these four papers, if he gets 4 wrong answers across all four papers? (1) 200 (2) 150 (3) 180 (4) 165

3. What is the difference between the highest marks

obtained by two persons one of whom wrote paper 1 and paper 3 and the other wrote paper 2 and paper 4, each getting 2 wrong answers overall? (1) 20 (2) 30 (3) 40 (4) 60

4. What is the least total score obtained by a person

who got 2 questions wrong and attempted 3 papers? (1) 145 (2) 150 (3) 160 (4) 165

Directions for questions 5 and 6: In a 9 seater van having 3 rows of 3 seats each, the seats in the first row are numbered 1, 2 and 3, those in the 2nd row are numbered 4, 5 and 6 and those in these last row are numbered 7, 8 and 9. The seats are arranged in a grid form with 3 rows and 3 columns. These seats are occupied by 9 persons from P, Q, R, S, T, U, V, W and X not necessarily in this order. The seats are occupied subject to the following conditions.. 1. Q the driver occupies seat 3 and both R and V

are in his column. 2. P is the son of W and occupies the seat in the

same row as V and S. 3. T occupies seat 5 and is in the same row as W

and is not seated immediately in front of S. 4. X and T are not seated in the same column.

Page 33: `DATA INTERPRETATION REPLICA QUESTIONS THAT HAVE APPEARED IN CAT IN THE LAST 4 YEARS

33

5. If persons sitting in the seats swap their positions in the following order (1,4) (2,6) (4,6) (5, 7), (4, 5), (7,2), then which of the following persons will be seated beside U? (1) X (2) R (3) S (4) T

6. If swapping of seats is done in the order (1,2)

(2,8) (9,2) (4,9), (7,5) (5,8) (8,4), then which of the following is true? (1) S is in the fifth seat. (2) W is in the ninth seat. (3) T is in the second seat. (4) P is in the second seat.

Directions for questions 7 to 9: A survey was conducted in four colonies A, B, C and D whose populations are in the ratio 3 :5 : 3 : 4, to find (i) the proportion of residents who prefer watching movie to eating out & (ii) the proportion of people who prefer surfing the internet to chatting with friends. The following was the data collected.

Proportion of residents

who preferred watching movies

Proportion of residents who

preferred surfing

Colony A 0.6 0.64 Colony B 0.45 0.55 Colony C 0.65 0.68 Colony D 0.48 0.75

7. If 61% of the residents in colony C preferred both

watching movies and surfing the net, then what percentage of the residents in colony C did not prefer any of the two? (1) 30% (2) 28% (3) 25% (4) 35%

8. In which colony do the maximum number of residents prefer chatting with friends?

(1) A (2) B (3) C (4) D 9. In how many colonies is the number of residents

who prefer surfing the net more than the average number of residents in the four colonies who prefer the same? (1) 0 (2) 1 (3) 2 (4) 3

10. There are eight oil tanks A, B, C, D, E, F, G & H

in a refinery by which oil is being transferred. Due to the lack of sufficient connecting routes, oil cannot be transferred from any tank to any other tank but can only be transferred subject to the following conditions. (a) Oil can be transferred from A to E or F (b) From E oil can be transferred to A, C or D (c) From D oil can be transferred to A, B or C (d) From F oil can be transferred to D or G (e) From H oil can be transferred to A or E (f) from G oil can be transferred to E, C or H (g) from C oil can be transferred to A or G (h) from B oil can be transferred only to E.

In how many ways can oil be transferred from tanks B to tank H if the same tank cannot be

revisited in the route, i.e. oil can pass through a particular tank only once? (1) 5 (2) 6 (3) 7 (4) 8

11. Arun, Varun and Kiranmala, subscribed to three

business magazines, among India-Day-to Day, India-Everyday and India-These-Days subject to the following conditions: 1. If Arun subscribes to India-Day-to-Day, then

Varun subscribes to India-Every-day. 2. If Arun subscribes to India-Every-day, then

Kiranmala subscribes to India-These-Days. 3. If Varun subscribes to India-These-Days,

then Arun subscribes to India-Every-Day. 4. If Kiranmala subscribes to India-Day-to-Day,

then Varun subscribes to India-These-Days. 5. If Kiranmala subscribes to India-Every-Day,

then Arun subscribes to India-These-Days 6. If Varun subscribes to India-Day-to-Day, then

Kiranmala subscribes to India-Every-Day. From the above constraints, which of the following can be definitely concluded, if it is given that each person subscribed for exactly one business magazine. (1) Arun subscribed to India-Day-to-Day (2) Kiranmala subscribed to India-Every-day (3) Varun did not subscribe to India-These-Days. (4) More than one of the above.

12. If it is given that Kiranmala did not subscribe to

India-These-Days, then which of the following magazines did Varun subscribe to?

(1) India-Day-to-Day (2) India-These-Days (3) India-Every-day (4) Cannot be determined 13. There are 6 baskets 1, 2, 3, 4, 5 and 6. Two

baskets have gold, two have silver and the other two have nothing in them. They are placed one beside the other in a row. Rajini, a contestant, has to select one of the baskets and can get whatever is present in the basket. No two baskets having the same thing inside are kept side by side. What is the probability that Rajini goes home by taking something?

(1) 61

(2) 32

(3) 31

(4) 21

14. In a forest there are P tigers and a goat. Grass is

eaten by both the goat and the tigers but the tigers would rather prefer eating a goat. If a tiger eats the goat then the tiger transforms into a goat and hence runs the risk of being eaten by another tiger. The tigers are intelligent and would not risk being eaten.

Which of the following is true? (1) If P is odd, no tiger eats the goat.

(2) If P is odd tiger close to the goat eats the goat.

(3) For any value of P, the goat is not eaten. (4) For any value of P, the goat gets eaten.

Page 34: `DATA INTERPRETATION REPLICA QUESTIONS THAT HAVE APPEARED IN CAT IN THE LAST 4 YEARS

34

Directions for questions 15 and 16: During his holidays Ram wanted to learn Tabla and Piano. Ram went to learn Tabla for 12 days and to learn Piano for 8 days. 15. If on 4 days he went to learn both the

instruments, then on how many days in all did Ram go to learn playing musical instruments?

(1) 8 (2) 10 (3) 14 (4) 16

16. It is known that on 6 days he went to learn only Tabla and not Piano, then on how many days in all did Ram go to learn playing musical instruments?

(1) 8 (2) 10 (3) 14 (4) 16

Directions for question 17: The following gives the process of arranging words by a word arrangement machine. Read the pattern below and answer the questions.

Input Sunday Monday Tuesday Wednesday Thursday Friday Saturday

Step 1 Saturday Sunday Monday Tuesday Wednesday Thursday Friday

Step 2 Friday Saturday Sunday Monday Tuesday Wednesday Thursday

Step 3 Thursday Friday Saturday Sunday Monday Tuesday Wednesday

17. Which step will be the same as the input? (1) Step 24 (2) Step 26 (3) Step 29 (4) Step 28

LA (Circular Arrangements) 1. Eight persons A, B, C,D,E,F,G and H are seated

around a circular table, such that each person has another person seated exactly opposite him. B is seated opposite to E who is seated to the immediate right of G.D is seated between E and C. If F is not seated adjacent to A, then who is seated opposite to H? (1) D (2) C (3) G (4) either C or D

2. Ten persons P, Q, R, S, T, U, V, W, X, and Y are

seated around a circular table with equal distance between any two adjacent persons such that any person can only see the person seated opposite him and the two persons seated on either side of that person (i.e on either side of the person seated opposite). Further the following information is known. P does not want to see Q, R, S, or T but wants to see X and Y. U does not want to see R, S, T or V, but wants to see P, W and Q.

T wants to see X and Y. Which of the following gives the correct seating arrangement?

(1) P T R U V X Y S Q W (2) P W Q V Y X U R S T (3) P S T R U Y X VQ W (4) P Q W V Y X R U S T 3. A company "XYZ Ltd" asks some of its Branch

coordinators to attend a Round Table Conference.

The coordinators sit around a circular table.

– The Mumbai coordinator sits to the left of the Bengaluru coordinator, who is opposite the Hyderabad co-ordinator

– The Delhi co-ordinator is not opposite the Chennai coordinator but is to the left of the Hyderabad coordinator

Who is opposite to the Delhi coordinator? (1) Mumbai co-ordinator (2) Chennai co-ordinator (3) Bengaluru co-ordinator (4) Hyderabad co-ordinator

4. Six employees P, Q, R, S, T and U of EMIT Pvt

Ltd. go to a party. They sit around a circular table. P and S do not sit together and R and T always sit together. In how many ways can they be seated?

(1) 12 (2) 24 (3) 6 (4) 18

LA (Distribution) Directions for questions 1 to 3: Twelve persons who are members of a rock band live in a building having twelve floors from the 1st floor to the 12th floor with the ground floor and the basement being used for parking. All the persons live in distinct floors. Among them there are 4 singers, 3 guitarists, 2 drummers, 2 instrumentalists and 1 keyboard player. There are seven male members in the group A, B, C, D, E, F and G and 5 female members P, Q, R, S and T. These members live in the building subject to the following conditions: 1. No two female members lived on adjacent floors

and no singer lived on floors at the extreme ends, i.e. the topmost floor or the bottom most floor.

2. D and R were both guitarists and lived on adjacent floors but neither of them lived on the topmost or the bottommost floor.

3. P and C were both singers and had exactly one guitarist living on an adjacent floor.

4. A was neither a drummer nor a guitarist whereas F was not a drummer.

Page 35: `DATA INTERPRETATION REPLICA QUESTIONS THAT HAVE APPEARED IN CAT IN THE LAST 4 YEARS

35

5. T lived on a floor adjacent to at least one drummer where as Q, the keyboard player lived on the 7th floor.

6. All the singers were 3 floors apart from each other where as all the guitarists were at least 2 floors apart from each other.

7. There were 3 floors between Q and B, who was an instrumentalist, and there was only one female guitarist.

8. Both the drummers were males and neither lived in the topmost floor or the bottommost floor.

1. How many male singers were there? (1) 1 (2) 2 (3) 3 (4) 4 2. How many floors were above the floor on which

G lived? (1) 4 (2) 6 (3) 8 (4) Cannot be determined

3. Who lived on the second floor? (1) A (2) S (3) F (4) Cannot be determined Directions for questions 4 and 5: Six couples got married on different dates in the years between 1993 and 1999. Only two couples got married in the same year and only two couples got married in the same month. The couple who got married on April 23rd, got married before the couple who got married on October 15th, but after the couple who got married on November 5th. There is only one couple who got married in 1993 and they got married in the same month as the couple who got married in 1998, there being only one couple who married in 1998. The couple who got married on October 15th was not the last couple to got married and the couple who got married on January 3rd was not the 3rd couple to have got married. Two couples got married on the same day in consecutive months but not in the same year. 4. Both the couples who got married in the same

month were married in the month of (1) October. (2) April. (3) November. (4) January.

5. What was the least difference between the

marriage dates of any two couples (in days)? (1) 49 (2) 78 (3) 160 (4) None of these

6. From a group of nine friends consisting of three boys Jalan, Jagan, and Jeevan and six girls Kekul, Kokila, Kavya, Kavita, Kaya and Kadambari, three groups each consisting of three members are formed from the above persons subject to the following conditions: 1. Kavya and Jeevan should be there in the

same group. 2. Kavya and Kekul should not be there in the

same group. 3. Kokila wants Jalan in her group. 4. Kaya and Kavitha must be in the same

group. 5. Jalan prefers not to be in the same group as

Kekul.

6. In a group, the number of boys must not be more than the number of girls.

Jagan is in the same group as (1) Kaya and Kavita. (2) Kaya and Kadambari. (3) Kavita and Kadambari. (4) None of these

Directions for questions 7 and 8: There are 5 cricketers, L, M, N, O and P from different countries (India, Australia, S.A, SL, England) and playing for 5 different teams in IPL (KKR, RCB, DC, DD, MI). These cricketers are made to stand in a row for post match presentation. – L, who is from SL, does not play for MI and is

standing at the extreme right end of the row. – The cricketer who is to the immediate left of N

plays for KKR and neither of them are from Australia.

– The cricketer who is from India plays for RCB, and is adjacent to only one player P who plays for DC.

– O is to the immediate left of L, and is from SA. 7. What is the correct combination of country and

the team for which M plays? (1) India, RCB (2) England, KKR (3) England, MI (4) SA, KKR 8. The player from Australia plays for which team in

the IPL? (1) DD (2) MI (3) KKR (4) DC 9. Four faculty members A, B, C, and D visited a

college on 4 consecutive days starting from Monday and taught 4 different subjects – Maths, Physics, Chemistry and Biology. Further it is known that A and C went to the college on consecutive days and taught Biology and Chemistry respectively. B went to the college on the last day and did not teach Physics. If Physics was taught immediately after Chemistry then when was Biology taught? (1) Monday (2) Tuesday (4) Wednesday (3) Thursday

Directions for questions 10 and 11: In a hockey tournament, 15 teams from ‘A’ to ‘O’ participated. The teams are arranged in ascending order of the points they scored in the tournament. The following information is know. (a) Team L scored the least number of points, i.e 96 (b) Team I scored the maximum number of points, i.e

364 (c) Team D, team G and team C were placed 4th, 7th

and 10th respectively with 116, 182, and 218 points.

(d) Team N and team B got 108, 165 points less than Team F.

(e) The points scored by Team 'O' = 251 (f) Team A got 361 points. (g) Team H got 4 points more than team M and team

E got 18 points less than Team N.

Page 36: `DATA INTERPRETATION REPLICA QUESTIONS THAT HAVE APPEARED IN CAT IN THE LAST 4 YEARS

36

(h) The sum of the total points for the following positions are as follows.

1 + 2 + 3 = 302; 4 + 5 + 6 = 412; 7 + 8 + 9 = 590; 10 + 11 + 12 = 753; 13 + 14 + 15 = 1046 (i) The ascending order of the teams is O, J, F, OJF. 10. What is the difference in the points obtained by

teams J and K? (1) 136 (2) 144 (3) 151 (4) 161

11. What is the position of team E?

(1) 7th (2) 8th (3) 9th (4) 10th Directions for questions 12 and 13: Seven friends P, Q, R, S, T, U and V went to a restaurant. Each friend is wearing a different coloured shirt among Violet, White, Orange, Blue, Green, Yellow and Red. They order seven different cool-drinks among Pepsi, Thums up, Sprite, Mazaa, Coke, Mountain dew and Fanta. – P, who wears an orange shirt drinks Sprite. – The friend who wears a green shirt drinks Pepsi. – R wears a red shirt. – U wears a blue shirt. – T drinks Thums up and Q drinks Coke. – The friend who wears a green shirt, the friend

who drinks Fanta and V ordered the same dish. – The friend who wears the violet shirt drinks

Mazaa. 12. Who among the following drinks Pepsi? (1) Q (2) S (3) T (4) V 13. Which of the following is a correct combination of

person, colour of shirt worn and the cool-drink he ordered?

(1) V, violet, Maaza (2) U, blue, Fanta (3) R, red, Mountain Dew (4) Q, yellow, Coke Directions for question 14 and 15: P, Q, R, S, T, U, V, W and X are students learning a foreign language. and were divided into 3 groups of 3 each for the purpose of presenting a seminar. Group I P, Q, R Group II S, T, U Group III V, W, X

But at the time of the seminar, some of the students were missing. So the teacher had to change the order of making presentations. – X presented before T. – R presented before V but after U. – Q presented after W, P and S – W presented the seminar before U and X but

after P and S. 14. Which among the following is the group of

students who presented their seminar in the 1st group?

(1) W, P, S (2) W, U, X (3) P, S, U (4) P, Q, R 15. Who among the following cannot be the first

person to make his presentation in the 2nd group? (1) X (2) U (3) Q (4) V 16. Seven students P, Q, R, S, T, U and V go to

tuition in different subjects-Mathematics, Physics and Chemistry. Out of 7 students, 3 were girls. It is further known that

– R goes for Physics tuition – P and U go to the same tuition – Every tuition is attended by at least 2 people – Q who goes to Maths tuition does not go to

the same tuition as S. – T and V go to same tuition.

Who among the following definitely goes to the tuition in which 3 students attend?

(1) S (2) T (3) V (4) Q

Quant ERPV 1. Two auditoriums are designed to be constructed

such that their floors are square shaped with the ratio of the sides of the squares being 1 : 2. A

group of N workers start working in A1. After 61th

of a day, 4

3th

of the workers working in A1 move

over to A2 and start working. At the end of the day the work in A1 just gets completed and the work in A2 gets completed when N workers work

for 87

th of the next day as well. Find the value of

N. (1) 24 (2) 36

(3) 48 (4) Cannot be determined

Page 37: `DATA INTERPRETATION REPLICA QUESTIONS THAT HAVE APPEARED IN CAT IN THE LAST 4 YEARS

37

12%

28%

16%18%

26%

4540

35

3034

0

5

10

15

20

25

30

35

40

45

50

Trucks Buses Cars 2 wheelers Others

Acc

iden

t Sev

erity

inde

x (p

erso

n ki

lled

per

100

acci

dent

s)

Line + Bar graph

Directions for questions 1 to 4: The following bar graph gives the accident severity index and the type of vehicles responsible for accidents in the year 2005. A total of 75000 accidents occured during 2005. 1. How many persons were killed in 2 wheeler

accidents? (1) 6780 (2) 7200 (3) 8400 (4) 9600

2. The ratio of people killed to people who got injured in the accidents is the highest for (1) 2 wheeler accidents (2) Other types of accidents (3) Car accidents (4) Truck accidents

3. How many persons were injured by car

accidents? (1) 6400 (2) 7800 (3) 4200 (4) 7200

4. By how much is the number of persons killed in truck accidents more than the persons injured in accidents caused by other types of vehicles? (1) 1440 (2) 1720 (3) 1480 (4) 1680

DI (Distribution)

Directions for questions 1 and 2: Atul has 6 policy accounts which are going to mature in between the years 1997 to 2002 (both the years included). Two policies have their maturity in the same month whereas two policies have their maturity in the same year. One of the policies matures on Feb 29th. It matures before the policy which matures on January 10th but after the policy which matures on August 8th. The first policy matures on August 24th 1997. The policy which matures on September 17th matures immediately before the policy which matures on May 21st which inturn is the last policy to mature. 1. Which of the following policies is the 3rd to

mature? (1) Feb 29th, 2000 (2) Jan 10th, 2001 (3) August 8th, 1999 (4) August 8th 1998

2. In which of the following years did Atul receive

money from 2 policies? (1) 2000 (2) 2001 (3) 1999 (4) 2002

LA(Linear Arrangement) Directions for questions 1 to 3: Ten friends who stay in 3 different countries among USA, UK and Australia meet at a reunion party in their school in India. They stand in a row according to their roll numbers in the school. There are 5 male friends among P, Q, R, S and T and 5 female friends among F, G, H, I and J. 5 friends stay in USA, 3 stay in the Australia and 2 stay in the UK. – No two friends from USA stand next to each

other. – P is from Australia and stands in between I and J. – The friends from UK are at least 4 places away

from each other. – S is from UK and is 3rd in the row from the left

end. – The extreme ends are occupied by the friends

who stay in USA. They positions are of different genders.

– F, Q, G stand in the 7th, 6th and 5th from the right end of the row respectively. Two of these stay in the USA.

– R is adjacent to I. 1. How many female friends stay in the USA?

(1) One (2) Two (3) Three (4) Four

2. How many friends stand in between friends from

UK? (1) Three (2) Four (3) Five (4) Six

3. Who among the following is the male friend from the

USA? (1) Q (2) T (3) R (4) Both A and B

Directions for question 4 and 5: Seven friends Pradip, Qureshi, Raju, Shyam, Tarun, Uttam and Vamsi stand in a row. It is further known that

Page 38: `DATA INTERPRETATION REPLICA QUESTIONS THAT HAVE APPEARED IN CAT IN THE LAST 4 YEARS

38

(i) Shyam is at the extreme left end of the row. (ii) Qureshi is two places to the right of Tarun. (iii) There are exactly 2 persons in between Uttam

and Vamsi. (iv) Raju has the same number of people to his left

as to his right. 4. Who is standing 3 places away to the left of

Qureshi? (1) Tarun (2) Shyam (3) Pradip (4) Raju 5. Who is standing to the immediate right of

Shyam? (1) Qureshi (2) Uttam (3) Pradip (4) Vamsi

LA (Sequencing) Directions for questions 1 to 3: Five students A, B, C, D and E are arranged according to their marks under the following conditions – C got more marks than A but less than E. – D got less marks than 3 other students. – Two students got the same marks. – The first and the last ranked students did not get the same marks as any other student.

1. Which student got the highest marks? (1) A (2) B (3) C (4) E

2. Which two students got the same marks?

(1) B and D (2) B and C (3) A and D (4) C and D

3. Which of the following is the descending order of

the students according to their marks? (1) ECBDA (2) EACDB (3) EBADC (4) ECADB

Directions for questions 4 and 5: Five movies P, Q, R, S, T were set to release on every Friday of a month. P was neither the first nor the last movie to be released. Q was released immediately before R and there were at least 2 movies to be released before R. There was one movie released in between S and T (Assume that no movie was scheduled to release during this period apart from these 5) 4. Which of the following movies was released last? (1) S (2) T (3) R (4) Q 5. How many movies were released before P? (1) 1 (2) 2 (3) 3 (4) 4

Line Graph + Table

Directions for question 1: The following line graph gives the details of the production of different plants per unit area: Profit/ton = Revenue/ton – Cost/ton The following table gives the cost and revenue obtained by these plants per ton in 2000.

Coffee Tea Rubber Steel

Cost/ton `12340 `13460 `8900 `15800

Revenue/ton ``15610 `14280 `12300 `19960

1. Which of the following plants has the maximum profit in 2000? (1) Coffee (2) Tea (3) Rubber (4) Steel

RubberTeaCoffee

Steel

0

10

20

30

40

50

1996 1997 1998 1999 2000

Coffee

– –Tea –– Rubber – – Steel

Page 39: `DATA INTERPRETATION REPLICA QUESTIONS THAT HAVE APPEARED IN CAT IN THE LAST 4 YEARS

1

SOLUTIONS FOR DATA INTERPRETATION REPLICA QUESTIONS THAT HAVE APPEARED IN CAT IN THE LAST 4 YEARS

TABLES

Solutions for questions 1 to 3: 1. Let the volume of data transfer in India and Singapore

be 100 units each. Revenue from data transfer in India = 100 × 1 = $100 Revenue from data transfer in Singapore = 100 × 9 = $900

Total revenue in India = 100 × 9

100 = $1111

Total revenue in Singapore = 900 × =21

100$4285

∴Total revenue in Singapore is about 4 times that in India. Choice (5)

2. Revenue from data transfer as a percentage of total

revenue for India in 2010 = 27% Revenue from data transfer as a percentage of total

revenue for Sweden in 2010 = 36% Let total revenue in India in 2010 be $200 and that in

Sweden be $100 ARDT of Sweden = $6

∴Volume of data transfer in Sweden = 66

36 =

∴Volume of data transfer in India = 6

∴ARDT in India = 96

54 =

∴ The percentage increase = 1001

19 ×−= 800%

Choice (3) 3. It can be seen that if the total revenue received is the

same for the given pairs of countries, only UK and Spain would have approximately the same volume of data transfer. Choice (4)

Solutions for questions 4 to 6: 4. To get calls from all the colleges, Arun should have

scored at least the highest value of cut-off in each section, i.e., 44 + 44 + 45 + 44 = 177 and also at least the highest value of aggregate cut-off for any institute, i.e., 176. Choice (2)

5. The minimum aggregate marks to get calls from two

colleges is 171. If he scores 50 each in three sections he needs to score at least 21 marks in the fourth section. Choice (3)

6. Four colleges have a cut-off for section C and the

remaining two colleges have a cut-off for section D. ∴ If a student misses the cut-off in these two sections,

he/she would miss calls from all the colleges. The maximum possible marks such a student gets is 50 + 50 + 40 + 42 = 182. Choice (3)

Solutions for questions 7 to 9: 7. The new gross pay of the employee transferred

= 16,000 + 10080 × 16000 = 16,000 + 12,800 = 28,800

The gross pay of the current employees in HR department = 16000 × 5 = 80000

New gross pay of the six employees = 80,000 + 28,800 = 1,08,800

Average gross pay = 181336

108800 =

∴ percentage increases = 10016000

1600018133 ×−

� 13% Choice (3) 8. As after the mutual transfer, the average age of the

Finance department increase by one, it means that the age of the person who came from the Marketing department was 20 years older than the age of the person who was transferred from the Finance department. Now after the transfer of the employee to the HR department, as the average age of the employees left in the Marketing department remained the same, the age of the employee transferred to the HR department, was 20 years younger than the average age, i.e., 36 – 20 = 16 years.

∴The new average age of the employees in the HR department

= 416

2466

116546 ==×+×years Choice (3)

9. The new average basic pay of employees in the HR

department = 8

1000,162000,125000,10 ×+×+×

= 8

000,16000,24000,50 ++ = 11250

890000 =

The percentage change = 12.5% Choice (2) Solutions for questions 10 to 13: 10. The drink must contain 10% minerals. As there are only

two drinks (A and C) with 10% minerals, the drink can be prepared in only one way. As A and C have 30% protein each, they can be mixed to form the drink. Choice (1)

11. None of the choices (1), (2) and (3) can be used to form

the drink with 10% fat and at least 30% protein. For C and E to form the drink with 10% fat and at least 30% protein, if they are mixed in the ratio x : y (say)

( ) ( )yx

050x+

+= 10, x : y = 1 : 4

∴ cost per unit = ( ) ( )

5600

510042001 =+

= 120

Similarly the ratio for D and E is 1 : 3 and the cost per

unit is 4

800= 200

∴ The cost per unit is the least for C and E. Choice (4) 12. The drink should have at least 60% carbohydrate.

Further in the mixture formed by B, C and E, the proportion of B should be maximum and the other two should be minimum to get the lowest per unit cost. Among the given options only Choice (2) and (5) satisfy the condition having 60% carbohydrate and of these, choice (5) has the lowest per unit cost. Choice (5)

13. A and B when mixed in equal proportions, the protein

content will be only 2

2030 += 25%, which is less than

required. D and E when mixed in equal proportion, the

Page 40: `DATA INTERPRETATION REPLICA QUESTIONS THAT HAVE APPEARED IN CAT IN THE LAST 4 YEARS

2

carbohydrate content will be only 2455+

= 25% which is

less than required. Similarly B and E and C and D when mixed in equal proportion the combination will have less than the required percentage of minerals and carbohydrate respectively. Only A and E when mixed in equal proportion would yield a mixture with all the contents in the required amount. Choice (5)

Solutions for questions 14 to 17:

14. If one observes the values given for the different parameters, the values that were varying with production, i.e., value was increasing when production increased and value decreasing when production decreased are material, labour and operating cost of machines. All the remaining costs, i.e., rent of building, consumables, rates and taxes, repair and maintenance expense and selling and marketing expenses are fixed. Hence, there will be no change in these costs. The total fixed cost = 1800 + 600 + 1200 + 8700 + 2100 = 14400 The cost/unit for different variable costs is as follows. Material = `50 per unit. Labour = `20 per unit Operating cost of machine = `30 per unit Total = `100 per unit Selling price per unit = `125 per unit

Total cost/unit for 2100 units is `100 + 2100

14400

= `107 Choice (2)

15. For one product, Selling price = `125 Variable cost = `100 _________________ Difference = `25 _________________

Now, to avoid loss, the company has to offset the fixed cost (i.e., 14400) for which it has to produce a total of

25

14400= 576 units. Choice (3)

16. The reduction in selling price per unit = 5% of 125 = 6.25

New selling price = 118.75 Total fixed costs = `14400 Variable cost per unit = `100

Now the total profit increases with the increase in number of units sold and the maximum profit is obtained when the company sells and 3000 units. Choice (5)

17. The given condition is that if the company sells upto 2100 units, the selling price per unit is `125 and if the company sells 2550 units, the selling price per unit for all the units is `120. The profit of the company increases upto a production figure of 2100 units, from the 2100th unit to the 2101st unit, the total profit decreases drastically and from the 2101st unit to the 2550th unit, the profit again increases.

Hence, the profit would be maximum at the production figure of 2100 units or at 2550 units.

Production 2100 units 2550 units Selling price / unit (s) `125 `120 Variable cost / unit (v) `100 `100 S – V `25 `20

(S – V) production 25 × 2100 = 52500

20 × 2550 = 51000

Total fixed cost 14400 14400 Total profit 38100 36600

The maximum profit is `38100 Choice (1)

Solutions for questions 18 to 21: 18. The costs of a refrigerator, an air conditioner and a

music system in different countries are. (’00 U.S. dollars)

India Thailand Malaysia Singapore USARefrigerator 11 + 5 13 + 5 11 + 6 13 + 4 20

Air conditioner 9 + 7 12 + 5 10 + 8 12 + 5 23 Music system 8.5 + 9 10 + 6 8 + 4 13 + 4 20

Total 49.5 51 47 51 63

The cheapest is in Malaysia. Choice (3) 19. As given in the previous question, the total cost will be

highest in India (850 + 900 = 1750) Choice (1) 20. Cost in India = 300 + 500 = 800 Cost in Thailand =450 + 600 = 1050 Difference = 250 × 32.9 = 8225 Duty = 1500 Required difference = 6725 Choice (4) 21. Cost in India with dollar at `40.92 = 550 × 40.92

⇒ 2500

Cost in India with dollar at 35 = 35

22500= 650

Cost in Singapore = 900 Required difference = 250 Choice (2)

Solutions for questions 22 to 26: 22. Let us check the possible short routes from A to J. Total cost Total distance

A 280

335.RsB

km11501135.Rs

J `1470 1430 km

A km25.Rs

625.RsD

km8251225.Rs

J `1850 1250 km

A km670

850.RsF

km485575.Rs

J `1425 1155 km

A km675

1225.RsG

km485445.Rs

J `1670 1090 km

A km975

925.RsH

km200210.Rs

J `1135 1175 km

A km395

675.RsC

km205215.Rs

F km485

575.RsJ

`1465 1085 km

The shortest possible route is A – C – F – J. The cost is `1465. Choice (4)

23. The route with the least cost is A – H – J, with a total

cost of `1135. As the cost of the new service is 5% less then `1135, it should be 1135 – (5% of 1135) = 1078. Choice (2)

24. If C, D and H are closed, then the minimum cost of

travel is for A – F – J, i.e., `1425. Choice (3)

25. We want the cetandis

icePr to be as minimum as possible.

It is less than 1 in only the cases A – H, B – J and

Page 41: `DATA INTERPRETATION REPLICA QUESTIONS THAT HAVE APPEARED IN CAT IN THE LAST 4 YEARS

3

C – D. Considering the cases involving the above routes.

Route Price / Distance Taking margin of 10% into Account

A – H – J 11751135

11751135

× 1110

A – B – J 14301470

14301470 ×

1110

It will be the least for A – H – J and is 11

10

1175

1135 ×

= 5.1172.103

= .88 Choice (2)

26. The cost / kilometer is the least for A – H – J and the

distance is 1175 km. Choice (4) Solutions for questions 27 to 30: With the given information we can deduce the number of males and postgraduates in the different departments as follows:

Department Total Male Post graduates Marketing 80 48 32 Accounts 80 44 40

Production 640 288 352 Total 800 380 424

27. Percentage of male employees in the production

department = 640

288 × 100 = 45 Choice (2)

28. Post graduates in the marketing department = 32

Male postgraduates = 10025 × 32 = 8

∴ Female post graduates = 32 – 8 = 24 Male non post graduates = 48 – 8 = 40 Required difference = 40 – 24 = 16 Choice (5)

29. Percentage of male post graduates in the marketing

department = 80

32 × 100 = 40 Choice (1)

30. The number of male post graduates in the production

department = 144. ∴ Female post graduates = 352 – 144 = 208 The number of male and female post graduates and male and female employees who are not post graduates are as follows.

Post graduate Non Post graduates

Male Females Males Females

144 208 144 144

It can be seen that except female post graduates all other groups (male posts graduates, male and female non post graduates) have the same number of employees. Choice (3)

Solutions for questions 31 to 33: 31. The total number of bookings made is the highest in Q3 and so the average number of bookings per month is also the

highest. Choice (C)

Month Jan Feb March April May June July Aug Sep Oct Nov Dec Number of bookings 346 412 380 450 308 359 462 333 345 250 506 370

Number of deliveries 200 216 (146)

160 (196)

225 (220)

170 (225)

159 (138)

296 (200)

134 (166)

50 (199)

125 (295)

278 (125)

32. The values shown in the brackets are of the booking

made 2 months ago. Number of deliveries made in August from the bookings made in June = 200. Number of deliveries made in December from the bookings made in November = 278

.39.1200

278= Choice (A)

33. We only need to check the revenue for quarters Q3 and

Q4. Revenue (in `) from Q3 = (462 + 333 + 345) × 43,100 = 4,91,34,000 Revenue (in `) from Q4 = (250 + 506 + 370) × 44,000 = 4,95,44,000 ∴ the highest revenue is obtained from Q4 i.e., `4,95,44,000 Choice (B)

Solutions for questions 34 to 36: 34. The given condition occurs in the case where the

‘number of wins’ is in the range 16 – 18. Choice (B) 35. From the given table, the 3rd least percentage occurs in the

last row i.e., for 31–33, which is 95

9499 −× 100 =

19

100

= 9

55 % Choice (C)

36. The given condition is satisfied in the case, where the ‘number of wins’ is in the range 25 – 27 i.e.,

653

636468=

++ = 2.5 × 26 Choice (A)

Solutions for questions 37 to 39: 37. Investment (in `) in NLP Industries before withdrawal

= 12.5% (12,00,000) = 1,50,000 Investment (in `) in NLP Industries after withdrawal = 16% (9,00,000) = 1,44,000 ∴ the percentage change in investment

= 100000,50,1

000,44,1000,50,1×

− = 4% Choice (D)

38. The return on investment for Mr. Anil

= 100

25

100

2× × 7,00,000 = 3500

The return on investment for Ms. Shivani

= 100

10

100

5.2× × 13,00,000 = 3250

Therefore the required difference = (3500 – 3250) = `250 Choice (A)

39. The three persons A, B, and C made an investment of

`10 lakh, `20 lakh and `21 lakh respectively such that their investments fall under the schemes X, Y, Z respectively.

Page 42: `DATA INTERPRETATION REPLICA QUESTIONS THAT HAVE APPEARED IN CAT IN THE LAST 4 YEARS

4

Their combined return on investment (in ` ) = 2% (10,00,000) + 2.5% (20,00,000) + 3% (21,00,000) = 20,000 + 50,000 + 63,000 = 1,33,000 Their combined return on investment after the firm increased the rate of return = 2.2% (10,00,000) + 3% (20,00,000) + 3.3% (21,00,000) = 22,000 + 60,000 + 69,300 = 1,51,300 ∴ The required increase (in `) = 1,51,300 – 1,33,000 = 18,300 Choice (D)

Solutions for questions 40 and 41: 40. Percentage contribution of mono speaker of the

company NOSY in 2001 = %3.121008100

1000=×

Percentage contribution of mono speaker of the

company BOSS in 2001 = %171009400

1600=×

Percentage contribution of mono speaker of the companies NOSY and BOSS in 2003 are 13.5% and 19.6% respectively. ∴ the percentage contribution of mono speakers of both the companies increased. Proceeding, similarly we observe that for no other type of music systems of both the companies, the percentage contribution increases. Choice (B)

41.

Type of music system Percentage contribution

in 2001

Percentage contribution

in 2003 Mono speaker 12.3% 13.54% Dual speaker – 1000w 22.22% 22.9% Dual speaker – 2000w 28.4% 18.75% Four speakers – 5000w 17.3% 23.95% Home theatre 19.75% 20.8

The maximum change in percentage points occurs for Dual speaker – 2000W. Choice (C)

Solution for question 42: 42. The total number of cars sold by showroom A and

showroom B at the end of 7 days are 209 and 221 respectively.

= %.5.94100221

209≈× Hence statement Ι is true.

The total number of cars sold by showrooms on odd numbered days = 16 + 35 + 33 + 51 + 60 = 195 The total number of cars sold by showroom B on even numbered days = 19 + 42 + 29 + 52 + 81 = 223. 90%(223) = 200.7 ∴ Statement ΙΙ is also true. Choice (C)

Solutions for questions 43 and 44:

43. Hyderabad:240

10126 5×= 52,500 > 51,860;

180

1075 5× < 42,500

∴ a restaurant in class B center but not class A center will earn more revenue then the establishment fee in one year.

Bengaluru: 240

10144 5× = 60,000 < 60,200

180

1090 5× = 50,000 < 50,246

The franchise in Bengaluru will earn more revenue then the establishment fees (in each of the two centres) after one year. Choice (D)

44. If a customer spends on an average `300 and `130 at a

Foodie restaurant in class A center and class B center respectively, then the total number of customers who are required to come such the revenues are not less than the establishment fees would be the i.e.,

300

10132 5× +

130

10104 5× = 44,000 + 80,000 = 1,24,000

Choice (B) Solutions for questions 45 and 46: 45. As no information is given regarding the percentage of

dropouts for districts R and S in few years, (A) cannot be definitely a false statement. As no information is given about the number of enrolments in each districts in any of the years, statements (B) and (D) cannot be confirmed. (C) is definitely false because the dropout percentage of district Q in any of the given years is greater than that of each of the other districts. So, the overall dropout percentage would also be the highest. Choice (C)

46.

District Minimum number of achievements P 5 Q 1 R 2 S 0 T 2

∴ total number of achievements (minimum = 5 + 1 + 2 + 0 + 2 = 10) Choice (B)

Solutions for questions 47 and 48:

47. The rise in temperature (in °C) per hour = 7

2546 − = 7

∴ temperature (in °C) in city Q at 10 a.m. = 3 × (10 – 5) + 25 = 40 Choice (D)

48. Temperature in city P at 3.30 p.m.

= 42 –

−6

2942 × 32

1

= 42 – 12

91 ≅ 34.5°C

Similarly, City Temperature at 3.30 p.m. Q 35.5 °C R 37 °C S 38.5 °C T 36.66 °C U 33.5 °C

∴ city S has the highest temperature at 3.30 p.m. Choice (C) Solutions for questions 49 to 52:

P Q R S T U Traffic flowing from 3,346 3,752 2,536 2,620 2,952 3,060

Traffic flowing to 3,504 2,612 3,308 2,852 3,050 2,940 49. Total traffic through the route P – Q = 964 + 846 = 1810.

Similarly verifying it is easy to see that the maximum traffic flow occurs through the road connecting PQ.

Choice (A)

Page 43: `DATA INTERPRETATION REPLICA QUESTIONS THAT HAVE APPEARED IN CAT IN THE LAST 4 YEARS

5

50. Looking at the table and relating the diagonal elements, it is easy to see that the 2nd least traffic flow occurs through the road connecting Q – S. Choice (D)

51. From the above table, traffic flowing from city Q is the

greatest i.e., 3752 vehicles. Choice (B)

52. From the above table, the difference in traffic flow is the least for city T i.e., 3050 – 2952 = 98. Choice (C)

Solutions for questions 53 and 54:

53. Interest amount for Mr. A (in `) = 3,600 × 2.2 = 7,920 Interest amount for Mr. B (in `) = 3,800 × 3.6 = 13,680 ∴ The required difference (in `) = 13,680 – 7,920

= 5,760 Choice (B)

54. The required average = 2

1[5.6 × 4,800 + 6.4 × 4,000]

= 2

1[26,880 + 25,600]

= `26,240. Choice (C) Solutions for questions 55 and 56: 55. Let the total number of employees in company X,

company Y and company Z be x, y and z respectively. Male employees in company x who owns both four wheeler and two wheeler = 0.7x × 0.15 (Q 45 + 65 + 5 – 100 = 15) = 105x Female employees in company y who owns both four wheeler and two wheeler = 0.3x × 0.1 = 0.03x. ∴ The total number of employees in company X who owns both four wheeler and two wheeler = (0.105 + 0.03)x = 0.135x

percentage of employees = 13.5% Similarly for company Y, the required percentage is 26% Similarly for company Z, the required percentage is 6%. Choice (B)

56. Let the number of employees in either of the companies be ‘n’. The number of male employees in companies Y who satisfy

the given condition = 100

1 [100 – (30 + 20)]n = 0.5n

similarly the required number of employees in company

Z = 100

1[100 – (20 + 10)]n = 0.7n.

∴ the required percentage = 100n2

n7.0n5.0 ×+ = 60%.

Choice (D) Solutions for questions 57 and 58:

Model No. of Bikes Sold Average selling price (in `) 2007 2008

RL-100 19,500 40,000 45,000BCZ 37,500 25,000 28,000

Thunder 30,000 31,000 35,000WB-150 45,000 20,000 23,000Muzzle 18,000 52,000 55,000

57. From the above table, the percentage increase in the

average selling price is the highest for WB-150.

%15100000,20

000,20000,23=×

− Choice (C)

58. The required average (in `)

= 3105

5220312540 ×++++

3105

168×= = 33,600 Choice (B)

Solutions for questions 59 and 60: 59. The gain from the shares of company IV in 2006 was =

132 + 2

)]456(2432448[ −+

= 132 – 16 = 116 Choice (1) 60. We can only evaluate the return from the shares of

company III in the years 2002 to 2009. The returns were as follows:

Year 2002 2003 2004 2005 2006 2007 2008 2009Gain 122.5 158.5 155 150 170 148 172 195

The highest percentage increase is from 2002 to 2003 and it is 29.38% Choice (2)

Solutions for question 61: 61. To find the median, arrange the per capita incomes in

descending (or ascending) or order.

Per capita income ($) Country 24,369 Switzerland 24,337 Germany 23,484 United states 19,207 United kingdom 15,350 New Zealand 13,746 Swedes 13,477 France 11,692 Spain 10,372 Hong Kong 5,663 Brazil 4,965 Latherier 3,523 Mexico 2,916 Romaine

Out of the 13 countries, the median is the country placed 7th. France with a per -capita income of $13,477. 40% of 13,477 = $ 5390.8. There are 10 whose per capita income is more than $5390.8. Choice (2)

Solutions for question 62: 62. Given, intra-state services accounted for 60% of total

revenues.

∴ Total revenues = 60.0

2880= `4800 crore.

Total revenues from non A/C general category in intra-services is given to be 50% of revenues from intra-state services. ∴ Revenues from non A/C general category in intra-state services = 50% of 2880

= `1440 crore Choice (2) Solutions for question 63: 63. The total production of the top four coal producing

countries is 2536.7 + 1039.2 + 478.2 + 393.9 = 4448 mt The total production of the bottom four coal producing countries is 76.7 + 76.6 + 145.8 + 174.9 = 474 mt.

The required percentage = 4448474

× 100 = 10.66%

Choice (1)

Page 44: `DATA INTERPRETATION REPLICA QUESTIONS THAT HAVE APPEARED IN CAT IN THE LAST 4 YEARS

6

Solutions for questions 64 and 65: 64. Male students who were eligible for selection were A, F,

G and N and the female students who were eligible for selection were L and O Therefore the required ratio is 2 : 1 Choice (1)

65. x = 6 and y = 4

There fore 2x = 3y is in the correct choice. Choice (4) Solutions for question 66 and 67: The Total costs, Operating Expenses, Revenue and the Profitability of the company in the five years are given in the following table.

Year Total cost (in

`) Operating

Expense (in `)Revenue

(in `) Profitability

(in `) 2009 92200 24050 104200 0.2308 2008 83700 21775 96600 0.2254 2007 89600 22000 112400 0.1957 2006 96600 24730 128200 0.1929 2005 104000 26580 130600 0.2035 66. The profitability of the company was the least in the

year 2006 Choice (2) 67. With respect to the previous year in the years 2006,

2007 and 2008 were 7.11%, 7.24% and 6.58% respectively. In 2009 the total cost increased when compared to 2008. Therefore the maximum decrease was in 2007 and it was 7.24%. Choice (2)

Solutions for questions 68 and 69:

Sections Score less than 45

Score from 45 to 85

Score greater than

85

Total no of students

A 28 72 24 124 B 15 68 36 119 C 18 52 28 98 D 29 58 47 134 E 30 60 35 125

Total 120 310 170 600 68. Percentage of the total number of students getting

scores less than 45 = 600120

× 100 = 20% Choice (4)

69. For the sections A, B, C, D and E, the maximum

number of students getting 48 or more in the examination was 96, 104, 80, 105 and 95 respectively. Thus the highest among the above values is 105. Therefore the maximum number of students from a section who passed in the examination was 105.

Choice (3) Solutions for questions 70 to 72: 70. The growth in exports of the 4 companies from 2003 -

04 to 2004 - 05 are as follows..

Rahual & co: 10015.1215.3 × = 25.9%

Chandu & co: 1001.141.2 × = 14.9%

Shiva & co: 1003.98.2 × = 30.1%

Kanta & co: 10041.10

32.3 × = 31.9%

Hence Kanta & co has the highest growth in exports. Choice (4)

71. The growth rate in imports of the 4 companies from 2002 – 03 to 2003 – 04 are as follows:

Rahual & co: 10014.553.6 × = 127%

Chandu & co: 10061.117.4 × = 40.5%

Shiva & co: 10072.867.0 × = 7.7%

Kanta & co: 10046.705.4 × = 54.3%

Hence the growth rate of imports is the least for Shiva & co. Choice (3)

72. Trade deficit = imports – exports

The trade deficit of the 4 companies in 2004 – 05 are as follows Rahul & co: 2.11 Shiva & co: 0.9 Chandu & co: 1.07 Kanta & co: 0.6 Rahul and co has the highest trade deficit in 2004 - 2005 Choice (1)

Solutions for questions 73 and 74: 73. The number of employees who did not cross the cut off

for all the 5 companies are as follows.

No. of employees who did not cross the cut off.A 120 B 225 C 100 D 200 E 275

Hence E has rejected the maximum number of employees. Choice (4)

74. The number of employees who got more than 90% for

the 5 companies are as follows:

Greater than 90% marks Cut off cleared

A 30 180 B 36 225 C 30 150 D 96 400 E 115 300

Total 275 1255

The required percentage is 1255307

= 24.5% ≈ 24%

Choice (3) 75. The sales of a company = The no .of units produced –

the closing stock. The sales of all the 5 companies in 2009 are as follows

Company Sales

P 10515 Q 14310 R 9225 S 7755 T 11135

Hence S had the least sales in 2009. Choice (2)

76. The sales of R in 2008 = 9000 – 675 = 8325

The sales of R in 2009 = 10000 – 775 = 9225 ∴R had lower sales in 2008. Choice (1)

Page 45: `DATA INTERPRETATION REPLICA QUESTIONS THAT HAVE APPEARED IN CAT IN THE LAST 4 YEARS

7

Solutions for question 77 and 78: 77.

Family: Total income

(in `)

Expenses + Overhead

(In `)

Savings (in `)

Kapoor Khanna Kirsten Kumble Khan Kittu Kala

147000 105000 168000 140000 165000 120000 196000

12000 13500 17750 16750 19000 17450 21375

135000 91500

150250 123250 146000 102550 174625

923175

The total savings made by all the families was `923175 Choice (2) 78. The increase in income of the Khan family is `3300

The decrease in expenses is `570 ∴The increase in saving is 3300 + 570 = `3870

Choice (3) Solution for question 79: 79. The healthy drinks are S are X

The other drinks are unhealthy. Hence the required ratio is 2 : 8 = 1 : 4 Choice (3)

Solutions for 80 and 81: 80. The difference in the number of students studying in

government schools in all the states in 2008 are as follows.

Difference

AP 1800 MP –1600 UP 1300 Karnataka –1400 Kerala 2200 Tamil Nadu 5200

The maximum increase is for Tamil Nadu, Choice (4)

81. We can observe that the state which has consistent

increase in the number of students from 2007 to 2009 is UP. Choice (2)

Solutions for questions 82 and 83: 82. The median of the total number of students is

2

1718 +=17.5

Hence A, C and F have more number of students than the median. Choice (2)

83. The number of failed students in each section is given

the table. Hence the number of failed students is the highest more in section C

Section No. of students failed

A 6 B 8 C 12 D 7 E 3 F 10

Choice (3)

Solutions for questions 84 and 85: 84. The yield return of R in the years are as follows:

Yield Return 2003 1557.14 2004 1574.1 2005 1884.6 2006 2021.7 2007 1595.2 2008 1525

Hence the highest yield return is in 2021

Choice (1) 85. The yield return for Q in the years are as follows

2003 Yield Return 1676.5

2004 2204.5 2005 2633.3 2006 2000 2007 2720 2008 4105.3

Hence the highest percentage increase is in 2008

Choice (4) Solutions for questions 86 and 87: 86. From the table we can easily observe that the average

marks are the highest for ΙΙ, ΙX and X. Hence these classes would satisfy the statement "the higher the average marks, the higher are the number of students". Choice (3)

87. The statement “the lower the number of students, the

higher the average marks” can be verified through the options. Classes Ι and ΙΙ have higher number of students, hence they do not satisfy the statement. ∴The correct choice is (A) Choice (1)

Solutions for question 88: 88. The number of students this year in the 6 states

Number of students AP 13,21,000 UP 17,46,000 MP 13,90,000

Bihar 19,14,000 Assam 12,88,000 Orissa 10,88,000

Hence MP has the third highest number of students

this year Choice (2) Solutions for question 89: 89. Given Lakshmi spends 20% of the revenue earned

from each investment to maintain her house. So let us calculate the revenue for each business in which she invested.

X Y Z

Investment 16.2 14.5 12.9 Revenue 16.96 14.72 13.2

Hence the maximum profit is obtained from X Choice (1)

Page 46: `DATA INTERPRETATION REPLICA QUESTIONS THAT HAVE APPEARED IN CAT IN THE LAST 4 YEARS

8

Solutions for questions 90 and 91: 90. The income and expenditure for the four regions in

2007 are as follows.

Income Expenditure Ratio

North 33.8 34.5 0.98

South 33.8 35.2 0.96

East 31.9 32.7 0.975

West 40.1 41.3 0.971

Hence the required ratio is the highest for the North region. Choice (1)

91. The states in which the per capita income increased by

more than 5% are J and K, West Bengal, Gujarat and Maharashtra In the remaining states the per capita income did not increase by more than 5%

Hence the required ratio is 1 : 1 Choice (2)

BAR GRAPH Solutions for questions 1 to 4: 1. The percentage growth rate in 2007 over 2006

= %5.31100190

190250 =×−

Had the percentage growth from 2007 to 2008 been 31.5%, the estimated revenue would have been

250 × 329100

5.131 =

The required difference 329 – 305 = 25 (approximately) Choice (1)

2. Let the number of people who used the company's

products in Asia in 2003 be 100. The number of men and women who used the product

in the different years are

Year 2003 2004 2005 2006 2007 2008 2009 2010Men 60 63 66.15 69.5 73 76.65 80.5 84.5

Women 40 44 48.4 53.25 58.5 64.5 71 78 Total 100 162.5

∴ the approximate percentage growth = 62 Choice (1) 3. The percentage change in the gap between the revenues

from the US and Asia in the different years are Year 2003 2004 2005 2006 2007 2008 2009 2010 Gap in million USD 150 170 150 140 110 85 60 50

Absolute percentage change

13 12 6 21 22 30 17

The absolute value of the percentage change in the

growth rate was the highest in 2008-09. Choice (4) 4. The growth rate in 2005 (over 2004)

= %5010090

90135 =×−

The growth rate in 2007 (over 2006)

= %32190

190250 =−

The required percentage

= 3510050

3250 =×−(approximately) Choice (3)

Solutions for questions 5 and 6:

5. Number of pythons in the world = 40

100 × 4800 = 12,000

Number of bears in the world = 42

100 × 4200 = 10,000

The number of deers and wild bisons in South America are 6,000 (25% of 24,000) and 5,400 (30% of 18,000) respectively. ∴ Number of wolves in South America = 25,800 – (4,800 + 6,000 + 5,400 + 4,200) = 5,400 ∴ total number of wolves in the Amazon forest = 75% of 5,400 = 4,050. Choice (B)

6.

Species Number in Amazon Forest Pythons 80% (4,800) = 3,840 Deers 70% (6,000) = 4,200 Wild Bison’s 80% (5,400) = 4,320 Wolves 75% (5,400) = 4,050 Bears 95% (4,200) = 3,990

Choice (B) Solutions for questions 7 to 9: 7. The percentage increase is maximum in case of

company R i.e., %5.201004480

44805400 ≈×−

Choice (C) 8. Since the cost of PC is same for all the companies

market share of Q in 2009

= 4500500042005600

4200+++

≈ 21.76%

Market share of Q in 2014

=)50042005600(%110

)4200(%110++

× 100 = 28.37%

∴ the difference in percentage points = 28.37 – 21.76 = 6.61. Choice (A)

9. Looking at the options it is enough if we check for the

market share of S in 2006 and 2007. Let the price per PC of A, B, C and D `x, `2x, `x and `2x respectively. Market share of S in 2006

= 100)25800448230004200

58002 ××++×+

×

= %441001168.446042

116≈×

+++

Market share of S in 2007

= 1002545.482285.43

254 ××++×+

× = 100

256

108×

= 42.2% Choice (A) 10. Let us consider the selling prices of the four models in

2004 as 3k, 4k, 5k and 6k respectively. Selling prices in the years.

Models 2004 2005 2006 P 3k 4.5k 6k Q 4k 6kl 8k R 5k 7.5k 10k S 6k 9k 12k

Page 47: `DATA INTERPRETATION REPLICA QUESTIONS THAT HAVE APPEARED IN CAT IN THE LAST 4 YEARS

9

Sales revenue of Q in 2004 = 750 (4k) = 3000k Sales revenue of R in 2006 = 500(10k) = 5000k

Therefore the sales revenue of Q in 2004 by

100k3000

k3000k5000×

−= 66

3

2% Choice (3)

Solutions for questions 11 to 13:

Car 2007 2008 2009

Production Sales Production Sales Production Sales

Alto 13000 8000 15000 10000 14000 9000

Swift 22000 20000 21000 18000 25000 22000

Estio 20000 18000 21000 20000 22000 16000

11. The total production of all three Cars in

2007 = 55000 2008 = 57000 2009 = 61000 The total sales of all three Cars in 2007 = 46000 2008 = 48000 2009 = 47000 The required ratio in 2007 = 1.19 2008 = 1.18 2009 = 1.29 Hence the ratio is the highest in 2009 Choice (3)

12. The ratio of production to sales of Alto in

2007 = 1.625 2008 = 1.5 2009 = 1.55 The required ratio is the highest in 2007

Choice (1) 13. The exports of Swift in

2007 = 2000 2008 = 3000 2009 = 3000 The ratio of exports to sales in 2007 = 0.1 2008 = 0.17 2009 = 0.14 The required ratio is the highest in 2008

Choice (2)

PIE CHART Solutions for questions 1 to 6: Section Exam QA LR VA RC DI Total

AIMCAT 1 51 69 60 45 75 300 AIMCAT 2 105 30.8 35 56 53.2 280 AIMCAT 3 120 45 60 54 81 360 AIMCAT 4 80 64 32 64 80 320 AIMCAT 5 84 42 91 70 63 350 QA LR VA RC DI Maximum actual score 100 80 50 60 100

1. The required percentages = %12010050

60=×

Choice (D) 2. Maximum possible ‘actual score’ in RC section = 60.

The least difference occurs in the AIMCATs 2 and 4 i.e., 60 – 56 = 64 – 60 = 4 Choice (A)

3. From the above tables, the given condition is satisfied

in AIMCAT 3 and AIMCAT 5. Choice (B)

4. The student showed the highest percentage increase in

QA section i.e., %7.6410051

5184 =×− Choice (B)

5.

Section Percentage change

QA 34

3456 −× 100 > 50%

LR 2.55

6.332.55 −× 100 ≃ 40%

VA 24

244.36 −× 100 > 50%

RC 36

3656 −× 100 > 50%

DI 5.62

4.525.62 −× 100 ≃ 16%

Choice (D) 6. The marks obtained by the student in the RC section is

the highest in AIMCAT 5 Choice (B) Solutions for question 7: 7. Given the total number of students = 1000

Dancing = 45% = 450 Embroidery Classes = 5% = 50 Singing = 20% = 200 Karate = 15% = 150 Painting = 15% = 150 Now only Boys chose Karate. Hence a total 150 students in Karate are all boys. Only girls chose Embroidery classes. Hence a total 50 students in Embroidery are all girls. Also 80% of students in Singing are girls. Hence 160 students in Singing are girls and 40 are boys. Similarly 80% of students in Dancing are boys. Hence 360 are boys and 90 are girls, who are in dancing In Painting the ratio of boys to girls is 1 : 1 Let us tabulate the data.

Boys Girls Total

Dancing 360 90 450 Singing 40 160 200 Painting 75 75 150 Karate 150 0 150

Embroidery Class 0 50 50 Total 625 375 1000

If Painting & Singing are mixed then the ratio of boys to girls is 115 : 235 = 23 : 47 Choice (3)

Page 48: `DATA INTERPRETATION REPLICA QUESTIONS THAT HAVE APPEARED IN CAT IN THE LAST 4 YEARS

10

Solutions for question 8: 8. Given the ratio of the number of employees in central to

state government jobs is 6 : 1. Let the central government jobs be 600 and the state government jobs be 100. The number of central government employees in A.P = 150. The number of state government employees in Kerala = 25.

∴The required ratio is 150 : 25 = 6 : 1 Choice (1) Solutions for 9 and 10: 9. The number of employees in each department of P

and Q.

P Q HR 1750 3780

Academic 10500 3060 Operations 5250 11160

Total number of employees in both companies in the HR department = 5530

Hence the required percentage = 355005530

= 15.5%

Choice (2) 10. The number of Academic employees = 13560

The number of Management department employees = 21940

The required ratio = 2194013560

= 0.618 ≈ 0.62

Choice (2)

Pie Charts + Bar Charts

Solutions for questions 1 and 2: Number of students in each discipline is as follows: Number of students

Discipline Number of students Marketing 3780 Finance 1260 Operations 1008 Systems 1512 HR 252

Number of males and females in each discipline are as follows:

Males Females Difference Marketing 2079 1701 378 Finance 819 441 378

HR 1008 1512 504 Systems 840 672 168

Operations 576 672 144 Total 5322 4758

1. The total number of female students in the institute was

less than the total number of male students by

532247585322− × 100 = 10.6% Choice (3)

2. The difference between the number of male and female

students was the highest for HR. Choice (1)

LINE GRAPH Solutions for questions 1 and 2: 1. Profit on a normal day = 7000 – 6500 = `500

Profit when 300 units are sold = 10,500 – 9000 = `1500

Required percentage = %200100500

5001500 =×−

Choice (C) 2. Cost when 200 units are produced = `6500

Cost when 350 units are produced ≃ `10,000

Additional cost /unit = 1503500

1506500000,10 =−

= `23.

Choice (B) Solutions for questions 3 and 4: The energy consumption of Geyser in a week is 7 kWh and we know the family uses the Geyser for 2 hrs in a day. Hence for 14 hrs in a week the energy consumption is 7 kWh. Hence the energy consumption of a Geyser per day is 1 kWh. Now, energy consumption of Refrigerator in a week is 14 kWh and the family uses Refrigerator throughout the day. Hence, the energy consumption of Refrigeration per day is 2 kWh. Similarly the energy consumption for TV in a day is 2 kWh. The energy consumption for Washing machine in a day is

74

kWh and for Grinder is 71

kWh

3. (a) Energy consumed by TV for 3 days = 6 kWh.

Energy consumed by Refrigerator for 3 days = 6 kWh. Hence Choice (1) is false.

(b) Energy consumed by Geyser for 4 days = 4 kWh. Energy consumed by Grinder for 7 days = 1 kWh.

Hence Choice (2) is false. (c) Energy consumed by Washing Machine in a week

= 4 kWh. Energy consumed by Geyser for 2 weeks = 14 kWh. Hence Choice (3) is true.

(d) Energy consumed by TV for 2 days = 4 kWh. Energy consumed by Washing machine for one week = 4 kWh Hence (d) is false Choice (3)

4. The fixed cost increased by 25 %. Hence the new fixed

cost is `60 + 41

(60) = `75.

Hence the increment in the total cost is

100)30

74

35.0(60

15 ×××+

= 10012015 × = 12.5%

Choice (2)

DATA SUFFICIENCY Solutions for questions 1 to 4: 1. From A, as 60% of the newly joined employees were

not managers, the remaining 40% of the newly joined employees were managers. It is given that 10 managers had newly joined. ∴ 40% = 10 ⇒ 100% = 25 Hence, A alone is sufficient. B gives no data, it is just an assumption. Choice (1)

2. From A and the given condition, either Babu or David

got the highest rank. Hence, A alone is not sufficient.

Page 49: `DATA INTERPRETATION REPLICA QUESTIONS THAT HAVE APPEARED IN CAT IN THE LAST 4 YEARS

11

From B and the given condition, either David or Amar can be the highest ranker. Hence, B alone is not sufficient. Combining A and B, David must get the highest rank. Choice (4)

3. It is given that, 30% of the students are boys, which

implies 70% of the students are girls. Also 10% of the girls are athletes. ⇒ 10% (70%) = 7% of the students are female athletes.

From A, 25% of the students are athletes Hence 25 − 7 = 18% of the students are boys who are athletes. So, A alone is sufficient. From B,

Number of boys who are athletes = 120% of the girls who are athletes. As 7% of the students are girls who are athletes, 120% (7%) = 8.4% of the students are boys who are athletes. So, B alone is also sufficient. Choice (3)

4. Clearly, A alone is not sufficient, as we do not know

how many points the opponent scored. B alone is also not sufficient, as we do not know how many points team A scored. Combing A and B, If the score at the half time was say 0-25, then the match would have ended in a tie at 35-35. So, team A did not win. Had the score at half time been, say, 10-35, then in the end it would have been 45-35 and team A would have won. So, we cannot answer the question even after combining both the statements. Choice (5)

PPL

1. Let the cost of the new car be `x.

Therefore the cost of the old car = 40% (x) =52

x.

From statement Ι, we cannot answer the question as neither the cost of the new car nor the cost of the old car is given. Statement Ι alone is not sufficient. From statement ΙΙ, we only know regarding his personal saving but nothing about the cost of the cars. Statement ΙΙ alone is not sufficient. By combining both the statements, we have the following information.

Amount borrowed from his friend = 60%

x

5

2

=

x

5

2

5

3=

25

6x

Money realised by selling the old car =5

2x

Money withdrawn from personal savings account to meet the cost of the new car.

= x –25

6x –

5

2x=

25

9x

Now, it is not known that 25

9x was what portion of his

personal savings balance. Thus the question cannot be answered even by combining both the statements. Choice (4)

(Numbers)

1. From statement A,

xy = 18 the different possibilities are: 1×18 2 × 9 3 × 6

Again yz = 21 the different possibilities are 1 × 21 3 × 7

But we do not know if x , y, z are natural numbers or not. For eg if y = 9, we can get.

xy = 18 as 2×9 and yz = 21 as 9 × 37

There will be infinite possibilities like this, so statement A is not sufficient. Statement B above is also not sufficient as it gives information regarding x and z only and nothing about y. Combining both the statements, we can conclude that x = 6, y = 3 and z = 7. Thus z is the maximum.

2. Using statement A alone we have SEVEN = 19 and FIVE = 14, without knowing anything about the values of individual alphabets, we cannot answer the question. Using both the statements together we can conclude that 2 (N) + I + E = 7 ⇒ N = 1 and I and E are 2and 3 or 3 and 2 So F + V = 14 – 5 9. F and V could be 4, 5 or 5,4. SEVEN = 19 If E = 2 and V = 5, we get S = 9 whereas if E – 3 and V = 5, we get S = 7. So we cannot determine S uniquely. Thus the question cannot be answered even by combining both the statements.

3. The number of days that Raju's dad goes to the Shiva

temple in a year is

3365

= 121 days.

The number of days that Raju's dad goes to the

Venkateshwara temple is

4365

= 91 days.

The number of days that Raju's dad goes to the

Saibaba temple is

7365

= 52 days.

The number of days that Raju's dad goes to both the

Shiva and the Venkateshwara temple is

12365

= 30 days. The number of days that Raju's dad goes to both the

Venkateshwara and the Saibaba temple is

28365

= 13 days. The number of days that Raju's dad goes to both the

Shiva and the Saibaba temple is

21365

= 17 days.

The number of days he goes to all the three temples is

84365

= 4 days.

Hence the number of days Raju's dad goes to exactly one temple is 121 + 91 + 52 – 30 – 13 – 17 – 4 = 204 days. Choice (1)

4. A + B +C + C + D + E +E +F + G = A + B + C + D + E

+F +G + C + E (1 + 2 + 3 + 4 + 5 + 6 + 7) + C +E

Now 28 + C +E = 33. ∴C + E = 5

C, E could be (1, 4), (4, 1) (2, 3) OR (3, 2)there are four possible ordered pairs of (C, E). Choice (3)

Page 50: `DATA INTERPRETATION REPLICA QUESTIONS THAT HAVE APPEARED IN CAT IN THE LAST 4 YEARS

12

Yellow C

Blue B

Indigo E

D Violet

F Green

A Orange

CASELET Solutions for questions 1 to 3: The arrangement of the buildings according to the given conditions is Height 1. E 2/3. B/D 4. A 5. C 6. F 1. The colour of the building diagonally opposite to the

yellow coloured building is Violet. Choice (4) 2. The second tallest building is either B or D. Choice (5) 3. The colour of the tallest building is Indigo.

Choice (2) Solutions for questions 4 to 7: Stage Ι As P, Q, S and T won at least one match, R and U lost all the three matches. As Q, S and T lost at least one match, P won all the three matches. In stage-Ι, there are a total of 9 matches and so 9 wins. Q, S and T won two matches each. As P (the top team in stage-Ι) did not play against U, P played matches against Q and R. ∴The ninth match was between Q and U. So the nine matches that have taken place are as follows.

Won Lost Won Lost Won Lost P S S R S U Q T T R T U P Q P R Q U

Stage-ΙΙ As each team played a total of five matches, in stage ΙΙ, the matches take place between the following pairs of teams. P – T, P – U, Q – R, Q – S, T – S and R – U Given that, in stage-ΙΙ, three teams lost all the two matches. Given P lost both the matches in stage-ΙΙ ∴Each of T and U won the two matches. ⇒ R and S lost the two matches. ∴Q also won two matches. 4. T and U defeated P (the top team in stage-Ι) Choice (2) 5. Only Q, T and U won both their matches in stage-ΙΙ. Choice (4) 6. S and U won exactly two matches in the event. Choice (5) 7. Q and T won exactly four matches each in the event.

Choice (5)

Solutions for questions 8 to 12: The trading pattern followed by each of the three traders is as follows

Anand Bala Chandu Buy Sell Buy Sell Buy Sell

10 a.m. 3 p.m. 10 a.m., 3 p.m. 10 a.m., 3 p.m.

11 a.m., 11 a.m.,

12 noon, 12 noon,

1 p.m., 2 p.m. 1 p.m.,

2 p.m.

8. As the direction of the price movement is not known,

the profits of Bala and Chandu depends upon the prices at which they bought gold i.e., if they buy at lesser price than that bought by Anand, their profits would be more, if not, the profits of Anand would be more than that of the other two. Hence the answer cannot be determined. Choice (5)

9. Anand buys the entire quantity at a single point of time,

whereas each of the other persons buy once every hour. As the direction of movement of gold is not given, we cannot compare the returns of Anand with the other two persons.

Bala: Bala buys the same quantity of gold every time, irrespective of the price.

Chandu: Chandu spends the same amount every time, his buying depends on the price of gold at the time he buys. The more the price, the lesser quantity he buys. As his strategy is based on prices, whenever the prices are changing, Chandu’s returns will be more than that of Bala. But if there is no change in the price of gold the returns of Bala and Chandu would be equal. Hence no conclusion can be made. Choice (5)

10. On a boom day, the price of gold keeps rising, hence it

will be the least in the morning. Hence, Anand who bought all his holdings in the morning will get the maximum profit. Between the remaining two, Bala bought the same quantity at every time, i.e he bought the same quantity even at higher prices whereas Chandu spent the same amount. Hence, Chandu bought less quantity of gold when prices were high and more when prices were less. Hence, Chandu’s returns are more than that of Bala's. Bala will have the least returns. Choice (1)

Let the prices of gold at different timings be as follows.

Time 10 a.m. 11 a.m. 12 noon 1 p.m. 2 p.m. 3 p.m. Price a b c d e f

We will look at the additional information given: The quantity bought by Anand at 10 a.m. is the same as the quantity he sold at 3 p.m. As it is given that Anand lost money, we can ignore the quantity bought/sold and can conclude that the price at 3 p.m. must be less than that at 10 a.m. ⇒ a > f → (Ι) Similarly the quantity of gold bought/sold by Emma in each instance is the same and it is given that Emma made a profit. Hence we can conclude that (c + f) > (a + d) → (ΙΙ) Also using similar logic in case of David, we conclude that (d + e + f) > (a + b + c) → (ΙΙΙ) It is given that the price increased from 2 p.m. to 3 p.m. ⇒ e < f → (ΙV) It is given that price at 12 noon was lower than the opening price ⇒ c < a → (V) From (i) and (ΙΙ) we can conclude that c > d → (VΙ) From (Ι), (ΙΙΙ) and (VΙ) we conclude that e > b → (VΙΙ) Hence a > f > e > b and a > c > d ⇒ a is the highest.

Page 51: `DATA INTERPRETATION REPLICA QUESTIONS THAT HAVE APPEARED IN CAT IN THE LAST 4 YEARS

13

b = 3k

f = k

a = 4k

d = 2k

e = k

c = k

HR

n

Finance Marketing

g=3k

11. The price of gold was the highest at 10 a.m. Choice (1) 12. As d < c, choice (4) is also necessarily false. Choice (4) Solutions for questions 13 to 15: 13. The different possibilities in which they could have

booked the rooms are as follows. Case Ι:

101 102 103 104 A C D B

Case ΙΙ:

101 102 103 104 B D C A

Since B booked an odd numbered room, we can conclude that as per case ΙΙ, B must have booked room number 101, in which case C would have booked room number 103. Choice (C)

14. It is given that two girls failed in the examination. Now

we have six possibilities in which we can select the two girls who failed. They are as follows:

Cases ↓ Dolly Molly Polly Kelly

1 Pass Pass Fail Fail 2 Pass Fail Pass Fail 3 Pass Fail Fail Pass 4 Fail Pass Pass Fail 5 Fail Pass Fail Pass 6 Fail Fail Pass Pass

Let us denote a true statement by T and a false statement by L (lie)

Cases→ 1 2 3 4 5 6

Dolly T T T L L L Molly L L L L T L Polly T T L L T T Kelly T L T T L T

As exactly three of them were telling the truth, only in case Ι it is so. Thus Molly was the person who was lying. Choice (B)

15.

It is given that n = O g = 37.5% of (a + b + c)

i.e., g = 8

3(a + b + c)

Let (a + b + c) be 8k. ∴ g = 3k Students who did not opt for Finance = b + f + c

Students who opted for Finance = a + e + d + g. (a + e + d + g) 50% = b + f + c. → (1) Students who did not opt for HR = a + e + b.

a + e + d + g = 4

5(a + e + b). → (2)

Again number of students who opted for only Finance and Marketing was 331/3% of those who opted for all three Number of students who opted for only Marketing and HR = Number of students who opted for only Finance and Marketing.

∴ f = e = 3

1g.

∴ f = e = k. From equation (1), a + e + d + g = 2 (b+ f + c)

a + k + d + 3k = 2 (b + c) + 2k a + d = 2(8k – a) – 2k. 3a + d = 14k → (3)

From equation (2), we get, 4d + 11k = 5b + a → (4) Again number of students who opted for only Finance & HR, i.e., d was 50% of those who opted for only Finance i.e., a. ⇒ d = 50% a. Substituting in equation (3), we get d = 2k and a = 4k. Substituting in equation (4) we get b = 3k. Now (a + b + c + d + e + f + g) = 15k 15k = 270 ⇒ k = 18 ∴Exactly two = d + e + f = 4k = 4 (18) = 72 students. Choice (B)

Solutions for questions 16 and 17: It is given that books D and F was read by the same person, A and B was not read by the same person and F and C was not read by the same person. The different combinations in which the books were read are as follows:

Ι ΙΙ ΙΙΙ ΙV A B A B A B A B D C D C D C C D F G F E F E G F E H G H H G H E

V VΙ

A B A B C D C D E F E F H G G H

16. As Akira read books E and G, the books that Akira read

could be either A, C, E and G or B, C, E and G. In either case, we can conclude that Aroki did not read book C. Choice (D)

17. As books C and E, were not read by the same person,

as in cases Ι and ΙV, books G and H were read by the same person. Choice (C)

Solutions for questions 18 to 25: 18. W5 and W7 are allotted a shift, one earlier than W6 and

W3 and W9 are also allotted a shift earlier than W6. Again as W3 is allotted a shift lower than W2, if we allot the afternoon shift for W3 and W9; W5 and W7 being one shift earlier than W6, we will have four workers in the Afternoon shift if W6 is allotted the Evening shift. Thus the only shift that can be alloted to W6 is the Night shift. The following table gives the workers and the shift they were allotted to.

Page 52: `DATA INTERPRETATION REPLICA QUESTIONS THAT HAVE APPEARED IN CAT IN THE LAST 4 YEARS

14

Morning W2

Afternoon W1, W3, W9

Evening W5, W7

Night W4, W6

Thus W8 can be allotted any shift other than the afternoon shift. Choice (B)

19. The following table lists down the matches and the

corresponding players who led the team as captain and vice captain.

Match Captain Vice Captain Match 1 B A/D Match 2 A C Match 3 B A/D Match 4 C A/B/D Match 5 D A/B/C

As D refused to lead the team as captain if A or B led the team as captain in the preceding match, we can conclude that D can be the captain of the team only in Match 5.

Again with D as Captain in Match 5, A must have captained the side in Match 2 for A cannot be the captain in Match 4.

Now with A as the captain in Match 2, C must have been the vice captain in that match.

Thus C was the vice captain in Match 2. Choice (C)

20. For B:

Person Appliances Day Water Purifier Monday Refrigerator Tuesday B

AC Wednesday

For A or C: AC must be bought before the water purifier.

Choice (B) 21. Number of matches played by Sachin is equal to that

played by Mongia. Number of matches played by Dravid is equal to that played by Hussey. Since Sachin has played more matches than Dravid, the average runs must be less than 45. Choice (A)

22. Average (Ramesh) = .28.407

282=

Average (Sanjay) = .437

301=

If the average score after the exclusion lies between 40.28 and 43, then the average of Ramesh will decrease while that of Sanjay will increase. Since, 92 is the only value lying in that range, so their score in the invalid question is 42. Choice (A)

23. A wins ⇒ B wins B wins ⇒ C does not win. That implies both A and C do not win together. That means at most one of A or C wins. That further

implies that D must win. Choice (A) 24. Case (1) Day 1 / 2 / 3 Schumi Mclaren Sebastian Schumi Mclaren Sebastian Sebastian Schumi Mclaren Case (2) Day 1 / 2 / 3 Schumi Sebastian Mclaren Schumi Mclaren Sebastian Sebasian Schumi Mclaren

Case (3) Day 1 / 2 / 3 Schumi Mclaren Sebastian Sebastian Schumi Mclaren Sebastian Schumi Mclaren If Schumi beats Mclaren on all the three days, then

Mclaren will come last all the three days (not possible). Choice (D)

25. (i) Both Sashi and Govind work together. This implies

Ryan and Mokambo will work together Choice (A) Solutions for questions 26 to 28: 26. Let the runs scored by Bhajji be x

Straight drive Pull shot Others Total

Pollard 5

40x + x + 40

Dumminy (0.6) (x+ 20) (0.15) (2x + 40) x + 20

Bhajji 204

x= x

Given 204x =

⇒ x = 80.

Straight drive Pull shot Others Total Pollard 24 120

Dumminy 60 10 30 100 Bhajji 20 80

Maximum possible difference = 95 – 10 = 85

Choice (D) 27. Maximum runs scored by Bhajji through straight drive = 59.

∴ Required percentage = 100300

59× = 192/3%

Choice (D) 28. Runs scored by Bhajji through ‘others’ cannot be

determined. Choice (D) Solutions for questions 29 to 41: 29. (1) All shoes are pens. (2) Not all pens are pencils. (3) All pens are chocolates. (4) Not all chocolates are pens. Analyzing the options: (A) Combining (1) and (4), we get “Some chocolates

are not shoes”. (B) Combining (1) and (3), we get that ‘some shoes

are chocolates’. Choice (D) 30.

→ t • • Ramesh’s house Umesh’s house ←

t54

Total time = t + t59

t54 =

( )445:7t

59 −= pm = 225 min.

t = 125 min

Page 53: `DATA INTERPRETATION REPLICA QUESTIONS THAT HAVE APPEARED IN CAT IN THE LAST 4 YEARS

15

T2 T1

x 300 – x 200 – x

y

Refrigerators Air Conditioners

LCD TVs

b

d 20 24

26

a c

When Ramesh reaches Umesh’s house, his watch was showing 4 p.m. + 125 mins = 6:05 p.m.

Umesh’s watch was showing 6:10 p.m. So, Ramesh’s watch is 5 mins slower than Umesh’s watch. Choice (D)

31.

yx200yx300

yx

+−+−=

Let 2y

x =

⇒ 400 – 2y = 300 – y y = 100

If 2y

x >

then y > 100 and x > 200. This is not possible so only one value exists Choice (A)

32.

Swimming Running Cycling Walking TotalW 5 6 15 X 1 6 Y 5 2 4 14 Z 1 18

In case of Z: A sum total of 18 is possible when two ‘6s’ and one ‘5’

is there in addition to ‘1’.

Swimming Running Cycling Walking TotalW 2 5 2 6 15 X 1 6 Y 5 2 4 3 14 Z 6 6 1 5 18

The above table gives the ranks obtained by the four

persons in the four events. In case of X, a sum total of 6 is possible only if

1 + (1 + 3 + 1) is there. He has to get rank 3 in cycling and rank 1 in each of running and walking.

Choice (A) 33.

Name Subjects

Adam Ben Cathy Dimitry Emmanuel

Mathematics 1 4 3 2 5 Physics 2 5 1 4 3 Chemistry 3/5 4 5/3 1 2 Biology 5/3 2 3/5 1 4 Total 11 15 12 8 14

Since the sum of the ranks of Dimitry was 8 and he got

the same rank in Chemistry and Biology, his ranks in Chemistry and Biology was 1.

Therefore Dimitry’s rank in Mathematics was 2 which was the same as Emmanuel’s rank in Chemistry.

Emmanuel’s rank in Physics must have been 3. Ben got the same rank in Mathematics and Chemistry. Remaining ranks of Ben is be 4, 4 and 12. Therefore

Ben or Emmanuel did not get the 1st rank in Mathematics. Thus Adam’s rank in Mathematics was 1. Cathy got 3rd rank in Mathematics, therefore Ben got the 4th rank in Mathematics.

Proceeding like this we can conclude that Cathy got the 1st rank and Adam got the 2nd rank in Physics Choice (B)

34. It is given that at least 40 families own both a

Refrigerator as well as a Air conditioner. ∴ b + d is at least 40. We have to find the maximum value of a. a will be maximum when (b + d) is minimum. i.e., when (b + d) is 40. Now a+ c+ (b + d) + 24 + 20 + 26 = 120. a + c = 120 – 70 – (b + d) a + c = 10. ∴ The maximum value of a is 10, when c is 0. Thus at most 10 families owes a refrigerator and a LCD

TV but not an Air Conditioner Choice (A) 35. The lectures and the days on which they deliver the

lectures are tabulated in the following figure. Monday Tuesday Wednesday Thursday

Lecturers L1 L4 L2 L1 L3 L4 can deliver the lecture either on Tuesday or on

Wednesday. Now, if L4 delivers his lecture on Wednesday, then L3

cannot deliver his lecture on any of the given days. [Since L3 delivers a lecture only if L2 delivered a lecture

on the preceding day and L3 and L4 do not deliver lectures on consecutive days.]

Thus L4 delivers the lecture on Tuesday. Now, the only day on which L3 could have delivered the

lecture was Thursday Choice (D) 36. The different ways in which the committee can be

formed is as follows: 1. B2 B4 B5 G2 G3 2. B1 B4 B5 G1 G3 3. B1 B4 B5 G2 G3 4. B1 B3 B5 G1 G3 5. B1 B3 B5 G2 G3 6. B2 B3 B5 G2 G3 Therefore there are six ways in which the committee

can be formed Choice (D) 37. In order to have the total machining time as minimum,

none of the machines must be idle at anytime and the total time taken must be 10 hours. (i.e., higher of the total machining times in the two machines).

Let us consider the answer options and check if it is possible.

Option A

Page 54: `DATA INTERPRETATION REPLICA QUESTIONS THAT HAVE APPEARED IN CAT IN THE LAST 4 YEARS

16

Hockey

e

g

b a

e

d f

Cricket Football

M1 M2

Duration

2 P3 P1

3 P1 P2

5 P2 (4) P3 (5)

Total = 10

Option B

M1 M2

Duration

3 P1 P2

2 P3 P1

5 P2 (4) P3 (5)

Total = 10

In case of option (C), if product P2 is machined in M1

before Product P1, since P2 takes 4 hours in M1, it can be done as follows:

Duration 0 – 4 4 – 7 7 – 9 M1 P2 P1 P3 Duration 0 – 5 5 – 7 7 – 10 M2 P3 P2 P1 The total time taken is again 10 hours Choice (D) 38. Those playing exactly 3 games = g Those playing exactly 1 game = a + b + c Those playing exactly 2 games = d + e + f Those playing at least 2 games = d + e + f + g It is given that d + e + f + g = 18 → (1) and (a + b + c) + (d + e + f + g) = 30 → (2). Therefore a + b + c = 12. Now (a + b + c) = 3 (g). or, 3g = 12 or, g = 4. Therefore d + e + f = 18 – 4 = 14. Thus the number of members playing exactly two

games is 14 Choice (B) 39. We can list down the names of the countries and the

athletes belonging to them as follows.

Countries UK Germany France Switzerland Turkey 1. M6 M7 2. M8

It is given that M6 and M8 belonged to UK where as M7

belonged to Turkey. Now M5 and M9 were from different countries and M9

did not belong to France or Switzerland. So M9 belonged to either Germany or Turkey. Now M1 and M3 belonged to the same country and so

did M2 and M4. Let us consider two cases.

Case 1 M9 belonged to Germany.

UK Germany France Switzerland TurkeyM6 M9 M2 M1 M7. M8 M10 M4 M3 M5

Case 2 M9 belonged to Turkey.

UK Germany France Switzerland Turkey M6 M5 M2 M1 M7 M8 M10 M4 M3 M9

(As M5 did not belong to France or Switzerland) Thus M10 belonged to Germany Choice (C) 40. It is given that sum of the costs of the gifts bought by

Sneha and Sushma was equal to the cost of the gift bought by Shikha.

We have four possibilities which satisfies this. Sneha Sushma Shikha Sushmita Case 1. 800 1200 2000 2800 Case 2. 1200 800 2000 2800 Case 3. 800 2000 2800 1200 Case 4. 2000 800 2800 1200 Again the difference between the cost of Sushma’s gift

and Sushmita’s gift was equal to the cost of Sneha’s gift. This is satisfied only in Case 3. Thus the cost of the gift bought was Shikha was `2800 and she bought a pair of shoes. Choice (D)

41. As per the conditions given the different ways in which

the terms for the two contest can be selected as follows:

Debate: PVQ PVT PVS PVR PVT PVRPVR PVS PVU PVU PVR Elocution: PURPUR PURPUT PUS PUSPUT PUT PQS PQT PUQ

Debate: PVS PVT Elocution: PUQ PUQ

Option A is false as can be seen in the following cases: Debate: PVR PVS PVT Elocution: PUQ PUQPVQ

Option B is true. If V and U are in the same category it must be for Debate. We know that U being in debate implies R is not in elocution. Again since only one among S and T can be selected for a particular category, Q must be selected.

Option C is true as can be seen in the following cases: Debate: PVR PVS PVT Elocution: PUQ PUQPVQ

Thus only statement given in option A is false Choice (A)

NETWORKS 1. The cost incurred will be minimum when the distance

travelled is the minimum. The distance travelled is minimum when he takes a bus going via A E D F G H. The minimum cost incurred by him = 5 + 8 + 4 (10) = `53 Choice (3)

2. If the road connecting A to E is under repair, then to incur minimum cost, one must board a bus going via the route A D F G H. Since the total distance travelled along this route is the least. The cost incurred = 5 + 8 + 4 (12) = `61

Choice (3)

QBR (Miscellaneous) 1. Given that the number of people rightly reported is 275.

This includes people under C3 and C4. ∴C3+ C4 = 275 .......... (1)

Page 55: `DATA INTERPRETATION REPLICA QUESTIONS THAT HAVE APPEARED IN CAT IN THE LAST 4 YEARS

17

∴Number of people wrongly reported = 450 –275 = 175 ∴C1+ C2 = 175 ………. (2) Given the number of infected people is 50% that of non-infected ∴Number of infected people = 150 = C1 + C4 ……. (3) And the number of non-infected people = 300 = C1 + C3 ……. (4) Required difference is between C2 and C4, obtained by (2) – (3) ⇒ C2 – C4 = 175 – 150 = 25 Choice (2)

DI (Miscellaneous)

1. Given that for every `2 increase in the selling price

per ball, the number of balls sold decreases by 20. ∴If the selling price of each ball is increased k times, selling price = `59 + 2k. ∴Profit per ball = (59 + 2k) – 50 = 9 + 2k. Number of balls sold = 700 – 20k. ∴Profit obtained = (9 + 2k) (700 +20k). Profit = 10 ((9 + 2k) (70 –2k))

= 10 (630 –18k + 140k –4k2) = 10 (630 – (4k2 –122k))

=

−−22

261

261

k263010

=

−−

+22

261

k2261

63010

∴Profit is maximum when 2k –2

61= 0 ⇒ 2k =

2

61

But since k is an integer, 2k must be an integer.

∴2k can be taken to be 2

62or

2

60

If 2k = 30, profit = 10 ((9 + 30) (70 – 30)) = 10 × 39 × 40 = `115600

If 2k = 31, profit = 10 ((9 + 31) (70 – 31)) = 10 × 40 × 39 = `115600

∴For maximum profit, Selling price = 59 + 2k = 59 + 30 = `89 or 59 + 31 = `90 When selling price is `89, balls sold = 700 – 300 = 400 When selling price is `90, balls sold = 700 – 320 = 380 Of the given choices, only (A) satisfies. Choice (1)

2. Total yield from scheme = 0.25 (–3) + 0.55(80) +

0.2(100) = –7.5 + 44 +20 = 56.5 Therefore the total yield from scheme II was also 56.5. Let the probability of the bearish market be p. ∴The probability of the bullish market = 1 – 0.4 – P = 0.6 – P Now, p (– 10) + 0.4 (60) + (0.6 – p) 100 = 5605 110 p = 60 + 24 – 56.5 ∴p = 0.25 Choice (3)

3. Scheme 1

Market conditions Probability Yield percentage

Bearish 0.2 –30

Steady 0.45 80

Bullish 0.35 100

The yield from scheme = 0.2 (– 30) + 0.45 (80) + 0.35 (100) = – 6 + 36 + 35 = 65 Increase in the total yield from scheme 1

= 1005.56

5.5665 ×−= 15% Choice (4)

Solutions for questions 4 and 5: 4. From the choices only option (a) indicates “low per

capita income and low happiness quotient.” Choice (1)

5. From the choices only option (b) indicates high

happiness quotient and high per capita income”. Choice (2)

Solution for question 6: 6.

Written WE Interview Essay writing GD

Rahul 5 3 2 3 4

Ramya 5 1 3 4 3

The cumulative score of Rahul is 5 × 0.3 + 3 × 0.1 + 2 × 0.25 + 3 × 0.1 + 4 × 0.25= 3.6 The cumulative score of Ramya is 5 × 0.3 + 1× 0.1 + 3 × 0.25 + 4 × 0.1 + 3 × 0.25 = 3.5 The required difference is 0.1

Choice (1) Solutions for 7 and 8: 7. The expected pay-out for Raju is 80 × 0.5 + 40 × 0.3 – 20 × 0.2 = 48 Choice (2) 8. The expected pay out for Ramu is 80 × 0.5 + 60 × 0.3 – 20 × 0.2 = 54

After the change of probability the expected pay out for Ramu is 80 × 0.3 + 60 × 0.5 – 20 × 0.2 = 50

∴The required percentage decrease is 100544 ×

= 7.4% Choice (1) Solutions for question 9: 9. The hotel cost for Ramu = $600 The cost incurred for city tour = $40 The cost incurred for tour of the Hunters valley = $35 Hence the total cost incurred by Ramu = $675 Choice (3) 10. Given that there has to be a male in every group.

Hence only three groups can be formed. Also given P, S are in same group and each group has atleast one JSE and one SSE.

As both P and S are JSE. The team should have one SSE. Given R is in a group of 3 people

Hence the three groups should have 3, 3 and 2 members in each. Now considering the condition one JSE and one SSE in each group we get the following possibilities.

(i) (ii)

Group 1: R X Y R X Y Group 2: Z P S Z Q Group 3: W Q W P S

From the above possibilities we can conclude that X should definitely be a member of a group which has 3 people. Choice (1)

Page 56: `DATA INTERPRETATION REPLICA QUESTIONS THAT HAVE APPEARED IN CAT IN THE LAST 4 YEARS

18

9–x

12–x x

15–x

XAT

FMS CAT

Solutions for questions 11 and 12: 11. Let the value of the number in column 'b' and row 'd' be

'x' and that in column 'b' and row 'b' be 'A'.

Given x = 31

(A + 26 + x)

⇒ 2x = A + 26 ⇒ x = )26A(21 +

As we know the grid contains only integers Therefore x should be an integer. Hence A should be even. From the choices only 16 is possible Choice (4)

12. Given the numbers in column 'a' are squares of the prime numbers, starting with the first odd prime number. Hence the numbers should be 32, 52, 72. 112 and 132. Thus, the required sum is 32 + 52 + 72 +112 + 132 = 373 Choice (4)

Solutions for questions 13 and 14:

Given total number of sarees = 400 The ratio of Kanchipattu, Benarasi and Mangalgiri sarees is 5 : 3 : 2 The number of Kanchipattu sarees = 200 The number of Benarasi sarees = 120 The number of Mangalgiri sarees = 80 Given on day 1 he sells 20% of the total which is 80 on day 2 he sells 200 and on day3 he sells 120. Also on each day he sells the sarees in the same ratio as he bought i.e 5 : 3 : 2

K B M Total Day1 40 24 16 80 Day2 100 60 40 200 Day3 60 36 24 120 Total 200 120 80 400

13. On the 3rd day he sold Benarasi saree at `480. ∴Total amount received by him on the 3rd day is 60 × 350 + 36 × 480 +24 × 375 = `47280 Choice (2) 14. 25% of the total number of sarees were slightly

damaged.

⇒ 41

(400) = 100

The ratio of the damaged sarees of each type is 5 : 3 : 2. Hence the number of damaged Kanchipattu sarees = 50

The number of the damaged Benarasi sarees = 30 The number of the damaged Mangalgiri sarees = 20 He sold all the damaged sarees at 20% loss. ∴ Total amount = 50 × 280 + 30 × 320 + 20 × 300 = `29600 Choice (3) Solutions for question 15:

15. The population of China in 2009 = 1.6 billiion As it increases by 12% per annum it becomes 5.56 billion in 2020 The population of China in 2020 is 15% of the total population

Hence the total population is 15.056.5

= 37 billion

Choice (2)

Solutions for questions 16 and 17: 16. The graph gives visibility index of 26 people. The visibility index of 14 people are more than U.

Hence the required ratio is 1002614 × = 53.8 ∽ 54%

Choice (2)

17. By observing we can easily find that R and S have the same visibility index. Choice (3)

Solutions for question 18: 18. The points of A = 1 × 30 + 2 × 20 + 2 × 10 + 1 × 5 = 95

The points of B = 2 × 30 + 1 × 20 + 1 × 10 + 3 × 5 = 105 The points of C = 3 × 30 + 2 × 20 + 1 × 10 + 1 × 5 = 115

The points of E = 1 × 30 + 2 × 20 + 1 × 10 = 80 ∴The winner is C Choice (3)

Quant SI – CI

1. Let the number of years after which his interest in scheme 3 will be more than his interest from scheme 2 be n.

Interest from scheme 2, Ι2 = =n)10000(100

151500n.

Interest from scheme 3, Ι3= 10000n2

200

101

+ – 10000.

For n = 8, Ι3 = `11829 & Ι2 = `12,000 For n = 9, Ι3 = `14066 & Ι2 = `13,500 ∴After 9 years, Ι3 > Ι2. Choice (2)

2. Let us consider the interests received by him from the

four schemes across the year with `1000 invested in each scheme.

Scheme 1 Scheme 2 Scheme 3 Scheme 4

Year 1 80 150 102.5 80 Year 2 166.4 300 216 166.4 Year 3 259.7 450 340 259.7 Year 4 360 600 477 360 Year 5 469 750 629 469 Year 6 587 900 796 587 Year 7 714 1050 980 714 Year 8 851 1200 1183 851

In scheme 4, amount at the end of the year = 1000 (1.2) = 1200. Amt. remaining after paying the administrative charges = 0.9 (1200) = 1080 Amt. at the end of the second year = 1080(1.2) = 1296 Amt. remaining after paying the administrative charges = 1296 (0.9) = 1166.4 This scheme is similar to scheme 1. Therefore scheme 2 produces the maximum interest at the end of 8 years. Choice (2)

LA (Venn Diagram)

1. Let the number of students who applied for all three examinations be x. The number of students who applied for at least 2 of the 3 examinations = 36 – 2x

It is given that 25% (36 –2x) = x 36 – 2x = 4x ⇒ x = 6 So the completed venn diagram will be as follows.

Page 57: `DATA INTERPRETATION REPLICA QUESTIONS THAT HAVE APPEARED IN CAT IN THE LAST 4 YEARS

19

3

6

11

6 9

18 9

XAT

FMS CAT

Speak in Hindi (480) Speak in English (500)

Own a car (400)

C = 180

d = 60

e = 240

t = 40

A B

120

The Times of India

The Hindu

A B

C

d

e f 9

The Telegraph

21

Number of students in the class = 18 + 3 + 6 + 9 + 6 + 11 = 62 Choice (3)

Solutions for questions 2 and 3: As per the data provided in the question, A + e + d = 360 (1) B + e + f = 380 (2) C + d + f = 280 (3) d + c = 24 (4) (3) – (4) gives f = 40 Again d + 120 = 180 ⇒ d = 60 ∴ c = 180 From (1), A + e = 300 and from (2), e +B = 340. Now A + e + e + B = 400 [ the total number of persons = 800] A + e + e + B = A + e + B + e = 300 + 340. ∴ e = 240 ∴ A = 60 and B = 100

2. The number of persons who can speak in both Hindi and English = 240 + 120 = 360

The required percentage = 500360

× 100 = 72%

Choice (3)

3. The proportion of people in the locality who do not own

a car or cannot speak in English = 800

CdAeB ++++

= 800640

= 0.8 Choice (4)

4. A + B + C + D +E +F = 120 – (21 + 9) = 90

We have the following information. The Hindu was read by 64 families. ∴ A + d + e = 55 (1) The times of India was read by 48 families. ∴ B + d + f = (2) The Telegraph was read by 45 families. ∴ C + e + f = 36 (3) Adding equations (1), (2) and (3), we get A + B + C + 2 (d + e+ f) = 130 Again A + B + C + d + e + f = 90 ∴ d + e + f = 40 ⇒ A + B + C = 90 – 40 = 50 Therefore exactly one newspaper was read by 50 families. Choice (3)

Solutions for questions 5 and 6: Given GT = 300 → 1 a + d + f + g = 60 → 2 b + d + e + g =120 → 3 c + e + f + g = 180 → 4 a + d = f + g → 5 a = f = 0 → 6 c + f = 30 → 7 Equations 5 and 6 ⇒ d = g Equations 6 and 7 ⇒ c =30 Using GT formula a + b + c + d + e + f + g + n = 300 ⇒ 0 + (b + d + e + g) + 30 + 0 + n = 300 ⇒ n = 60 Using equations 2, 5 & 6, we get d = g = 30.

5. 10030030

100GTg ×=× = 10 %

Choice (2) 6. From 4 and c = 30, we get e = 120

Substituting in 2, Eqn. we get b = 30 Therefore b + n = 90.

Choice (4) 7. Given, of the 300 students, 70 choose MS

Hence 230 choose MBA. Given g = 20 → 1

n = 0 a + d + f + g = 100 → 2 b + e + d + g = 150 → 3 d = 2f = g ⇒ d = 20, f = 10 → 4 From 1, 3 and 4, we get b + e = 110 GT = a + b + c + d + e + f + g + n 230 = 100 + b + e + c ⇒ c = 20 Choice (3)

8. Given the number of children who buy T & J (A) = 16

The number of children who buy C & H (B) = 26 The number of children who buy B & B (C) = 34 We know A + B + C = Ex1 + 2Ex2 + 3Ex3

Where Ex1, Ex2 and Ex3 denote the number of children buying exactly one, exactly two and exactly three toys respectively Given every child buys exactly 2 toys Hence Ex1 = Ex3 = 0 ⇒ 76 = 2Ex2

∴Ex2 = 38 Hence there are 38 children who visited the shop

Choice (4) Solutions for questions 9 and 10: Given P = 24 ……… (1) Q = 36 ……… (2)

d

e

c

gf

a b

C

S H

GT 300

n

d

e

c

g f

a b

H

FM

230

n = 0

Page 58: `DATA INTERPRETATION REPLICA QUESTIONS THAT HAVE APPEARED IN CAT IN THE LAST 4 YEARS

20

P Q

R

S

a e b

f

k

j n i d h

c o

l g m

P

R = 29 ……….. (3) S = 25 ……….. (4) e + l + o + n = 12 …….. (5) f + l + k + o = 18 ……… (6) g + m + l + o = 16 .......... (7) K = 8 and l + o = 6 …….. (8) From (5) & (8) we get e + n = 6 From (1) & (6) we get a + e + f + l + k + o + j + n =

24 a + e + n + j = 6 but e + n = 6 ⇒ a = j = 0 As the people who like S also like R. Hence d = i = n = j = 0. h + m + k + o = 25 Therefore e = 6 and b = 4.[As b + (g + l + m + o)= 36, where

(g + l + m + o) = 16] From (6) & (8) f = 4 From (7) & (8) g + m = 10 From (3) c + h + g +m + l + o + f + k = 29 c + g + l + f + 25 = 29 c = g = l = 0 Therefore h = 1, m = 10 and o =6 9. The number of people who like only Q = 14 Choice (4) 10. The number of people who like all 4 movies is 6 Choice (2)

LA (Miscellaneous)

1. Ranking of the stores and the total net scores for the pizza stores are as follows.

Stores Ranking as per price.

Ranking as per delivery

time Total net score

A 3 3 3(0.7) +3(0.3) = 3.0 B 6 2 6 (0.7) +(0.3) = 4.8 C 1 1 1(0.7) + 1 (0.3) = 1.0 D 2 6 2(0.7) + 6(0.3) = 3.2 E 5 3 5(0.7) + 3 (0.3) = 4.4 F 3 5 3(0.7 + 5(0.3) = 3.6

Therefore pizza store D got the third lowest net score. Choice (2) Solutions for Questions 2 to 4: 2. The least score obtained by a person in the four papers

can be obtained as follows.

Paper 1 Paper 2 Paper 3 Paper 4 Correct 4 3 1 4 Wrong 0 1 3 0 Score 60 45 –20 80

Therefore the minimum net score that the person can get is 165. Choice (4)

3.

Paper 1 Paper 3 Correct 2 4 Wrong 2 0 Score 20 100

Total score = 120

Paper 2 Paper 4 Correct 4 1 Wrong 0 2 Score 110 30

Total score = 140

Therefore the difference between the highest marks obtained is 20. Choice (1)

4. The least score is obtained when the person attempts

the following 3 papers.

Paper 1 Paper 2 Paper 4 Correct 4 2 4 Wrong 0 2 0 Score 60 10 80

Total score = 150 Choice (2)

Solutions for questions 5 and 6: The final arrangement of the persons in the 9 seater van was as follows:

1

U

2

X

3

Q 4

W

5

T

6

R 7

S

8

P

9

V

5. After the given swapping the final arrangement will be

as follows:

1

W

2

T

3

Q

4

S

5

X

6

U

7

R

8

P

9

V

Therefore X will be seated beside U

6. After the given swappings the final arrangement is as

follows:

1

X

2

V

3

Q 4

S

5

U

6

R 7

T

8

P

9

W

From the choices only "W is in the 9th seat" is correct.

Choice (2) Solutions for questions 7 to 9: 7. The proportion of residents who prefer watching movies

PA is 0.65 The proportion of residents who prefer surfing the net, PB is 0.68 The proportion of residents who prefer doing both, PA∪B is 0.61 The proportion of residents who prefer at least one between watching movies or surfing net is PA + PB –PA∩B = 0.65 + 0.68 – 0.61 = 0.72 ∴Proportion of residents who neither watch movies nor surf net is 1 – 0.72 = 0.28

8. Let the population of A be 3k.

⇒ Population of B is 5k, that of C is 3k and that of D is 4k.

Page 59: `DATA INTERPRETATION REPLICA QUESTIONS THAT HAVE APPEARED IN CAT IN THE LAST 4 YEARS

21

A/H

B

F

C

D

E

G

H/A H/F

F/H

B

A

C

D

E

G

T P

W

Q

S

X V

R

Y

U

T P

W

Q S

X

V R

Y U

g a b

P T

GT

n

∴No. of residents who prefer chatting with friends in A is 0.36 × 3k = 1.08k in B is 0.45 × 5k = 2.25k in C is 0.32 × 3k = 0.96k in D is 0.25 × 4k = 1.00k ∴Highest number is in B Choice (2)

9. No. of residents who prefer chatting with friends was calculated in the previous question. The average number of residents who prefer surfing net

= 4

k3k04.2k75.2k9.1 +++= 2.4275k

∴ 2 colonies have more than the average number. Choice (3)

10. The different ways in which oil can be transferred from tank B to tank H are 1. B E A F D C G H 2. B E A F G H 3. B E C A F G H 4. B E C G H 5. B E D A F G H 6. B E D C A F G H 7. B E D C G HS Thus there are seven possibilities in all.

Choices (c) 11. Based on the conditions given in the question, we get

the following possibilities.

Arun Varun Kiranmala 1. India-Day-to-Day India-Every-day India-These-days 2. India-These-Days India-Day-to-Day India-Every-day

We can conclude that Varun did not subscribe to India-These Days. Choice (3)

12. If Kiranmala did not subscribe for India-These Days, then Varun subscribed for India-Day to Day.

Choice (1)

Solutions for question 13: 13. Given out of the six baskets four baskets have either

gold or silver.

Hence the probability of Rajini taking something home

is 32

C

C

16

14

= [since all the baskets are equally likely to

get selected by Rajini] Choice (2)

Solutions for question 14:

14. Let us consider P = 1. When there is one goat and one tiger then the tiger eats the goat and gets transformed into a goat and stays happily in the forest. Now if P = 2 when there are two tigers. Now, if one of the tigers eat the goat then it gets transformed into a goat and then the second tiger would kill it. Hence when two tigers are there they would not kill the goat. Let P = 3. When three tigers are there. One of the three tigers kills the goat and becomes a goat. Hence the remaining tigers would not kill the goat. Hence when the tigers are odd numbered then they would kill the goat, else the goat is not eaten by any tiger. Choice (2)

Solutions for questions 15 and 16: Given a + g = 12

b + g = 8

15. g = 4

The number of days that Ram learnt an instrument is a + b + g = 16 Choice (4)

16. Given a = 6 Hence g = 6 and b = 2 He learnt an instrument on a + b + g i.e 14 days in all. Choice (3)

17. From the given pattern we can understand that it is a cyclic pattern.

Hence the input is repeated in every 7th step. Therefore step 28 would be the same as the input Choice (4)

LA (Circular Arrangements)

1. The sitting arrangement was as follows

Therefore from the above possibilities we can conclude that H is sitting opposite to C or D. Choice (4)

2. From option (a)

U cannot see P, W and Q, so option (a) is not correct From option (b)

Page 60: `DATA INTERPRETATION REPLICA QUESTIONS THAT HAVE APPEARED IN CAT IN THE LAST 4 YEARS

22

T

P W

Q

S

X

V R

Y U

T P

W

Q

S

X

V

R Y

U

H

M

B

H

M

B

D

R/T

T/R

R/T

T/R

P S

All the conditions are satisfied, so (b) can be the answer From option (c)

T cannot see y, so option (c) is not correct From option (d)

Bring the line below the diagram U cannot see P, so Choice (4) is not correct Choice (2)

Solutions for questions 3: Using the first clue we can draw the figure as follows: Using the 2nd clue we get,

Hence we can say that the Mumbai co-ordinator is opposite the Delhi co-ordinator. Choice (1) Solutions for question 4: 4. Given R and T sit together. We can arrange them in 2

ways. P and S do not sit together, so we can arrange them in 6 ways.

We can arrange the remaining two employees Q & U

in 2 ways. Therefore, the total number of ways = 2 × 6 × 2

= 24 ways Alternate solution: Considering R and T as a single unit, we get 3 units

(R, T), Q and U which can be arranged around the table (3 – 1)! × 2! ways [2! since RT can be arranged among the themselves] Now the remaining 2 persons can be placed in 2 of the 3 positions in 3C2 × 2! ways.

Therefore the total number of arrangements = (3 – 1)! (2!) (3C2) 2! = 24 ways. Choice (B)

LA (Distribution) Solutions for questions 1 to 3: From the given data we can conclude that the twelve persons were living in the building as follows.

Guitarist Singer Instrumentalist Drummer Singer Drummer Key board player Singer Guitarist Guitarist Singer Instrumentalist

F S B E/B T G/E Q C R D P A

Floor 1 Floor 2 Floor 3 Floor 4 Floor 5 Floor 6 Floor 7 Floor 8 Floor 9 Floor 10 Floor11 Floor 12

R/T

T/R P

S

R/T

T/R P

S

Page 61: `DATA INTERPRETATION REPLICA QUESTIONS THAT HAVE APPEARED IN CAT IN THE LAST 4 YEARS

23

1. The four singers were S, T, C and P. Therefore only one male singer was there in the band. Choice (1)

2. Above the floor in which G lived there were 6 floors or 8

floors. Choice (4) 3. S lived in the second floor. Choice (2) Solutions for questions 4 and 5: 4. From the given data, the only possibility is 1993 November 5th

1994 1995 1996 February 29th 1997 April 23rd, October 15th 1998 November 15th 1999 January 3rd. Both the couples who got married in the same month got married in the month of November. Choice (3)

5. The least difference between the marriage dates of any

two couples was between the couple who got married

on November 15th, 1998.and the one who got married on January 3rd, 1999. And it was 49 days. Choice (1)

6. Based on the conditions given, the groups are as

follows: Jalan, Kokila and Kadambar

Jagan, Kavya and Kavita, Jeevan, Kavya and Kekul Therefore Jagan is in the same group as Kavya and Kavita Choice (1)

7. Given B went to college on Thursday and did not teach

Physics. As A, B and C did not teach Physics, we can conclude that D taught Physics. As A and C went to college on consecutive days it can be either on Monday and Tuesday, or Tuesday and Wednesday. They cannot go to college on Tuesday and Wednesday as Physics is taught after Chemistry. Hence A and C go to college on Monday and Tuesday respectively. ∴Biology is taught on Monday.

Choice (1)

Solutions for questions 8 and 9:

Given Team I scored maximum number of points, 364 is at the 15th place and Team A got 361 points. Given the sum of the points scored by teams at (13 + 14 + 15) is 1046. 364 + 361 + x = 1046 ⇒ x = 321 Hence Team A is in the 14th place. As team C got 218 points and is in the 10th place and Team O got 251 points. Hence Team O should be placed between 11 -13. Given the ascending order of teams according to their points is O J F. Hence Team O is in the 11th place, team J is in the 12th place and Team F is in the 13th place with 321 points. Now we know the points of the teams in the 10th and the 11th place. Hence the points of the team in the 12th place is 284 points (from 10 + 11 + 12 = 753). As Team N and team B got 108 and 165 points respectively less than team F. Team N got 213 points and Team B got 156 points. The clue 7 + 8 + 9 = 590 implies that we can calculate the points of the third team in the above group. [We know N is one of the teams in the group] Hence 213 + 182 + x = 590. ⇒ x = 195 points. Hence Team N is in the 9th place. The clue Team E got 18 points less than Team N, implies that team E is in the 8th place. From the clue 4 + 5 + 6 = 412, we know B is one of teams in the group. Hence 116 + 156 + x = 412 ⇒ x = 140 Hence Team B is in the 6th place. The clue Team H got 4 points more than Team M, implies that Team K is in the 5th place. From the clue 1 + 2 + 3 = 302, we get 96 + x + 4 + x = 302 ⇒ x = 101 Hence Team M got 101 points and is in the 2nd place. Therefore Team H got 105 points and is in the 3rd place. The final arrangement is as follows.

1st 2nd 3rd 4th 5th 6th 7th 8th 9th 10th 11th 12th 13th 14th 15th Team L M H D K B G E N C O J F A I Points 96 101 105 116 140 156 182 195 213 2182 251 284 321 361 364

8. The required difference is 284 – 140 = 144 Choice (2)

9. The position of team E is 8th . Choice (2)

Solutions for questions 10 and 11:

Given P wears the Orange shirt and he orders Sprite and the person wearing the Green shirt orders Pepsi. From the clues, we know R wears the Red shirt, U wears the Blue shirt, T drinks Thumsup and Q drinks Coke. Hence R, Q, T and U neither wear Green nor order Pepsi. So the Green shirt is worn either by S or V. But from the clue, the person wearing Green shirt is the person who

orders Fanta and V orders the same dish. Hence we know that V does not wear the Green shirt. Therefore S wear the Green shirt and he orders Pepsi.

Let us tabulate the data.

Shirt Drink P Orange Sprit Q Coke R Red S Green Pepsi T Thums up U Blue V

Page 62: `DATA INTERPRETATION REPLICA QUESTIONS THAT HAVE APPEARED IN CAT IN THE LAST 4 YEARS

24

O SA

L SL

O SA

L SL

P DC

IND RCB

O SA KKR

L SL DD

P AUS, DC

M IND, RCB

N ENG,MI

Now from the last clue the person wearing the Violet shirt ordered Maaza. Hence V ordered Maaza and is wearing the Violet shirt. 10. S drinks Pepsi. Choice (2) 11. V Violet shirt, Maaza is the correct combination. Choice (1) Solutions for questions 12 and 13: From the first and the last clue we get the following arrangement. From the third clue we know that the cricketer from India is at the extreme left end of the row and he plays for RCB. Also P is to the immediate right of him who plays for DC. Now from the 2nd clue we know that N is to the immediate left to the player who plays for KKR. From the above consideration we get the final arrangement as 12. M plays for RCB and comes from India. Choice (1) 13. The player from Australia plays for DC. Choice (4) Solution for questions 14 and 15: 14. Given X gave the presentation before T, R gave the

presentation before V but after U, also W gave the presentation after P and S but before U and X. So, Q, U, X, T, R and V (need not be in the same order) conducted the seminars after W. Hence W should be giving his presentation in group 1 with P and S. Choice (1)

15. From the choices we can say that X, U and Q can be

the first person to give the presentation in 2nd group but V cannot be the person. Choice (4)

16. Given R attends the Physics tuition, Q attends the

Maths tuition. Also P and U same tuition, T and V attend same tuition and S does not attend the same tuition as Q. Hence S should attend a tuition with at least 2 students in it. i.e. S should either attends Physics or Chemistry as there should be at least 2 students in each tuition. Q should attend a tuition to which 3 people go. Choice (4)

Quant ERPV

1. The ratio of the floor areas of A1 and A2 is 1 : 4

Now the total work required to complete the work in

A1 = N61

(D) + 4N

D

65

= 249

ND, where D = no. of

hours ∴ The total work required to complete the work in A

must be 4

ND

249

.

It is given that the total work required to complete work

in A2 = 43

N

D

65

+ N

D

87

=2436

(ND)

This will be true for infinite values of N. Thus a unique value of N cannot be determined from the given information. Choice (4)

Line + Bar graph

Solutions for questions 1 to 4: 1. The number of accidents caused because of two

wheelers is 10028

× 75000 = 21000.

The accident severity index for two wheelers is 40. i.e. for every 100 accidents, 40 persons are killed. So for 21000 accidents 210 × 40 = 8400 persons are killed. Choice (3)

2.

Total accidents

Persons killed

Persons injured x

Trucks 19500 6630 12870 0.52

Bus 13500 4050 9450 0.43

Car 12000 4200 7800 0.54

Two wheelers 21000 8400 12600 0.67

Others 9000 4050 4950 0.82

x = the ratio of the persons killed to the persons injured. The required ratio is the highest for other types of accidents. Choice (2)

3. The number of people who got injured by car accidents

was 7800 Choice (2) 4. The number of persons killed in truck is 6630 and

number of persons injured in other type of accidents is 4950. The required difference is 1680 Choice (4)

DI (Distribution)

Solutions for questions 1 and 2: As policies mature in between 1997 and 2002 and a policy matures on Feb 29th, the policy should mature on 29th Feb 2000. Now the policy which matures on Jan 10th matured after Feb 29th. Hence the policy on Jan 10th can be in 1997 or in 2002. But from one of the clues the policy on May 21st is the last and the Sep 17th policy is immediately before the May 21st policy. Hence the policies on Sep 17th and Jan 10th should mature the same year. The policy on August 8th is before Feb 29th. Hence August 8th policy is for either 98 or 99. The final arrangement is as follows.

1st 2nd 3rd 4th 5th 6th

24th Aug' 1997

8th Aug 1998/1999

29th Feb2000

10th Jan 2001

17th Sep 2001

May 21st

2002

1. The third matured policy is on 29th Feb 2000

Choice (1) 2. In the year 2001, Atul receives money from two policies.

Choice (2)

Page 63: `DATA INTERPRETATION REPLICA QUESTIONS THAT HAVE APPEARED IN CAT IN THE LAST 4 YEARS

25

S R

S R V/U U/V

S R V/U U/V P T Q

LA(Linear Arrangement) Solutions for questions 1 to 3: Given S is in 3rd place from the left end and the positions of F, Q, G are also given. As two of F, Q, G are from USA and there is atleast one person between any two friends from USA, we can say that F and G are from USA. Also the extreme ends are occupied by friends from USA. Now the friends from UK are separated by atleast four friends. So the other friend from UK can come either in the 2nd or the 4th position from the right. But from one of the clues P is from Australia and is in between Ι and J. Hence we can get it as. Now as R is adjacent to Ι and the friends at extreme ends are of different gender, we can get the final arrangement as follows. 1. F, G, Ι and J stay in the USA. Therefore four female

friends stay in the USA. Choice (4) 2. Three friends are in between the friends from the UK Choice (1) 3. T is the only male friend from USA. Choice (2) Solution for question 4 and 5: Given from (i) and (iv), the arrangement would be From the (iii) clue, U and V have 2 persons in between them. Hence we have only one possibility.[since there is only one person between Tarun and Qureshi] From the (ii) clue we get the final arrangement as 4. Raju is 3 places away to the left of Qureshi. Choice (4) 5. Pradip is to the immediate right of Shyam. Choice (3)

LA (Sequencing) Solutions for 1 to 3: From the clue, D got less than 3 other students hence D got the 4th rank. From the other clue E > C > A. Let us assume B got the highest marks. Then the order is B > E > C > A.

Now D got the 4th rank ⇒ D gets less than C. But the condition that one pair should have same marks is not satisfied. Hence E should get the highest marks. So E > C > A Now D cannot have more marks than C as D should have marks less than 3 students, hence C > D. As the first and the last ranked students do not have same marks as any other student, B should get the same marks as 'C'. Hence all the conditions are satisfied and the final arrangement is E > C = B> D > A. 1. E got the highest marks. Choice (4) 2. B and C got the same marks. Choice (2) 3. The descending order is ECBDA. Choice (1) Solutions for questions 4 and 5: Given at least 2 movies were released before R and there is one movie released between S and T. Hence R cannot be released on the 3rd or the 4th week of the month. It can only be released on the last week. Therefore Q is released on the 4th week. As S and T have one movie released between them, P is released on the 2nd week. Hence the final order would be as S/T, P, T/S, Q, R. 4. R is released last. Choice (3) 5. Only one movie is released before P. Choice (1)

Line Graph + Table Solutions for question 1: 1. From the table we can identify that the profit/ton in 2000

is the highest for Steel. From the line graph it is clear that the production of Steel is the highest.

Hence the profit of Steel should be the highest. Choice (4)

S UK

F Q G

S UK USA

F USA

Q G USA USA USA

S UK USA

F USA

Q Aus

G USA

Ι/J USA

J/Ι USAAus UK

P Aus

S UK

T USA

F USA

Q Aus

G USA

Ι USA

J USA

H Aus

R UK

P Aus